CAT Previous Year Paper 2003

CAT 2003  

Data Interpretation 

Instructions 

DIRECTIONS for the following three questions: In each question, there are two statements: A and B, either of which can be true or false on the basis of the information given below. 

A research agency collected the following data regarding the admission process of a reputed management school in India.

Q. 1 Statement A: The success rate of moving from written test to interview stage for males was worse than for females in 2003. 

 

Statement B: The success rate of moving from written test to interview stage for females was better in 2002 than in 2003. 

A Only Statement A is true. 

B Only Statement B is true. 

C Both the Statements are true. 

D Neither of the two Statements is true. 

Answer: D 

Explanation: 

The success rate of moving from written test to interview stage for males in 2003 can be given by 637/60133 and for females in the same year can be given by 399/40763 . We can see that the rate of males is clearly more that that of females. hence statement A is false. 

Now the success rate of moving from written test to interview stage for females in 2002 was 138/15389 and for females in the year 2003 was 399/40763.So the rate in 2002 is less than that in 2003. . Hence both statements are false. 

Q. 2 Statement A: In 2002, the number of females selected for the course as a proportion of the number of females who bought application forms, was higher than the corresponding proportion for males. 

 

Statement B: In 2002, among those called for interview, males had a greater success rate than females. 

A Only Statement A is true. 

B Only Statement B is true. 

C Both the Statements are true. 

D Neither of the two Statements is true. 

Answer: D 

Explanation: 

In 2002, the number of females selected for the course as a proportion of the number of females who bought application forms was 48/19236 and for males was 171/61205. So rate for males was higher than that of female. Hence option A is false. 

In 2002, among those called for interview, the success rate for females was 48/138 and for males 171/684. So the rate was higher for females .Both are false. Hence option D. 

 

Q. 3 Statement A: The percentage of absentees in the written test among females decreased from 2002 to 2003. 

 

Statement B: The percentage of absentees in the written test among males was larger than among females in 2003. 

A Only Statement A is true. 

B Only Statement B is true. 

C Both the Statements are true. 

D Neither of the two Statements is true. 

Answer: A 

Explanation: 

The percentage of absentees in the written test among females in 2002 was 3847/19236 and in 2003 was 4529/45292. Thus percentage of absentees decreased from 02 to 03. Hence statement A is true. 

The percentage of absentees in the written test among males in 2003 is 3065/63298 which is clearly less than that of female in 2003. Hence statement b is false . Hence option A. 

 

Instructions 

DIRECTIONS for the following three questions: Answer the questions on the basis of the information given below. The length of an infant is one of the measures of his/her development in the early stages of his/her life. The figure below shows the growth chart of four infants in the first five months of life. 

Q. 4 After which month did Seeta’s rate of growth start to decline? 

A Second month 

B Third month 

C Fourth month 

D Never 

Answer: B 

Explanation: 

The rate of growth is the slope of the line. In the second and third months, there is an increase in the slope of the line. After the third month, there is a fall in the slope of the line i.e., the rate of growth is declining. Therefore, option b) is the correct answer. 

Q. 5 Who grew at the fastest rate in the first two months of life? 

A Geeta 

B Seeta 

C Ram 

D Shyam 

Answer: A 

Explanation: 

The rate of growth is the slope of the line. 

From the graphs, we can see that the slope of the line is maximum for Geeta (red line) in the first two months. So, option a) is the correct answer. 

Q. 6 The rate of growth during the third month was the lowest for 

A Geeta 

B Seeta 

C Ram 

D Shyam 

Answer: A 

Explanation: 

The rate of growth is the slope of the line. During the third month, Geeta has a negative slope whereas all the others have a positive slope. So, the rate of growth of Geeta is the least. Option a) is the correct answer. 

Q. 7 Among the four infants, who grew the least in the first five months of life? (in %) 

A Geeta 

B Seeta 

C Ram 

D Shyam 

Answer: B 

Explanation: 

Seeta’s growth percentage = (57-50)/50 * 100% = 14% 

Geeta’s growth percentage = (61-49)/49 * 100% = 24.5% 

Ram’s growth percentage = (61-52)/52 * 100% = 17.30% 

Shyam’s growth percentage = (61 – 53.5)/53.5 * 100% = 14.01% 

So, Seeta’s growth percentage is the least. 

 

Instructions 

DIRECTIONS for the following three questions: Answer the questions on the basis of the information given below. 

The table below provides certain demographic details of 30 respondents who were part of a survey. The demographic characteristics are: gender, number of children, and age of respondents. The first number in each cell is the number of respondents in that group. The minimum and maximum age of respondents in each group is given in brackets. For example, there are five female respondents with no children and among these five, the youngest is 34 years old, while the oldest is 49. 

Q. 8 The percentage of respondents aged less than 40 years is at least 

A 10% 

B 16.67% 

C 20.0% 

D 30% 

Answer: D 

Explanation: 

The number of people with age less than 40 are at least: 

In no of children 0 category => 1 male and 1 female 

In no of children 1 category => 1 male and 1 female 

In no of children 2 category => 1 male and 1 female 

In no of children 3 category => 2 male and 1 female 

There are at least 9 people who are less than 40. 

9 people out of 30 people => 30% 

Q. 9 Given the information above, the percentage of respondents older than 35 can be at most 

A 30% 

B 73.33% 

C 76.67% 

D 90% 

Answer: C 

Explanation: 

There are at most 23 people with age greater than 35 . Hence (23/30)*100 = 76.67 . So option C. 

Q. 10 The percentage of respondents that fall into the 35 to 40 years age group (both inclusive) is at least 

A 6.67% 

B 10% 

C 13.33% 

D 26.67% 

Answer: C 

Explanation: 

There are at least 4 respondents that fall into the 35 to 40 years age group (both inclusive) . So 400/30 = 13.333 % . Hence , the percentage of respondents that fall into the 35 to 40 years age group (both inclusive) is at least 13.333%. 

 

Instructions 

DIRECTIONS for the following three questions: Answer the questions on the basis of the information given below. 

Spam that enters our electronic mailboxes can be classified under several spam heads. The following table shows the distribution of such spam worldwide over time. The total number of spam emails received during December 2002 was larger than the number received in June 2003. The total number of spam emails received during September 2002 was larger than the number received in March 2003. The figures in the table represent the percentage of all spam emails received during that period, falling into those respective categories. 

 

Q. 11 In which category was the percentage of spam emails increasing but at a decreasing rate? 

A Financial 

B Scams 

C Products 

D None of the above 

Answer: C 

Explanation: 

It is clear from the table that for the products category the percentage of spam mail is increasing from sep02 to June 03 . But the rate at which it is growing is decreasing. Hence option c . 

 

Q. 12 In the health category, the number of spam emails received in December 2002 as compared to June 2003. 

A was larger 

B was smaller 

C was equal 

D cannot be determined 

Answer: A 

Explanation: 

In dec 2002 , 19 % of all spam mails were from the health category and In jun 2003 , 18 % of all spam mails were from the health category. Now since the total spam mails received in dec 02 were greater than that in jun 03 . Number of spam mails in the health category in dec 02 will be greater than that in jun 03. 

 

Q. 13 In the financial category, the number of spam emails received in September 2002 as compared to March 2003. 

A was larger 

B was smaller 

C was equal 

D cannot be determined 

Answer: D 

Explanation: 

Since we don’t know by how much more total spam mails were received in Sep 02 as compared to mar 03 we can’t compare the number of spam emails in the financial category received in September 2002 as to that in March 2003. Hence option D. 

 

Instructions 

DIRECTIONS for the following three questions: Answer the questions on the basis of the information given below. 

One of the functions of the Reserve Bank of India is to mobilize funds for the Government of India by issuing securities. The following table shows details of funds mobilized during the period July 2002 – July 2003. Notice that on each date there were two rounds of issues, each with a different maturity. 

Q. 14 How many times was the issue of securities under-subscribed, i.e., how often did the total amount mobilized fall short of the amount notified? 

A 0 

B 1 

C 2 

D 3 

Answer: B 

Explanation: 

Only one security issued on 17 July 2002 was undersubscribed. 

Q. 15 Which of the following is true? 

A The second round issues have a higher maturity than the first round for all dates. 

B The second round issue of any date has a lower maturity only when the first round notified amount exceeds that of the second round. 

C On at least one occasion, the second round issue having lower maturity received a higher number of competitive bids. 

D None of the above three statements is true. 

Answer: C 

Explanation: 

The security issued in June 2003 had lower maturity in 2nd round but no. of bids were more . Hence option C. 

 

Q. 16 Which of the following statements is NOT true? 

A Competitive bids received always exceed non-competitive bids received. 

B The number of competitive bids accepted does not always exceed the number of non-competitive bids accepted. 

C The value of competitive bids accepted on any particular date is never higher for higher maturity. 

D The value of non-competitive bids accepted in the first round is always greater than that in the second round. 

Answer: D 

Explanation: 

For the security issued in 4th June 2003 , the value of non-competitive bids accepted in the first round islesas than that in the second round Hence option D is not true. 

 

Instructions 

DIRECTIONS for the following three questions: Answer the questions on the basis of the information given below.

Each point in the graph below shows the profit and turnover data for a company. Each company belongs to one of the three industries: textile, cement and steel. 

 

Q. 17 For how many companies does the profit exceed 10% of turnover? 

A 8 

B 7 

C 6 

D 5 

Answer: B 

Explanation: 

There are in total 7 companies – 3 steels , 2 cement , 2 textile for which the profit is more than 10% of the turnover. 

 

Q. 18 For how many steel companies with a turnover of more than 2000 is the profit less than 300? 

A 0 

B 1 

C 2 

D 7 

Answer: C 

Explanation: 

There are 2 steel companies with turnover of more than 2000 and having profit less than 300. 

 

Q. 19 An investor wants to buy stock of only steel or cement companies with a turnover more than 1000 and profit exceeding 10% of turnover. How many choices are available to the investor? 

A 4 

B 5 

C 6 

D 7 

Answer: B 

Explanation: 

There are 3 steel and 2 cement companies with a turnover more than 1000 and profit exceeding 10% of turnover. hence there are 5 choices for the investor. 

Instructions 

DIRECTIONS for the following three questions: Answer the questions on the basis of the information given below. Details of the top 20 MBA schools in the US as ranked by US News and World Report, 1997 are given below. 

 

Q. 20 Madhu has received admission in all schools listed above. She wishes to select the highest overall ranked school whose annual tuition fee does not exceed $23,000 and median starting salary is at least $70,000. Which school will she select? 

A University of Virginia. 

B University of Pennsylvania 

C Northwestern University 

D University of California – Berkeley 

Answer: D 

Explanation: 

From table it is clear that for University of California – Berkeley annual tuition fee does not exceed $23,000 and median starting salary is at least $70,000. Hence option D. 

 

Q. 21 In terms of staring salary and tuition fee, how many schools are uniformly better (higher median starting salary AND lower tuition fee) than Dartmouth College? 

A 1 

B 2 

C 3 

D 4 

Answer: B 

Explanation: 

There are only 2 colleges – New York university and stanford university which are uniformly better (higher median starting salary AND lower tuition fee) than Dartmouth College. 

 

Q. 22 How many schools in the list above have single digit rankings on at least 3 of the 4 parameters (overall ranking, ranking by academics, ranking by recruiters and ranking by placement)? 

A 10 

B 5 

C 7 

D 8 

Answer: D 

Explanation: 

We can clearly make out from the given table that there are 8 schools in the list which have single digit rankings on at least 3 of the 4 parameters. 

 

Instructions 

DIRECTIONS for the following three questions: Answer the questions on the basis of the information given below.

Table A below provides data about ages of children in a school. For the age given in the first column, the second column gives the number of children not exceeding the age. For example, first entry indicates that there are 9 children aged 4 years or less. Tables B and C provide data on the heights and weights respectively of the same group of children in a similar format. Assuming that an older child is always taller and weighs more than a younger child, answer the following questions. 

Q. 23 What is the number of children of age 9 years of less whose height does not exceed 135 cm? 

A 48 

B 45 

C 3 

D Cannot be determined 

Answer: B 

Explanation: 

Number of children of age 9 years or less are 48 and those whose height does not exceed 135 cm are 45 . 45 is lesser, hence the answer is 45. 

Q. 24 How many children of age more than 10 years are taller than 150 cm and do not weigh more than 48 kg? 

A 16 

B 40 

C 9 

D Cannot be determined 

Answer: A 

Explanation: 

There are 40 children of age more than 10 years and 25 children that are taller than 150 cm. Considering 25 which is less than 40 . Also there are 9 children whose weight is more than 48 kg. Hence there are 25-9 = 16 children whose weight will be less than 48 kg and all other requirements. 

Q. 25 Among the children older than 6 years but not exceeding 12 years, how many weigh more than 38 kg.? 

A 34 

B 52 

C 44 

D Cannot be determined 

Answer: C 

Explanation: 

Number of children older than 6 years but not exceeding 12 years are 55 and number older than 12 years are 23 . Also children weighing more than 38 kgs are 67. Out of these 67 , 23 will have age more than 12 . Hence we have 67 – 23 = 44 childrens will the given requirement. Hence option C. 

 

Instructions 

DIRECTIONS for the following two questions: Answer the questions on the basis of the information given below.

An industry comprises four firms (A, B, C, and D). Financial details of these firms and of the industry as a whole for a particular year are given below. Profitability of a firm is defined as profit as a percentage of sales. 

Q. 26 Which firm has the highest profitability? 

A A 

B B 

C C 

D D 

Answer: D 

Explanation: 

From the table we can see that , profitability in A , B ,C , D (4914)/(24568) , (4075)/(25468) , (4750)/(23752) , (3946)/(15782) . So clearly D has the highest profitability. 

 

Q. 27 If firm A acquires firm B, approximately what percentage of the total market (total sales) will they corner together? 

A 55% 

B 45% 

C 35% 

D 50% 

Answer: A 

Explanation: 

If A acquires firm B , the total sales of 2 firms is 50036 . now the percent market share can be given by 50036*100/89576 which is equal to around 55% . Hence option A. 

 

Instructions 

DIRECTIONS for the following three questions: Answer the questions on the basis of the information given below. 

A, B, C, D, E, and F are a group of friends. There are two housewives, one professor, one engineer, one accountant and one lawyer in the group. There are only two married couples in the group. The lawyer is married to D, who is a housewife. No woman in the group is either an engineer or an accountant. C, the accountant, is married to F, who is a professor. A is married to a housewife. E is not a housewife. 

Q. 28 Which of the following is one of the married couples? 

A A & B 

B B & E 

C D & E 

D A & D 

Answer: D 

Explanation: 

According to given conditions, we are able to infer following relations 

So A and D are married couples. 

Q. 29 What is E’s profession? 

A Engineer 

B Lawyer 

C Professor 

D Accountant 

Answer: A 

Explanation: 

According to given conditions we are able to infer, 

Hence E is an engineer. 

Q. 30 How many members of the group are males? 

A 2 

B 3 

C 4 

D Cannot be determined 

Answer: B 

Explanation: 

According to given conditions we are able to infer, 

Thus, there are 3 males. 

Instructions 

DIRECTIONS for the following two questions: Answer the questions on the basis of the information given below. 

The Head of a newly formed government desires to appoint five of the six elected members A, B, C, D, E and F to portfolios of Home, Power, Defence, Telecom and Finance. F does not want any portfolio if D gets one of the five. C wants either Home or Finance or no portfolio. B says that if D gets either Power or Telecom then she must get the other one. E insists on a portfolio if A gets one. 

Q. 31 Which is a valid assignment? 

A A-Home, B-Power, C-Defence, D-Telecom, E-Finance. 

B C-Home, D-Power, A-Defence, B-Telecom, E-Finance. 

C A-Home, B-Power, E-Defence, D-Telecom, F-Finance. 

D B-Home, F-Power, E-Defence, C-Telecom, A-Finance. 

Answer: B 

Explanation: 

Since C wants either home or finance or none so options A and D are eliminated. 

Since F does not want any portfolio if D gets one, Option C is eliminated. 

Q. 32 If A gets Home and C gets Finance, then which is NOT a valid assignment of Defense and Telecom? 

A D-Defence, B-Telecom. 

B F-Defence, B-Telecom. 

C B-Defence, E-Telecom. 

D B-Defence, D-Telecom. 

Answer: D 

Explanation: 

B says that if D gets power or telecom then he must get the other one.Option D clearly violates that. Instructions 

DIRECTIONS for the following three questions: Answer the questions on the basis of the information given below.

Rang Barsey Paint Company (RBPC) is in the business of manufacturing paints. RBPC buys RED, YELLOW, WHITE, ORANGE, and PINK paints. ORANGE paint can be also produced by mixing RED and YELLOW paints in equal proportions. Similarly, PINK paint can also be produced by mixing equal amounts of RED and WHITE paints. Among other paints, RBPC sells CREAM paint, (formed by mixing WHITE and YELLOW in the ratio 70:30) AVOCADO paint (formed by mixing equal amounts of ORANGE and PINK paint) and WASHEDORANGE paint (formed by mixing equal amounts of ORANGE and WHITE paint). The following table provides the price at which RBPC buys paints. 

Q. 33 The cheapest way to manufacture AVOCADO paint would cost 

A Rs. 19.50 per litre. 

B Rs. 19.75 per litre 

C Rs. 20.00 per litre. 

D Rs. 20.25 per litre. 

Answer: B 

Explanation: 

AVOCADO paint can be manufactured by adding orange and pink in equal quantity. 0.5 ltr of orange would cost 11 and the cheapest way to make pink would be by mixing white and red , so the cost for 0.5 ltr of pink comes out to be 8.75 . SO total cost becomes 11+8.75 = 19.75 Rs. which is the cheapest. 

Q. 34 WASHEDORANGE can be manufactured by mixing 

A CREAM and RED in the ratio 14:10. 

B CREAM and RED in the ratio 3:1. 

C YELLOW and PINK in the ratio 1:1. 

D RED, YELLOW, and WHITE in the ratio 1:1:2. 

Answer: D 

Explanation: 

WASHEDORANGE can be made by mixing orange and white , orange can be made by mixing equal quantities red and yellow which would be 1/2 of white quantity. Thus RED, YELLOW, and WHITE in the ratio 1:1:2 are needed. 

Q. 35 Assume that AVOCADO, CREAM and WASHEDORANGE each sells for the same price. Which of the three is the most profitable to manufacture? 

A AVOCADO 

B CREAM 

C WASHEDORANGE 

D Sufficient data is not available. 

Answer: B 

Explanation: 

Cream would undoubtedly be the most profitable as the maximum amount of white paint is used in it and white is the cheapest out of all other paints. Hence option B. 

 

Instructions 

DIRECTIONS for the following three questions: Answer the questions on the basis of the information given below. 

Seven varsity basketball players (A, B, C, D, E, F, and G) are to be honoured at a special luncheon. The players will be seated on the dais in a row. A and G have to leave the luncheon early and so must be seated at the extreme right. B will receive the most valuable player’s trophy and so must be in the centre to facilitate presentation. C and D are bitter rivals and therefore must be seated as far apart as possible. 

Q. 36 Which of the following cannot be seated at either end? 

A C 

B D 

C F 

D G 

Answer: C 

Explanation: 

According to given conditions we can conclude , 

Hence , F cannot be seated at either end. 

 

Q. 37 Which of the following pairs cannot be seated together? 

A B & D 

B C & F 

C D & G 

D E & A 

Answer: D 

Explanation: 

According to given conditions following arrangements are possible, 

Hence E and A cannot be seated together. 

 

Q. 38 Which of the following pairs cannot occupy the seats on either side of B? 

A F & D 

B D & E 

C E & G 

D C & F 

Answer: C 

Explanation: 

According to given conditions possible arrangement is as, 

Hence E and G cannot occupy seats on either sides of B . 

 

Instructions 

DIRECTIONS for the following two questions: Answer the questions on the basis of the information given below. 

Some children were taking free throws at the basketball court in school during lunch break. Below are some facts about how many baskets these children shot. 

i. Ganesh shot 8 baskets less than Ashish. 

ii. Dhanraj and Ramesh together shot 37 baskets. 

iii. Jugraj shot 8 baskets more than Dhanraj. 

iv. Ashish shot 5 baskets more than Dhanraj. 

v. Ashish and Ganesh together shot 40 baskets. 

 

Q. 39 Which of the following statements is true? 

A Dhanraj and Jugraj together shot 46 baskets. 

B Ganesh shot 18 baskets and Ramesh shot 21 baskets. 

C Dhanraj shot 3 more baskets than Ramesh. 

D Ramesh and Jugraj together shot 29 baskets. 

Answer: A 

Explanation: 

Let a,d,j,g be the shots put by ashish,dhanraj,ganesh and jugraj respectively. According to given conditions we have , g=a-8; 

d+r=37; 

j=d+8; 

a=5+d; 

a+g=40 . 

Solving we have a=24, d=19 and j=27.so d+j=45 

 

Instructions 

DIRECTIONS for the following three questions: Answer the questions on the basis of the information given below. 

Five women decided to go shopping in M.G. Road, Bangalore. They arrived at the designated meeting place in the following order: 

1. Archana, 

2. Chellamma, 

3. Dhenuka, 

4. Helen, and 

5. Shahnaz. 

Each woman spent at least Rs. 1000. Below are some additional facts about how much they spent during their shopping spree. 

i. The woman who spent Rs. 2234 arrived before the lady who spent Rs. 1193. 

ii. One woman spent Rs. 1340 and she was not Dhenuka. 

iii. One woman spent Rs. 1378 more than Chellamma. 

iv. One woman spent Rs. 2517 and she was not Archana. 

v. Helen spent more than Dhenuka. 

vi. Shahnaz spent the largest amount and Chellamma the smallest. 

 

Q. 40 What was the amount spent by Helen? 

A Rs. 1193 

B Rs. 1340 

C Rs. 2234 

D Rs. 2517 

Answer: B 

Explanation: 

From the given statements, we can infer that the amounts spent by the women are Rs.2234, Rs.1193, Rs.1340, and Rs.2517. Also, we know that one of the 5 women spent Rs.1378 more than Chellamma. Also, we know that each woman spent at least Rs.1000. 

The person who spent Rs.2234 cannot be the person who spent Rs.1378 more than Chellamma. Also, we know that Chellamma spent the least among the five women. 

Case 1: 

Chellamma spent Rs.1193. 

=> One of the 5 women spent 1193+1378 = Rs.2571. 

We know that Shahnaz spent the most. Therefore, Shahnaz should have spent Rs.2571. 

Archana didn’t spend Rs.2517. Also, Helen spent more than Dhenuka. 

Therefore, Helen should have spent Rs.2517. 

Dhenuka didn’t spend Rs.1340. 

Therefore, Dhenuka should have spent Rs. 2234 and Archana should have spent Rs.1340. 

However, it has been given that the woman who spent Rs. 2234 arrived before the lady who spent Rs. 1193. According to the given order, Archana arrived before Dhenuka. Therefore, we can eliminate this case. 

Case 2: 

Rs.2517 is the highest amount spent. Shahnaz spent Rs.2517. 

=> Amount spent by Chellamma = 2517 – 1378 = Rs.1139 

Dhenuka didn’t spend Rs.1340. Helen spent more than Dhenuka. Therefore, Dhenuka should have spent Rs.1193. 

Now, we know that the woman who spent Rs. 2234 arrived before the lady who spent Rs. 1193. Helen did not arrive before Dhenuka but Archana did. Therefore, Archana should have spent Rs.2234 and Helen should have spent Rs.1340. 

According to given conditions amount spent by everyone is, 

Hence, option B is the right answer. 

 

Q. 41 Which of the following amounts was spent by one of them? 

A Rs. 1139 

B Rs. 1378 

C Rs. 2571 

D Rs. 2718 

Answer: A 

Explanation: 

From the given statements, we can infer that the amounts spent by the women are Rs.2234, Rs.1193, Rs.1340, and Rs.2517. Also, we know that one of the 5 women spent Rs.1378 more than Chellamma. Also, we know that each woman spent at least Rs.1000. 

The person who spent Rs.2234 cannot be the person who spent Rs.1378 more than Chellamma. Also, we know that Chellamma spent the least among the five women. 

Case 1: 

Chellamma spent Rs.1193. 

=> One of the 5 women spent 1193+1378 = Rs.2571. 

We know that Shahnaz spent the most. Therefore, Shahnaz should have spent Rs.2571. 

Archana didn’t spend Rs.2517. Also, Helen spent more than Dhenuka. 

Therefore, Helen should have spent Rs.2517. 

Dhenuka didn’t spend Rs.1340. 

Therefore, Dhenuka should have spent Rs. 2234 and Archana should have spent Rs.1340. 

However, it has been given that the woman who spent Rs. 2234 arrived before the lady who spent Rs. 1193. According to the given order, Archana arrived before Dhenuka. Therefore, we can eliminate this case. 

Case 2: 

Rs.2517 is the highest amount spent. Shahnaz spent Rs.2517. 

=> Amount spent by Chellamma = 2517 – 1378 = Rs.1139 

Dhenuka didn’t spend Rs.1340. Helen spent more than Dhenuka. Therefore, Dhenuka should have spent Rs.1193. 

Now, we know that the woman who spent Rs. 2234 arrived before the lady who spent Rs. 1193. Helen did not arrive before Dhenuka but Archana did. Therefore, Archana should have spent Rs.2234 and Helen should have spent Rs.1340. 

According to given conditions amount spent by everyone is, 

Hence, option A is the right answer. 

 

Q. 42 The woman who spent Rs. 1193 is 

A Archana 

B Chellamma 

C Dhenuka 

D Helen 

Answer: C 

Explanation: 

From the given statements, we can infer that the amounts spent by the women are Rs.2234, Rs.1193, Rs.1340, and Rs.2517. Also, we know that one of the 5 women spent Rs.1378 more than Chellamma. Also, we know that each woman spent at least Rs.1000. 

The person who spent Rs.2234 cannot be the person who spent Rs.1378 more than Chellamma. Also, we know that Chellamma spent the least among the five women. 

Case 1: 

Chellamma spent Rs.1193. 

=> One of the 5 women spent 1193+1378 = Rs.2571. 

We know that Shahnaz spent the most. Therefore, Shahnaz should have spent Rs.2571. 

Archana didn’t spend Rs.2517. Also, Helen spent more than Dhenuka. 

Therefore, Helen should have spent Rs.2517. 

Dhenuka didn’t spend Rs.1340. 

Therefore, Dhenuka should have spent Rs. 2234 and Archana should have spent Rs.1340. 

However, it has been given that the woman who spent Rs. 2234 arrived before the lady who spent Rs. 1193. According to the given order, Archana arrived before Dhenuka. Therefore, we can eliminate this case. 

Case 2: 

Rs.2517 is the highest amount spent. Shahnaz spent Rs.2517. 

=> Amount spent by Chellamma = 2517 – 1378 = Rs.1139 

Dhenuka didn’t spend Rs.1340. Helen spent more than Dhenuka. Therefore, Dhenuka should have spent Rs.1193. 

Now, we know that the woman who spent Rs. 2234 arrived before the lady who spent Rs. 1193. Helen did not arrive before Dhenuka but Archana did. Therefore, Archana should have spent Rs.2234 and Helen should have spent Rs.1340. 

According to given conditions amount spent by everyone is, 

Hence, option C is the right answer. 

Instructions 

DIRECTIONS for the following three questions: Answer the questions on the basis of the information given below. 

Five friends meet every morning at Sree Sagar restaurant for an idli-vada breakfast. Each consumes a different number of idlis and vadas. The number of idlis consumed are 1, 4, 5, 6, and 8, while the number of vadas consumed are 0, 1, 2, 4, and 6. Below are some more facts about who eats what and how much. 

i. The number of vadas eaten by Ignesh is three times the number of vadas consumed by the person who eats four idlis. ii. Three persons, including the one who eats four vadas eat without chutney. 

iii. Sandeep does not take any chutney. 

iv. The one who eats one idli a day does not eat any vadas or chutney. Further, he is not Mukesh. v. Daljit eats idli with chutney and also eats vada. 

vi. Mukesh, who does not take chutney, eats half as many vadas as the person who eats twice as many idlis as he does. vii. Bimal eats two more idlis than Ignesh, but Ignesh eats two more vadas than Bimal. 

 

Q. 43 Which one of the following statements is true? 

A Daljit eats 5 idlis 

B Ignesh eats 8 idlis 

C Bimal eats 1 idli. 

D Bimal eats 6 idlis. 

Answer: A 

Explanation: 

According to given conditions , we get 

Hence , diljit eats 5 idlis. 

Q. 44 Which of the following statements is true? 

A Sandeep eats 2 vadas. 

B Mukesh eats 4 vadas. 

C Ignesh eats 6 vadas. 

D Bimal eats 2 vadas. 

Answer: C 

Explanation: 

According to given conditions , we get 

Hence, ignesh eats 6 wadas. 

 

Q. 45 Which of the following statements is true? 

A Mukesh eats 8 idlis and 4 vadas but no chutney. 

B The person who eats 5 idlis and 1 vada does not take chutney. 

C The person who eats equal number of vadas and idlis also takes chutney. 

D The person who eats 4 idlis and 2 vadas also takes chutney. 

Answer: C 

Explanation: 

According to given conditions , we get 

Hence ignesh is the person who eats equal number of vadas and idlis also takes chutney. 

Instructions 

DIRECTIONS for the following two questions: Answer the questions on the basis of the information given below. 

A certain perfume is available at a duty-free shop at the Bangkok international airport. It is priced in the Thai currency Baht but other currencies are also acceptable. In particular, the shop accepts Euro and US Dollar at the following rates of exchange: 

US Dollar 1 = 41 Bahts 

Euro 1= 46 Bahts 

The perfume is priced at 520 Bahts per bottle. After one bottle is purchased, subsequent bottles are available at a discount of 30%. Three friends S, R and M together purchase three bottles of the perfume, agreeing to share the cost equally. R pays 2 Euros. M pays 4 Euros and 27 Thai Bahts and S pays the remaining amount in US Dollars. 

Q. 46 How much does R owe to S in Thai Baht? 

A 428 

B 416 

C 334 

D 324 

Answer: D 

Explanation: 

Total to be paid = 1248 Baht 

Each has to pay 1248/3 = 416 Baht 

R paid 92 Baht 

M paid 184+27 = 211 Baht 

So, R owes S 416 – 92 = 324 Baht 

 

Q. 47 How much does M owe to S in US Dollars? 

A 3 

B 4 

C 5 

D 6 

Answer: C 

Explanation: 

Total to be paid = 1248 Baht 

Each has to pay 1248/3 = 416 Baht 

R paid 92 Baht 

M paid 184+27 = 211 Baht 

So, R owes S 416 – 92 = 324 Baht 

M owes S 416-211 Baht = 205 Baht = 5 US Dollars 

 

Instructions 

DIRECTIONS for the following two questions: Answer the questions on the basis of the information given below. 

New Age Consultants have three consultants Gyani, Medha and Buddhi. The sum of the number of projects handled by Gyani and Buddhi individually is equal to the number of projects in which Medha is involved. All three consultants are involved together in 6 projects. Gyani works with Medha in 14 projects. Buddhi has 2 projects with Medha but without Gyani, and 3 projects with Gyani but without Medha. The total number of projects for New Age Consultants is one less than twice the number of projects in which more than one consultant is involved. 

Q. 48 What is the number of projects in which Gyani alone is involved? 

A Uniquely equal to zero. 

B Uniquely equal to 1. 

C Uniquely equal to 4. 

D Cannot be determined uniquely. 

Answer: D 

Explanation: 

The total number of projects = 2(3+6+8+2) – 1 = 38 – 1 = 37 

So, 19 + 2(x+y) – 16 = 37 

=> x+y = 17 

The number of projects in which Medha alone is involved is 17-16 = 1 

But the number of projects in which Gyani alone is involved cannot be uniquely determined 

Q. 49 What is the number of projects in which Medha alone is involved? 

A Uniquely equal to zero. 

B Uniquely equal to 1. 

C Uniquely equal to 4. 

D Cannot be determined uniquely. 

Answer: B 

Explanation: 

The total number of projects = 2(3+6+8+2) – 1 = 38 – 1 = 37 

So, 19 + 2(x+y) – 16 = 37 

=> x+y = 17 

The number of projects in which Medha alone is involved is 17-16 = 1 

But the number of projects in which Gyani alone is involved cannot be uniquely determined 

Instructions 

DIRECTIONS for the following three questions: Answer the questions on the basis of the information given below. A city has two perfectly circular and concentric ring roads, the outer ring road (OR) being twice as long as the inner ring road (IR). There are also four (straight line) chord roads from E1, the east end point of OR to N2, the north end point of IR; from N1, the north end point of OR to W2, the west end point of IR; from W1, the west end point of OR, to S2, the south end point of IR; and from S1 the south end point of OR to E2, the east end point of IR. Traffic moves at a constant speed of 30π km/hr on the OR road, 20π km/hr on the IR road, and 15√5 km/hr on all the chord roads. 

 

Q. 50 The ratio of the sum of the lengths of all chord roads to the length of the outer ring road is

A √5 : 2

B √5 : 2π

C √5 : π

D None of the above. 

Answer: C 

Explanation: 

Let the radius of outer circle be 2R and the centre of both the circles be O. 

Triangle ON2E1 and all the other 3 similar triangles form a right angle at the centre . SO using pythagoras theorem the value of chords come out to be √5* R so total length of the chords 4 *√5 * R And circumference is equal to 2 *Pi*2R . Ratio gives option C. 

Quant 

Instructions 

For the following questions answer them individually 

Q. 51 F and M are father and mother of S, respectively. S has four uncles and three aunts. F has two siblings. The siblings of F and M are unmarried. How many brothers does M have? 

A. F has two brothers. 

B. M has five siblings. 

A The questions can be answered by one of the statements, but not by the other 

B The Q. can be answered using either of the two statements alone. 

C The questions can be answered using both the statements together, but cannot be answered using either statement alone. 

D The Q. cannot be answered even by using both the statements together. 

Answer: A 

Explanation: 

Statement B is redundant. We already know that M has 5 siblings from the question. From statement A alone, we know that F has 2 brothers and M has 2 brothers and 3 sisters. 

Q. 52 A game consists of tossing a coin successively. There is an entry fee of Rs. 10 and an additional fee of Re. 1 for each toss of coin. The game is considered to have ended normally when the coin turns heads on two consecutive throws. In this case the player is paid Rs. 100. Alternatively, the player can choose to terminate the game prematurely after any of the tosses. Ram has incurred a loss of Rs. 50 by playing this game. How many times did he toss the coin? 

A. The game ended normally. 

B. The total number of tails obtained in the game was 138. 

A The questions can be answered by one of the statements, but not by the other 

B The Q. can be answered using either of the two statements alone. 

C The questions can be answered using both the statements together, but cannot be answered using either statement alone. 

D The Q. cannot be answered even by using both the statements together. 

Answer: B 

Explanation: 

Considering statement A. The game ended normally. Here we know that last 2 results were heads and the person receives 100 Rs. and since there was overall loss of 50 Rs we can calculate no. of matches which ended in tails or non consecutive heads . hence , statement a is sufficient to answer. 

Considering statement B. The total number of tails obtained in the game was 138. So we know person lost here rs.138 and we can add any no. of single heads in between those tails such that the overall loss will be 50Rs. Hence this statement is also sufficient to answer. 

Q. 53 Each packet of SOAP costs Rs. 10. Inside each packet is a gift coupon labelled with one of the letters S, O, A and P. If a customer submits four such coupons that make up the word SOAP, the customer gets a free SOAP packets. Ms. X kept buying packet after packet of SOAP till she could get one set of coupons that formed the word SOAP. How many coupons with label P did she get in the above process? 

A. The last label obtained by her was S and the total amount spent was Rs. 210. 

B. The total number of vowels obtained was 18. 

A The questions can be answered by one of the statements, but not by the other 

B The Q. can be answered using either of the two statements alone. 

C The questions can be answered using both the statements together, but cannot be answered using either statement alone. 

D The Q. cannot be answered even by using both the statements together. 

Answer: C 

Explanation: 

Considering statements we can deduce that she got letter S in her 21st purchase. In the previous 20 purchases she had either A, O or P . Hence statement a is not enough to answer. If we consider statement b we will know that out of 20 purchases 18 are vowel so 2 purchases contained letter P . So both statements are necessary to answer the question. 

Q. 54 If A and B run a race, then A wins by 60 seconds. If B and C run the same race, then B wins by 30 seconds. Assuming that C maintains a uniform speed what is the time taken by C to finish the race? 

A. A and C run the same race and A wins by 375 metres. 

B. The length of the race is 1 km. 

A The questions can be answered by one of the statements, but not by the other 

B The Q. can be answered using either of the two statements alone. 

C The questions can be answered using both the statements together, but cannot be answered using either statement alone. 

D The Q. cannot be answered even by using both the statements together. 

Answer: C 

Explanation: 

Let the distance be x. Time taken by A is T, so time taken by B is T+60 and time taken by C is T+90. From statement A, we know that the ratio of speeds of A and C is x:(x-375) = x/T : x/(T+90) 

We have only 1 equation and 2 unknows. We need the second statement to find the values. So, the Q. can be answered by using both the statements together. 

Q. 55 The number of non-negative real roots of  2x x − 1 = 0 equals 

A 0 

B 1 

C 2 

D 3 

Answer: C 

Explanation: 

2x x − 1 = 0 for this equation only 0 and 1 i.e 2 non-negative solutions are possible. Or we can plot the graph of 2x and x+1 and determine the number of points of intersection and hence the solution. 

Q. 56 When the curves y = log10x and y = x−1  are drawn in the x-y plane, how many times do they intersect for x ≥ 1 values ? 

A Never 

B Once 

C Twice 

D More than twice 

Answer: B 

Explanation: 

Graph of log x goes on increasing in 1st quadrant and graph of 1/x goes no decreasing with both intersecting only once

Q. 57 Let A and B be two solid spheres such that the surface area of B is 300% higher than the surface area of A. The volume of A is found to be k% lower than the volume of B. The value of k must be

A 85.5

B 92.5 

C 90.5 

D 87.5 

Answer: D 

Explanation: 

Surface area of sphere A (of radius a) is 4π a2

Surface area of sphere B (of radius b) is 4π b2

=> 4π a2/4π b2 = 1/4 => a:b = 1:2 

Their volumes would be in the ratio 1:8 

Therefore, k = 7/8 * 100% = 87.5% 

Q. 58 Which one of the following conditions must p, q and r satisfy so that the following system of linear simultaneous equations has at least one solution, such that p + q + r ≠ 0? 

x+ 2y – 3z = p 

2x + 6y – 11z = q 

x – 2y + 7z = r 

A 5p -2q – r = 0 

B 5p + 2q + r = 0 

C 5p + 2q – r = 0 

D 5p – 2q + r = 0 

Answer: A 

Explanation: 

Substitute value of p,q,r in the options only option A satisfies . 

5(x+2y-3z)-2(2x+6y-11z)-(x-2y+7z) = 5x+10y-15z-4x-12y+22z-x+2y-7z = 0 

Q. 59 A leather factory produces two kinds of bags, standard and deluxe. The profit margin is Rs. 20 on a standard bag and Rs. 30 on a deluxe bag. Every bag must be processed on machine A and on Machine B. The processing times per bag on the two machines are as follows: 

The total time available on machine A is 700 hours and on machine B is 1250 hours. Among the following production plans, which one meets the machine availability constraints and maximizes the profit? 

A Standard 75 bags, Deluxe 80 bags 

B Standard 100 bags, Deluxe 60 bags 

C Standard 50 bags, Deluxe 100 bags 

D Standard 60 bags, Deluxe 90 bags 

Answer: A 

Explanation: 

Let x be no. of standard bags and y be no. of deluxe bags. According to given conditions we have 2 equations 4x+5y<=700 and 6x+10y<=1250. Here option A satisfies both the equations. 

Q. 60 The sum of 3rd and 15th elements of an arithmetic progression is equal to the sum of 6th, 11th and 13th elements of the same progression. Then which element of the series should necessarily be equal to zero? 

A 1st 

B 9th 

C 12th 

D None of the above 

Answer: C 

Explanation: 

The sum of the 3rd and 15th terms is a+2d+a+14d = 2a+16d 

The sum of the 6th, 11th and 13th terms is a+5d+a+10d+a+12d = 3a+27d 

Since the two are equal, 2a+16d = 3a+27d => a+11d = 0 

So, the 12th term is 0 

Q. 61 A test has 50 questions. A student scores 1 mark for a correct answer, -1/3 for a wrong answer, and -1/6 for not attempting a question. If the net score of a student is 32, the number of questions answered wrongly by that student cannot be less than 

A 6 

B 12 

C 3 

D 9 

Answer: C 

Explanation: 

Let the number of questions answered correctly be x and the number of questions answered wrongly be y. So, number of questions left unattempted = (50-x-y) 

So, x – y/3 – (50-x-y)/6 = 32 

=> 6x – 2y – 50 + x + y = 192 => 7x – y = 242 => y = 7x – 242 

If x = 35, y = 3 

If x = 36, y = 10 

So, min. the value of y is 3. 

The number of wrongly answered questions cannot be less than 3. 

Q. 62 Twenty-seven persons attend a party. Which one of the following statements can never be true? 

A There is a person in the party who is acquainted with all the twenty-six others. 

B Each person in the party has a different number of acquaintances. 

C There is a person in the party who has an odd number of acquaintances. 

D In the party, there is no set of three mutual acquaintances. 

Answer: B 

Explanation: 

From the options a, c and d all can possibly occur. Hence option b. Besides, if all people have different number of acquaintances, then first person will have 26 acquaintance, second person will have 25 acquaintance, third person will have 24 and so on till 27 th person will have 0 acquaintance. 0 acquaintance is practically not possible. 

Q. 63 Let g(x) = max(5 – x, x + 2). The smallest possible value of g(x) is 

A 4.0 

B 4.5 

C 1.5 

D None of the above 

Answer: D 

Explanation: 

Smallest possible value would be at 5-x = x+2 i.e. x= 1.5 as shown 

Substituting we get the smallest value as 3.5. 

Q. 64 The function f(x) = |x – 2| + |2.5 – x| + |3.6 – x|, where x is a real number, attains a minimum at 

A x = 2.3 

B x = 2.5 

C x = 2.7 

D None of the above 

Answer: B 

Explanation: 

f(x) = |x – 2| + |2.5 – x| + |3.6 – x| 

For x belonging to (-infinity to 2), f(x) = 2-x + 2.5-x + 3.6-x = 8.1-3x 

This attains the minimum value at x=2. Value = 2.1 

For x belonging to (2 to 2.5), f(x) = x-2 + 2.5-x + 3.6-x = 4.1-x 

Attains the minimum value at x = 2.5. Value = 1.6 

For x belonging to (2.5 to 3.6), f(x) = x-2 + x-2.5 + 3.6-x = x-0.9 

Attains the minimum at x=2.5, value = 1.6 

For x > 3.6, f(x) = x-2+x-2.5+x-3.6 = 3x – 8.1 

Attains the minimum at x= 3.6, value = 2.7 

So, min value of the function is 1.6 at x=2.5 

Q. 65 How many even integers n, where 100 ≤ n ≤ 200, are divisible neither by seven nor by nine? 

A 40 

B 37 

C 39 

D 38 

Answer: C 

Explanation: 

Between 100 and 200 both included there are 51 even nos. There are 7 even nos which are divisible by 7 and 6 nos which are divisible by 9 and 1 no divisible by both. hence in total 51 – (7+6-1) = 39 

There is one more method through which we can find the answer. Since we have to find even numbers, consider the numbers which are divisible by 14, 18 and 126 between 100 and 200. These are 7, 6 and 1 respectively. 

Q. 66 A positive whole number M less than 100 is represented in base 2 notation, base 3 notation, and base 5 notation. It is found that in all three cases the last digit is 1, while in exactly two out of the three cases the leading digit is 1. Then M equals 

A 31 

B 63 

C 75 

D 91 

Answer: D 

Explanation: 

Since in all three cases the last digit is 1, the number should give remainder 1 when divided individually by 2,3,5 . So the no. may be 31 or 91 . Now 31 in base 2,3 and 5 give the first digit as 1 in all the 3 cases while 91 gives exactly two out of the three cases the leading digit as 1. Hence option D. 

Q. 67 In a 4000 meter race around a circular stadium having a circumference of 1000 meters, the fastest runner and the slowest runner reach the same point at the end of the 5th minute, for the first time after the start of the race. All the runners have the same starting point and each runner maintains a uniform speed throughout the race. If the fastest runner runs at twice the speed of the slowest runner, what is the time taken by the fastest runner to finish the race? 

A 20 min 

B 15 min 

C 10 min 

D 5 min 

Answer: C 

Explanation: 

Let A , B and f,s be the distance traveled and speed of the fastest and the slowest person respectively. Also f=2s so in the given time A=2B. Since the ration of the speeds is 2:1, they will meet at 2-1 points = 1 point. 

Both meet each other for the first time at the starting point . let b travel distance equal to 1 circumference i.e. 1000m so A=2000m . Both meet after 5 min so the slowest is 1000/5=200m/min . So the speed of the fastest is 400m/min. So time taken by A to complete race 4000/400 = 10 min 

Q. 68 Is a44 < b11, given that a = 2 and b is an integer? 

A. b is even 

B. b is greater than 16 

A The questions can be answered by one of the statements, but not by the other 

B The Q. can be answered using either of the two statements alone. 

C The questions can be answered using both the statements together, but cannot be answered using either statement alone. 

D The Q. cannot be answered even by using both the statements together. 

Answer: A 

Explanation: 

If b = 4, then the answer to the Q. is no. If b = 32, then the answer is yes. So, using statement A alone, we cannot answer the question. 

Using statement B alone, we can conclude that a44 < b11 

Q. 69 What are the unique values of b and c in the equation 4x2 + bx + c = 0 if one of the roots of the equation is (- 1/2)? 

A. The second root is 1/2. 

B. The ratio of c and b is 1. 

A The questions can be answered by one of the statements, but not by the other 

B The Q. can be answered using either of the two statements alone. 

C The questions can be answered using both the statements together, but cannot be answered using either statement alone. 

D The Q. cannot be answered even by using both the statements together. 

Answer: B 

Explanation: 

Using statement A, sum of roots = 0 and product of roots = 0, so b = c = 0. 

Using statement B, sum of roots = x – 1/2 = -b/4 and product of roots = c/4 = b/4 = -x/2 

So, we can calculate the values of b and c using either statement alone 

Q. 70 AB is a chord of a circle. AB = 5 cm. A tangent parallel to AB touches the minor arc AB at E. What is the radius of the circle? 

A. AB is not a diameter of the circle. 

B. The distance between AB and the tangent at E is 5 cm. 

A The questions can be answered by one of the statements, but not by the other 

B The Q. can be answered using either of the two statements alone. 

C The questions can be answered using both the statements together, but cannot be answered using either statement alone. 

D The Q. cannot be answered even by using both the statements together. 

Answer: A 

Explanation: 

Let the radius be r. Using statement B alone, r, r-5 and 2.5 form a right-angled triangle. So, we can answer the Q. using statement B alone. 

Q. 71 (1/a2 + 1/a4 + 1/a6 + …) > (1/a + 1/a3 + 1/a5 + …) Is ? 

 

A. 0 < a ≤ 1 

B. One of the roots of the equation 4x2 − 4x + 1 = 0 is a 

A The questions can be answered by one of the statements, but not by the other 

B The Q. can be answered using either of the two statements alone. 

C The questions can be answered using both the statements together, but cannot be answered using either statement alone. 

D The Q. cannot be answered even by using both the statements together. 

Answer: A 

Explanation: 

Consider the first statement: 

When the common ratio is less than 1 we can apply the formula of sum of infinite terms. 

So, LHS = 1/(a2 − 1) 

RHS = a/(a2 − 1) 

If a<1 then LHS<RHS 

If a = 1,then LHS = RHS 

So, we cannot answer the Q. using statement 1 alone 

Using statement 2 alone, we know that a = 1/2. So, RHS > LHS 

Hence, option a) 

Q. 72 D, E, F are the mid points of the sides AB, BC and CA of triangle ABC respectively. What is the area of DEF in square centimeters? 

A. AD = 1 cm, DF = 1 cm and perimeter of DEF = 3 cm 

B. Perimeter of ABC = 6 cm, AB = 2 cm, and AC = 2 cm. 

A The questions can be answered by one of the statements, but not by the other 

B The Q. can be answered using either of the two statements alone. 

C The questions can be answered using both the statements together, but cannot be answered using either statement alone. 

D The Q. cannot be answered even by using both the statements together. 

Answer: B 

Explanation: 

From statement 1 alone, we can infer that the triangle ABC is an equilateral triangle with side = 2 cm Similarly, from statement 2 alone, we can infer that the triangle ABC is an equilateral of side 2 cm So, the Q. can be answered using either statement alone 

Q. 73 At the end of year 1998, Shepard bought nine dozen goats. Henceforth, every year he added p% of the goats at the beginning of the year and sold q% of the goats at the end of the year where p > 0 and q > 0. If Shepard had nine dozen goats at the end of year 2002, after making the sales for that year, which of the following is true? 

A p = q 

B p < q 

C p > q 

D p = q/2 

Answer: C 

Explanation: 

By the end of the year 2002, Shepard bought 4 times and sold 4 times. He is left with the initial number of goats that he had in 1998. If the percentage of goats bought is equal to or lesser than the percentage of goats sold, then there would be a net decrease in the total number of goats. For the number of goats to remain the same, p has to be greater than q, because p% is being calculated in a lesser number and q% is being calculated on a greater number. Hence, p > q. 

Q. 74 Each side of a given polygon is parallel to either the X or the Y axis. A corner of such a polygon is said to be convex if the internal angle is 90° or concave if the internal angle is 270°. If the number of convex corners in such a polygon is 25, the number of concave corners must be 

A 20 

B 0 

C 21 

D 22 

Answer: C 

Explanation: 

Let the total number of sides be x. 

Sum of internal angles in a polygon = (x-2)*180 where x is the number of sides. 

It is given that the polygon has 25 convex sides, then the number of concave sides = x-25 (25*90)+(x-25)*270 = (x-2)180 

x = 46 

Number concave corners = x-25 = 46-25 = 21 

Q. 75 The 288th term of the series a,b,b,c,c,c,d,d,d,d,e,e,e,e,e,f,f,f,f,f,f… is 

A u 

B v 

C w 

D x 

Answer: D 

Explanation: 

1, 2, 3, 4,….n such that the sum is greater than 288 

If n = 24, n(n+1)/2 = 12*25 = 300 

So, n = 24, i.e. the 24th letter in the alphabet is the letter at position 288 in the series 

So, answer = x 

Q. 76 Let p and q be the roots of the quadratic equation x2 − (α − 2)x α − 1 = 0 . What is the minimum possible p2 + q2 

value of ? 

A 0 

B 3 

C 4 

D 5 

Answer: D 

Explanation: 

Let α be equal to k. 

=> f(x) = x2 − (k − 2)x − (k + 1) = 0

p and q are the roots 

=> p+q = k-2 and pq = -1-k 

We know that (p + q)2 = p2 + q2 + 2pq 

=>(k − 2)2 = p2 + q2 + 2(−1 − k

 =>p2 + q2 = k2 + 4 − 4k + 2 + 2

 =>p2 + q2 = k2 − 2k + 6 

This is in the form of a quadratic equation. 

The coefficient of k2 is positive. Therefore this equation has a minimum value. 

We know that the minimum value occurs at x = −b/2

Here a = 1, b = -2 and c = 6 

=> Minimum value occurs at k =2/2  = 1 

If we substitute k = 1 in k2 − 2k + 6 , we get 1 -2 + 6 = 5. 

Hence 5 is the minimum value that p2 + q2  can attain. 

Q. 77 There are two concentric circles such that the area of the outer circle is four times the area of the inner circle. Let A, B and C be three distinct points on the perimeter of the outer circle such that AB and AC are tangents to the inner circle. If the area of the outer circle is 12 square centimeters then the area (in square centimeters) of the triangle ABC would be 

A π√12

B 9/π

C 93/π

D 63/π

Answer: C 

Explanation: 

Let R ,r be radius of big and small circles respectively. We know that R=2r. And since area = 12 ; 

R2 = 12/π

By pythagoras theorem in the small triangle with side ‘x’ we have x = 3 /√π. This triangle is a 30-60-90 right triangle. => Triangle ABC is an equilateral triangle. 

So side of triangle ABC = 2x = 6 /√π

Also Angle OAB = 30 (as the side opposite to the angle is half of the hypotenuse ). 

Hence Angle CAB = 60. 

Hence the required triangle is an equilateral triangle. So area =√3 * (2x)2 / 4 . Substituting we get . 

Area = 9√3/π

Q. 78 Let a, b, c, d be four integers such that a+b+c+d = 4m+1 where m is a positive integer. Given m, which one of the following is necessarily true? 

A The minimum possible value of is a2 + b2 + c2 + d2 4m2 − 2m + 1

B The minimum possible value of is a2 + b2 + c2 + d2 4m2 + 2m + 1

C The maximum possible value of is a2 + b2 + c2 + d2 4m2 − 2m + 1

D The maximum possible value of is a2 + b2 + c2 + d2 4m2 + 2m + 1

Answer: B 

Explanation: 

Taking lowest possible positive value of m i.e. 1 . Such that a+b+c+d=5 , so at least one of them must be greater than 1 , take a=b=c=1 and d=2 

we get a2 + b2 + c2 + d2 = 7 which is equal to 4m2 + 2m + 1 for other values it is greater than 4m2 + 2m + 1 . so option B 

Q. 79 Three horses are grazing within a semi-circular field. In the diagram given below, AB is the diameter of the semi-circular field with center at O. Horses are tied up at P, R and S such that PO and RO are the radii of semi-circles with centers at P and R respectively, and S is the center of the circle touching the two semi circles with diameters AO and OB. The horses tied at P and R can graze within the respective semi-circles and the horse tied at S can graze within the circle centred at S. The percentage of the area of the semicircle with diameter AB that cannot be grazed by the horses is nearest to 

A 20 

B 28 

C 36 

D 40 

Answer: B 

Explanation: 

Let R be radius of big circle and r be radius of circle with centre S. Radius of 2 semicircles is R/2. 

From Right angled triangle OPS, using pythagoras theorem we get 

(r + 0.5R)2 = (0.5R)2 + (R r)2 . We get R=3r . 

Now the area of big semicircle that cannot be grazed is Area of big S.C – area of 2 semicircle – area of small circle = π R2 /2 – 2*π* (0.5R)2/2- πr

= π R2 /2 – 2* π ∗(0.5R)2 π /2 – π ∗(R/3)2

= π R2 /2 – π R2/4 – π ∗(R)2/9

= 5*π * R2/36. this is about 28 % of the area π R2/2 . Hence option B. 

Q. 80 In the figure below, ABCDEF is a regular hexagon and ∠AOF= 90° . FO is parallel to ED. What is the ratio of the area of the triangle AOF to that of the hexagon ABCDEF? 

A 1/12 

B 1/6 

C 1/24 

D 1/18 

Answer: A 

Explanation: 

When the hexagon is divided into a number of similar triangles AOF we get 12 such triangles . Hence the required ratio of area is 1/12

Q. 81 How many three digit positive integers, with digits x, y and z in the hundred’s, ten’s and unit’s place 

respectively, exist such that x < y, z < y and x ≠ 0 ? 

A 245 

B 285 

C 240 

D 320 

Answer: C 

Explanation: 

x, y and z in the hundred’s, ten’s and unit’s place. So y should start from 2 

If y=2 , possible values of x=1 and z = 0,1 .So 2 cases 120,121. 

Also if y=3 , possible values of x=1,2 and z=0,1,2. 

Here 6 three digit nos. possible . 

Similarly for next cases would be 3*4=12,4*5=20,5*6=30,…..,8*9=72 . Adding all we get 240 cases.

Q. 82 A vertical tower OP stands at the center O of a square ABCD. Let h and b denote the length OP and AB respectively. Suppose ∠APB = 60° then the relationship between h and b can be expressed as 

A 2b2 = h2

B 2h2 = b2

C 3b2 = 2h2

D 3h2 = 2b2

Answer: B 

Explanation: 

Consider the triangle APB. ∠P = 600 and AP = BP => APB is an equilateral triangle.

Hence AP = b…(1) 

AC2  = AB2 + BC2 

AC2  = b2 + b2 => AC = √2 × b => AO = AC/2 = b/√2

AP2 = AO2  +OP2 

b2 =b2 /2 + h2  …From (1)

2h2 = b2 

Hence, option B is the correct answer. 

Q. 83 In the triangle ABC, AB = 6, BC = 8 and AC = 10. A perpendicular dropped from B, meets the side AC at D. A circle of radius BD (with center B) is drawn. If the circle cuts AB and BC at P and Q respectively, the AP:QC is equal to 

A 1:1 

B 3:2 

C 4:1 

D 3:8 

Answer: D 

Explanation: 

Let BD = x .Semi-perimeter of triangle ABC = 12. Now by herons formula area of ABC is 24. Also Area = 0.5*x*10 . We get x = 24/5 . AP = 6/5 and CQ = 16/5 . Hence the required ratio is 3:8. 

Q. 84 In the diagram given below, ∠ABD = ∠CDB = ∠PQD = 90° . If AB:CD = 3:1, the ratio of CD: PQ is

A 1 : 0.69 

B 1 : 0.75 

C 1 : 0.72 

D None of the above. 

Answer: B 

Explanation: 

Let BQ = z , QD = y , PQ = x. 

From similar triangles PQD and ABD we have 

(y/x) = (z+y)/3 . 

Also from similar triangles PQB and CBD we have 

(z/x) = z+y . 

Solving we get z = 3*y. 

Now the required ratio is (z+y)/z. 

We get equal to 4/3 which is equal to 1:0.75. 

Q. 85 There are 8436 steel balls, each with a radius of 1 centimeter, stacked in a pile, with 1 ball on top, 3 balls in the second layer, 6 in the third layer, 10 in the fourth, and so on. The number of horizontal layers in the pile is 

A 34 

B 38 

C 36 

D 32 

Answer: C 

Explanation: 

For the given problem , 

n(n + 1)/2 = 8436  which is 

 

n2/2 + ∑n/2 = 8436 which is equal to 

 

n*(n+1)(2n+1)/12 + n*(n+1)/4 = 8436 , solving we get n=36. 

Solving the equation might be lengthy. you can substitute the values in the options to arrive at the answer. 

Q. 86 If the product of n positive real numbers is unity, then their sum is necessarily 

A a multiple of n 

B equal to n + 1/n 

C never less than n 

D a positive integer 

Answer: C 

Explanation: 

 

Let a1, a2….an.the numbers be 

Since the numbers are positive, 

AM GM 

(a1+a2+a3….+an )/ n ≥(a1 ∗ a2…. ∗ an)1/n 

a1 + a2 + a3…. + an

Q. 87 If log32, log3(2x − 5), log3(2x − 7/2) are in arithmetic progression, then the value of x is equal to 

A 5 

B 4 

C 2 

D 3 

Answer: D 

Explanation: 

2log(2x − 5) = log2 + log(2x − 7/2) 

Let 2x = t

=> (t − 5)2 = 2(t − 7/2)

=> t2 + 25 − 10t = 2t − 7 

=> t2 − 12t + 32 = 0

=> t = 8, 4 

Therefore, x = 2 or 3, but 2x > 5, so x = 3  

Q. 88 In the figure below, AB is the chord of a circle with center O. AB is extended to C such that BC = OB. The straight line CO is produced to meet the circle at D. If ∠ACD= y degrees and ∠AOD= x degrees such that x = ky, then the value of k is 

A 3 

B 2 

C 1 

D None of the above. 

Answer: A 

Explanation: 

Since Angle BOC = Angle BCO = y. 

Angle OBC = 180-2y . 

Hence Angle ABO = z = 2y = Angle OAB. 

Now since x is the exterior angle of triangle AOC . 

We have x = z + y = 3y. 

Hence option A. 

Q. 89 In the figure below, the rectangle at the corner measures 10 cm × 20 cm. The corner A of the rectangle is also a point on the circumference of the circle. What is the radius of the circle in cm? 

A 10 cm 

B 40 cm 

C 50 cm 

D None of the above. 

Answer: C 

Explanation: 

As seen in the fig. we have a right angled triangle with sides r ,r-10 , r-20. 

r2 = (r − 10)2 + (r − 20)2 

Using pythagoras we have . 

Solving the equation, we get r = 10 or 50. 

But 10 is not possible , so r = 50. 

Hence the radius is 50. 

Q. 90 Given that −1 ≤ v ≤ 1, −2 ≤ u ≤ −0.5 and −2 ≤ z ≤ −0.5 and w = vz/u , then which of the following is necessarily true? 

A −0.5 ≤ w ≤ 2

B −4 ≤ w ≤ 4

C −4 ≤ w ≤ 2

D −2 ≤ w ≤ −0.5

Answer: B 

Explanation: 

We know w = vz/u so taking max value of u and min value of v and z to get min value of w which is -4. Similarly taking min value of u and max value of v and z to get max value of w which is 4 

Take v = 1, z = -2 and u = -0.5, we get w = 4 

Take v = -1, z = -2 and u = -0.5, we get w = -4 

Q. 91 There are 6 boxes numbered 1,2,… 6. Each box is to be filled up either with a red or a green ball in such a way that at least 1 box contains a green ball and the boxes containing green balls are consecutively numbered. The total number of ways in which this can be done is 

A 5 

B 21 

C 33 

D 60 

Answer: B 

Explanation: 

If there is only 1 green ball, it can be done in 6 ways 

If there are 2 green balls, it can be done in 5 ways. 

If there are 6 green balls, it can be done in 1 way. 

So, the total number of possibilities is 6*7/2 = 21 

Q. 92 Consider the following two curves in the x-y plane: 

y = x3 + x2 + 5 

y = x2 + x + 5 

Which of the following statements is true for −2 ≤ x ≤ 2 ? 

A The two curves intersect once. 

B The two curves intersect twice. 

C The two curves do not intersect 

D The two curves intersect thrice. 

Answer: D 

Explanation: 

Equate the 2 equations we get the value of x = 1 and -1 . Also we notice that there is an intersection at x=0 . hence D 

Q. 93 In a certain examination paper, there are no questions. For j = 1,2 …n, there are 2nj  students who answered j or more questions wrongly. If the total number of wrong answers is 4095, then the value of n is 

A 12 

B 11 

C 10 

D 9 

Answer: A 

Explanation: 

Let there only be 2 questions. 

Thus there are 22−1= 2 students who have done 1 or more questions wrongly, 22−2 = 1 students who have done all 2 questions wrongly . 

Thus total number of wrong answers = 2 + 1 = 3= 2n − 1. 

Now let there be 3 questions. Then j = 1,2,3 

Number of students answering 1 or more questions incorrectly = 4 

Number of students answering 2 or more questions incorrectly = 2 

Number of students answering 3 or more questions incorrectly = 1 

Total number of incorrect answers = 1(3)+(2-1)*2+(4-2)*1 = 7 = 23 − 1

According to the Q. , the total number of wrong answers = 4095 = 212 − 1 .

Hence Option A.  

Q. 94 If x, y, z are distinct positive real numbers the (x2(y + z) + y2(x + z) + z2(x + y))/xyz would be 

A greater than 4 

B greater than 5 

C greater than 6 

D None of the above 

Answer: C 

Explanation: 

For the given expression value of x,y,z are distinct positive integers . So the value of expression will always be greater than value when all the 3 variables are equal . substitute x=y=z we get minimum value of 6 . 

(x2(y + z) + y2(x + z) + z2(x + y))/xyz = x/z + x/y + y/z + y/x + z/y + z/x 

Applying AM greater than or equal to GM, we get minimum sum = 6 

Q. 95 A graph may be defined as a set of points connected by lines called edges. Every edge connects a pair of points. Thus, a triangle is a graph with 3 edges and 3 points. The degree of a point is the number of edges connected to it. For example, a triangle is a graph with three points of degree 2 each. Consider a graph with 12 points. It is possible to reach any point from any point through a sequence of edges. The number of edges, e, in the graph must satisfy the condition 

A 11 ≤ e ≤ 66

B 10 ≤ e ≤ 66

C 11 ≤ e ≤ 65

D 0 ≤ e ≤ 11

Answer: A 

Explanation: 

Take any 12 points. 

The maximum number of edges which can be drawn through these 12 points are 12C2 = 66 

The minimum number of edges which can be drawn through these 12 points are 12-1 = 11 as the resulting figure need not be closed. It might be open. 

Q. 96 The number of positive integers n in the range 12 ≤ n ≤ 40 such that the product (n -1)*(n – 2)*…*3*2*1 is not divisible by n is 

A 5 

B 7 

C 13 

D 14 

Answer: B 

Explanation: 

positive integers n in the range 12 ≤ n ≤ 40 such that the product (n -1)*(n – 2)*…*3*2*1 is not divisible by n, implies that n should be a prime no. So there are 7 prime nos. in the given range. Hence option B.  

Q. 97 Let T be the set of integers {3,11,19,27,…451,459,467} and S be a subset of T such that the sum of no two elements of S is 470. The maximum possible number of elements in S is 

A 32 

B 28 

C 29 

D 30 

Answer: D 

Explanation: 

No. of terms in series T , 3+(n-1)*8 = 467 i.e. n=59. 

Now S will have at least 59 terms i.e 29 . 

Also the sum of 29th term and 30th term is less than 470. 

Hence, the maximum possible elements in S is 30. 

 

Instructions 

DIRECTIONS for the following two questions: Answer the questions on the basis of the information given below. 

Some children were taking free throws at the basketball court in school during lunch break. Below are some facts about how many baskets these children shot. 

i. Ganesh shot 8 baskets less than Ashish. 

ii. Dhanraj and Ramesh together shot 37 baskets. 

iii. Jugraj shot 8 baskets more than Dhanraj. 

iv. Ashish shot 5 baskets more than Dhanraj. 

v. Ashish and Ganesh together shot 40 baskets. 

 

Q. 98 Which of the following statements is true? 

A Ramesh shot 18 baskets and Dhanraj shot 19 baskets 

B Ganesh shot 24 baskets and Ashish shot 16 baskets 

C Jugraj shot 19 baskets and Dhanraj shot 27 baskets 

D Dhanraj shot 11 baskets and Ashish shot 16 baskets 

Answer: A 

Explanation: 

Let a,d,j,g be the shots put by Ashish, Dhanraj, Ganesh and Jugraj respectively. 

According to given conditions we have , 

g = a-8; d + r = 37; j = d + 8; a = 5 + d; a + g = 40 

Solving these equations, we have a = 24, d = 19 and j = 27 and r = 18. Hence option A is the correct answer. 

 

Instructions 

DIRECTIONS for the following three questions: Answer the questions on the basis of the information given below. 

A city has two perfectly circular and concentric ring roads, the outer ring road (OR) being twice as long as the inner ring road (IR). There are also four (straight line) chord roads from E1, the east end point of OR to N2, the north end point of IR; from N1, the north end point of OR to W2, the west end point of IR; from W1, the west end point of OR, to S2, the south end point of IR; and from S1 the south end point of OR to E2, the east end point of IR. Traffic moves at a constant speed of 30π km/hr on the OR road, 20π km/hr on the IR road, and 15√5 km/hr on all the chord roads. 

Q. 99 Amit wants to reach N2 from S1. It would take him 90 minutes if he goes on minor arc S1 – E1 on OR, and then on the chord road E1 – N2. What is the radius of the outer ring road in kms? 

A 60 

B 40 

C 30 

D 20 

Answer: C 

Explanation: 

We know that the total time taken is 1.5 hrs. Calculating the individual time taken and the adding and then equating to 1.5.

R)/(2∗30∗π) + (√5∗R)/ 2∗15∗√5 = 1.5 

solving we get R=30. 

Q. 100 Amit wants to reach E2 from N1 using first the chord N1 – W2 and then the inner ring road. What will be his travel time in minutes on the basis of information given in the above question? 

A 60 

B 45. 

C 90 

D 105 

Answer: D 

Explanation: 

Let the radii of 2 circles be R and r respectively such that R=2*r. Triangle O $$N_2E_1 And all the other 3 similar triangles form a right angle at the centre. So using pythagoras theorem the value of chords come out to be 52∗R/2.Hence the total distance traveled is 52* R/2 + 0.5*R*pi. Total time required can be calculated by distance / speed which comes out to be 3.5*R. Among options only 105 is integral multiple of 3.5. 

 

 

Verbal 

Instructions 

At the heart of the enormous boom in wine consumption that has taken place in the English speaking world over the last two decades or so is a fascinating, happy paradox. In the days when wine was exclusively the preserve of a narrow cultural elite, bought either at auctions or from gentleman wine merchants in wing collars and bow-ties, to be stored in rambling cellars and decanted to order by one’s butler, the ordinary drinker didn’t get a look-in. Wine was considered a highly technical subject, in which anybody without the necessary ability could only fall flat on his or her face in embarrassment. It wasn’t just that you needed a refined aesthetic sensibility for the stuff if it wasn’t to be hopelessly wasted on you. It required an intimate knowledge of what came from where, and what it was supposed to taste like. 

Those were times, however, when wine appreciation essentially meant a familiarity with the great French classics, with perhaps a smattering of other wines — like sherry and port. That was what the wine trade dealt in. These days, wine is bought daily in supermarkets and high-street chains to be consumed that evening, hardly anybody has a cellar to store it in and most don’t even possess a decanter. Above all, the wines of literally dozens of countries are available on our market. When a supermarket offers its customers a couple of fruity little numbers from Brazil, we scarcely raise an eyebrow. 

It seems, in other words, that the commercial jungle that wine has now become has not in the slightest deterred people from plunging adventurously into the thickets in order to taste and see. Consumers are no longer intimidated by the thought of needing to know their Pouilly-Fume from their Pouilly-Fuisse, just at the very moment when there is more to know than ever before. 

The reason for this new mood of confidence is not hard to find. It is on every wine label from Australia, New Zealand, South Africa and the United States: the name of the grape from which the wine is made. At one time that might have sounded like a fairly technical approach in itself. Why should native English-speakers know what Cabernet Sauvignon or Chardonnay were? The answer lies in the popularity that wines made from those grape varieties now enjoy. Consumers effectively recognize them as brand names, and have acquired a basic lexicon of wine that can serve them even when confronted with those Brazilian upstarts. 

In the wine heartlands of France, they are scared to death of that trend—not because they think their wine isn’t as good as the best from California or South Australia (what French winemaker will ever admit that?) but because they don’t traditionally call their wines Cabernet Sauvignon or Chardonnay. They call them Château Ducru Beaucaillou or Corton Charlemagne, and they aren’t about the change. Some areas, in the middle of southern France, have now produced a generation of growers using the varietal names on their labels and are tempting consumers back to French wine. It will be an uphill struggle, but there is probably no other way if France is to avoid simply becoming a specialty source of old fashioned wines for old fashioned connoisseurs. 

Wine consumption was also given a significant boost in the early 1990s by the work of Dr. Serge Renaud, who has spent many years investigating the reasons for the uncannily low incidence of coronary heart disease in the south of France. One of his major findings is that the fat-derived cholesterol that builds up in the arteries and can eventually lead to heart trouble, can be dispersed by the tannins in wine. Tannin is derived from the skins of grapes, and is therefore present in higher levels in red wines, because they have to be infused with their skins to attain the red colour. That news caused a 

huge upsurge in red wine consumption in the United States. It has not been accorded the prominence it deserves in the UK, largely because the medical profession still sees all alcohol as a menace to health, and is constantly calling for it to be made prohibitively expensive. Certainly, the manufacturers of anticoagulant drugs might have something to lose if we all got the message that we would do just as well by our hearts by taking half a bottle of red wine every day! 

Q. 101 The tone that the author uses while asking “what French winemaker will ever admit that?” is best described as 

A caustic 

B satirical 

C critical 

D hypocritical 

Answer: B 

Explanation: 

The tone which the author uses while asking “what French winemaker will ever admit that? ” is not at all harsh , so option a) is out . Also the author doesn’t criticize while asking the Q. ,so Option c ) is clearly not the answer. We don’t find any author professing any feelings which he doesn’t have , hence the tone is not hypocritical. Option b ,suits the best i.e. his tone is satirical. 

Q. 102 What according to the author should the French do to avoid becoming a producer of merely old fashioned wines? 

A Follow the labeling strategy of the English-speaking countries 

B Give their wines English names 

C Introduce fruity wines as Brazil has done 

D Produce the wines that have become popular in the English-speaking world 

Answer: A 

Explanation: 

Refer to the part of the passage which says ‘ …not because they think their wine isn’t as good as the best from California …and they aren’t about the change…probably no other way if France is to avoid simply becoming a specialty source of old fashioned wines for old fashioned connoisseurs. ‘ . 

From this, we can see that the author feels that the French should adopt the labeling strategy of the English-speaking countries to avoid becoming a producer of merely old-fashioned wines. 

Option a) is the correct answer. 

Q. 103 The development which has created fear among winemakers in the wine heartland of France is the 

A tendency not to name wines after the grape varieties that are used in the wines 

B ‘education’ that consumers have derived from wine labels from English speaking countries. C new generation of local winegrowers who use labels that show names of grape varieties 

D ability of consumers to understand a wine’s qualities when confronted with “Brazilian upstarts”. 

Answer: B 

Explanation: 

Consider the following part of the passage ‘ .. Consumer effectively recognize them as brand names, and have acquired a basic lexicon of wine …. with those Brazilian upstarts. In the wine heartlands of France, they are scared to death of that trend.. ‘. So, above all, French winemakers fear the knowledge or education that the consumers have derived from wine labels from the English speaking countries. Option b) is the correct answer. 

Q. 104 Which one of the following, if true, would provide most support for Dr. Renaud’s findings about the effect of tannins? 

A A survey showed that film celebrities based in France have a low incidence of coronary heart disease. 

B Measurements carries out in southern France showed red wine drinkers had significantly higher levels of coronary heart incidence than white wine drinkers did. 

C Data showed a positive association between sales of red wine and incidence of coronary heart disease. 

D Long-term surveys in southern France showed that the incidence of coronary heart disease was significantly lower in red wine drinkers than in those who did not drink red wine. 

Answer: D 

Explanation: 

Dr. Renaud findings suggest that fat-derived cholesterol can be dispersed by the tannins in wine. So, a survey that validates this finding would provide the most support. The survey in option d) is precisely one such survey. Option d) is the correct answer. 

Q. 105 Which one of the following CANNOT be reasonably attributed to the labeling strategy of followed by wine producers in English speaking countries? 

A Consumers buy wines on the basis of their familiarity with a grape variety’s name. 

B Even ordinary customers now have more access to technical knowledge about wine. 

C Consumers are able to appreciate better quality wines. 

D Some non-English speaking countries like Brazil indicate grape variety names on their labels. Answer: C 

Explanation: 

Consumers’ appreciation of better quality wines is something that does not come because of the labeling or branding. So, this appreciation cannot be attributed to the labeling strategy followed by wine producers in English speaking countries. 

Option c) is the correct answer. 

 

Instructions 

Right through history, imperial powers have clung to their possessions to death. Why, then, did Britain in 1947 give up the jewel in its crown, India? For many reasons. The independence struggle exposed the hollowness of the white man’s burden. Provincial self-rule since 1935 paved the way for full self-rule. Churchill resisted independence, but the Labour government 

of Atlee was anti-imperialist by ideology. Finally, the Royal Indian Navy mutiny in 1946 raised fears of a second Sepoy mutiny, and convinced British waverers that it was safer to withdraw gracefully. But politico-military explanations are not enough. The basis of empire was always money. The end of empire had much to do with the fact that British imperialism had ceased to be profitable. World War II left Britain victorious but deeply indebted, needing Marshall Aid and loans from the World Bank. This constituted a strong financial case for ending the no-longer profitable empire. 

Empire building is expensive. The US is spending one billion dollars a day in operations in Iraq that fall well short of full scale imperialism. Through the centuries, empire building was costly, yet constantly undertaken because it promised high returns. The investment was in armies and conquest. The returns came through plunder and taxes from the conquered. 

No immorality was attached to imperial loot and plunder. The biggest conquerors were typically revered (hence titles like Alexander the Great, Akbar the Great, and Peter the Great). The bigger and richer the empire, the more the plunderer was admired. This mindset gradually changed with the rise of new ideas about equality and governing for the public good, ideas that culminated in the French and American revolutions. 

Robert Clive was impeached for making a little money on the side, and so was Warren Hastings. The white man’s burden came up as a new moral rationale for conquest. It was supposedly for the good of the conquered. This led to much muddled hypocrisy. On the one hand, the empire needed to be profitable. On the other hand, the white man’s burden made brazen loot impossible. 

An additional factor deterring loot was the 1857 Sepoy Mutiny. Though crushed, it reminded the British vividly that they were a tiny ethnic group who could not rule a gigantic subcontinent without the support of important locals. After 1857, the British stopped annexing one princely state after another, and instead treated the princes as allies. Land revenue was fixed in absolute terms, partly to prevent local unrest and partly to promote the notion of the white man’s burden. The empire proclaimed itself to be a protector of the Indian peasant against exploitation by Indian elites. This was denounced as hypocrisy by nationalists like Dadabhoy Naoroji in the 19th century, who complained that land taxes led to an enormous drain from India to Britain. Objective calculations by historians like Angus Maddison suggest a drain of perhaps 1.6 percent of Indian Gross National Product in the 19th century. But land revenue was more or less fixed by the Raj in absolute terms, and so its real value diminished rapidly with inflation in the 20th century. By World War II, India had ceased to be a profit center for the British Empire. 

Historically, conquered nations paid taxes to finance fresh wars of the conqueror. India itself was asked to pay a large sum at the end of World War I to help repair Britain’s finances. But, as shown by historian Indivar Kamtekar, the independence movement led by Gandhiji changed the political landscape, and made mass taxation of India increasingly difficult. By World War II, this had become politically impossible. Far from taxing India to pay for World War II, Britain actually began paying India for its contribution of men and goods. Troops from white dominions like Australia; Canada and New Zealand were paid for entirely by these countries, but Indian costs were shared by the British government. Britain paid in the form of nonconvertible sterling balances, which mounted swiftly. The conqueror was paying the conquered, undercutting the profitability on which all empire is founded. Churchill opposed this, and wanted to tax India rather than owe it money. But he was overruled by Indian hands who said India would resist payment, and paralyze the war effort. Leo Amery, Secretary of State for India, said that when you are driving in a taxi to the station to catch a life-or-death train, you do not loudly announce that you have doubts whether to pay the fare. Thus, World War II converted India from a debtor to a creditor with over one billion pounds in sterling balances. Britain, meanwhile, became the biggest debtor in the world. It’s not worth ruling over people you are afraid to tax. 

Q. 106 Why didn’t Britain tax India to finance its World War II efforts? 

A  Australia, Canada and New Zealand had offered to pay for Indian troops. 

B India has already paid a sufficiently large sum during World War I. 

C It was afraid that if India refused to pay, Britain’s war efforts would be jeopardized. 

D The British empire was built on the premise that the conqueror pays the conquered. 

Answer: C 

Explanation: 

Consider the following lines from the passage: “But he was overruled by Indian hands who said India would resist payment and paralyse the war effort”. From this, we can understand the reason why the British didn’t tax India to finance its war efforts. It was afraid that if India refused to pay, Britain’s war efforts would be jeopardized. Option c) is the correct answer. 

Q. 107 What was the main lesson the British learned from the Sepoy Mutiny of 1857. 

A That the local princes were allies, not foes. 

B That the land revenue from India would decline dramatically. 

C That the British were a small ethnic group. 

D That India would be increasingly difficult to rule. 

Answer: C 

Explanation: 

Refer to the 5th paragraph. It says “Though crushed, it reminded the British vividly that they were a tiny ethnic group who could not rule a gigantic subcontinent without the support of the important locals”. From this, we can understand that the main lesson that the British learnt from the Sepoy Mutiny of 1857 was that they were a small ethnic group. Option c) is the correct answer

Q. 108 Which of the following was NOT a reason for the emergence of the ‘white man’s burden’ as a new rationale for empire-building in India? 

A The emergence of the idea of the public good as an element of governance. 

B The decreasing returns from imperial loot and increasing costs of conquest. 

C The weakening of the immorality attached to an emperor’s looting behaviour. 

D A growing awareness of the idea of equality among peoples. 

Answer: B 

Explanation: 

If the returns from conquest decreased and the costs increased, it wouldn’t make sense to continue empire-building in India. So, the sentence in option b) is not a reason for the emergence of the ‘white man’s burden’ as a new rationale for empire-building in India. 

Q. 109 Which of the following best captures the meaning of the ‘white man’s burden’, as it is used by the author? 

 

A The British claim to a civilizing mission directed at ensuring the good of the natives. 

B The inspiration for the French and American revolutions. 

C The resource drain that had to be borne by the home country’s white population. 

D An imperative that made open looting of resources impossible. 

Answer: A 

Explanation: 

White man’s burden refers to the claim made by the British that the natives of the conquered countries were in need of the ‘good’ provided by them. This was a justification for their conquests. Option a) captures this idea succinctly. 

Q. 110 Which one of the following best expresses the main purpose of the author? 

A To present the various reasons that can lead to the collapse of an empire and the granting of independence of the subjects of an empire. 

B To point out the critical role played by the ‘white man’s burden’ in making a colonizing power give up its claims to native possessions. 

C To highlight the contradictory impulse underpinning empire building which is a costly business but very attractive at the same time. 

D To illustrate how erosion of the financial basis of an empire supports the granting of independence to an empire’s constituents. 

Answer: D 

Explanation: 

Throughout the passage, the author talks about the various financial reasons for conquest and explains how the British 

were forced to grant independence when their returns from India diminished after the war. The main idea of the passage is to illustrate how the erosion of the financial basis of an empire supports the granting of independence to an empire’s constituents. Option d) is the correct answer.  

 

Instructions 

The controversy over genetically modified food continues unabated in the West. Genetic modification (GM) is the science by which the genetic material of a plant is altered, perhaps to make it more resistant to pests or killer weeds, or to enhance its nutritional value. Many food biotechnologists claim that GM will be a major contribution of science to mankind in the 21st century. On the other hand, large numbers of opponents, mainly in Europe, claim that the benefits of GM are a myth propagated by multinational corporations to increase their profits, that they pose a health hazard, and have therefore called for government to ban the sale of genetically-modified food. 

The anti-GM campaign has been quite effective in Europe, with several European Union member countries imposing a virtual ban for five years over genetically-modified food imports. Since the genetically-modified food industry is particularly strong in the United States of America, the controversy also constitutes another chapter in the US-Europe skirmishes which have become particularly acerbic after the US invasion of Iraq. 

To a large extent, the GM controversy has been ignored in the Indian media, although Indian biotechnologists have been quite active in GM research. Several groups of Indian biotechnologists have been working on various issues connected with crops grown in India. One concrete achievement which has recently figured in the news is that of a team led by the former vice-chancellor of Jawaharlal Nehru university, Asis Datta — it has successfully added an extra gene to potatoes to enhance the protein content of the tuber by at least 30 percent. It is quite likely that the GM controversy will soon hit the headlines in India since a spokesperson of the Indian Central government has recently announced that the government may use the protato in its midday meal programme for schools as early as next year. 

Why should “scientific progress”, with huge potential benefits to the poor and malnourished, be so controversial? The anti GM lobby contends that pernicious propaganda has vastly exaggerated the benefits of GM and completely evaded the costs which will have to be incurred if the genetically-modified food industry is allowed to grow unchecked. In particular, they allude to different types of costs. 

This group contends that the most important potential cost is that the widespread distribution and growth of genetically modified food will enable the corporate world (alias the multinational corporations – MNCs) to completely capture the food chain. A “small” group of biotech companies will patent the transferred genes as well as the technology associated with them. They will then buy up the competing seed merchants and seed-breeding centers, thereby controlling the production of food at every possible level. Independent farmers, big and small, will be completely wiped out of the food industry. At best, they will be reduced to the status of being subcontractors. 

This line of argument goes on to claim that the control of the food chain will be disastrous for the poor since the MNCs, guided by the profit motive, will only focus on the high-value food items demanded by the affluent. Thus, in the long run, the production of basic staples which constitute the food basket of the poor will taper off. However, this vastly overestimates the power of the MNCs. Even if the research promoted by them does focus on the high-value food items, much of biotechnology research is also funded by governments in both developing and developed countries. Indeed, the protato is a by-product of this type of research. If the protato passes the field trials, there is no reason to believe that it cannot be marketed in the global potato market. And this type of success story can be repeated with other basic food items. 

The second type of cost associated with the genetically modified food industry is environmental damage. The most common type of “genetic engineering” involved gene modification in plants designed to make them resistant to applications of weed-killers. This then enables farmers to use massive dosages of weedkillers so as to destroy or wipe out all competing varieties of plants in their field. However, some weeds through genetically-modified pollen contamination may acquire resistance to a variety of weed-killers. The only way to destroy these weeds is through the use of ever stronger herbicides which are poisonous and linger on in the environment. 

Q. 111 The author doubts the anti-GM lobby’s contention that MNC control of the food chain will be disastrous for the poor because 

A MNCs will focus on high-value food items. 

B MNCs are driven by the motive of profit maximization. 

C MNCs are not the only group of actors in genetically-modified food research. 

D Economic development will help the poor buy MNC-produced food. 

Answer: C 

Explanation: 

By the line “Even if the research promoted by them ………………. in both developing and developed countries”, we can say that not only MNCs but also governments are involved in the research development. 

So, MNCs are not the only group actors that are involved in genetically modified food research. Hence, option C is the answer. 

Q. 112 Using the clues in the passage, which of the following countries would you expect to be in the forefront of the anti-GM campaign? 

A USA and Spain. 

B India and Iraq. 

C Germany and France. 

D Australia and New Zealand. 

Answer: C 

Explanation: 

According to the passage, European nations are anti GM. So, among the given options we must select those countries that are present in Europe. 

USA is not in Europe => option A is wrong. 

India is not in Europe => option B is wrong. 

Australia is not in Europe => option D is wrong. 

Both Germany and France are in Europe => option C is the answer. 

 

Q. 113 Genetic modification makes plants more resistant to killer weeds. However, this can lead to environmental damage by 

A wiping out competing varieties of plants which now fall prey to killer weeds. 

B forcing application of stronger herbicides to kill weeds which have become resistant to weak herbicides. C forcing application of stronger herbicides to keep the competing plants weed-free. 

D not allowing growth of any weeds, thus reducing soil fertility. 

Answer: B 

Explanation: 

Refer to the last lines of the passage:”However, some weeds through genetically-modified pollen contamination may acquire resistance to a variety of weed-killers. The only way to destroy these weeds is through the use of ever-stronger herbicides which are poisonous and linger on in the environment.” This line indicates the point made in 2 that once the weeds acquire resistance to weak herbicides, we have to apply stronger ones to eradicate them. 

Q. 114 According to the passage, biotechnology research 

A is of utility only for high value food items. 

B is funded only by multinational corporations. 

C allows multinational corporations to control the food basket of the poor. 

D addresses the concerns of developed and developing countries. 

Answer: D 

Explanation: 

Refer to the lines made in the paragraph:”Even if the research promoted by them does focus on the high-value food items, much of biotechnology research is also funded by governments in both developing and developed countries. Indeed, the protato is a by-product of this type of research. If the protato passes the field trials, there is no reason to believe that it cannot be marketed in the global potato market. And this type of success story can be repeated with other basic food items.” Here the author wants to illustrate that biotechnology resarch helps to address the concerns of the developing countries. For this illustration, the author gives the exmaple of potatoes. 

Q. 115 Which of the following about the Indian media’s coverage of scientific research does the passage seem to suggest? 

A Indian media generally covers a subject of scientific importance when its mass application is likely. 

B Indian media’s coverage of scientific research is generally dependent on MNCs interests. 

C Indian media, in partnership with the government, is actively involved in publicizing the results of scientific research. 

D Indian media only highlights scientific research which is funded by the government. 

Answer: A 

Explanation: 

Refer to the following lines in the passage:”It is quite likely that the GM controversy will soon hit the headlines in India since a spokesperson of the Indian Central government has recently announced that the government may use the protato in its midday meal programme for schools as early as next year. Why should “scientific progress”, with huge potential benefits to the poor and malnourished, be so controversial?” Here the author wants to highlight that the scientific progress which has a huge impact on large number of people is likely to be covered by the media. 

 

Instructions 

Social life is an outflow and meeting of personality, which means that its end is the meeting of character, temperament, and sensibility, in which our thoughts and feelings, and sense perceptions are brought into play at their lightest and yet keenest. 

This aspect, to my thinking, is realized as much in large parties composed of casual acquaintances or even strangers, as in intimate meetings of old friends. I am not one of those superior persons who hold cocktail parties in contempt, looking upon them as barren or at best as very tryingly kaleidoscopic places for gathering, because of the strangers one has to meet in them; which is no argument, for even our most intimate friends must at one time have been strangers to us. These large gatherings will be only what we make of them if not anything better, they can be as good places to collect new friends from as the slave markets of Istanbul were for beautiful slaves or New Market for race horses. 

But they do offer more immediate enjoyment. For one thing, in them one can see the external expression of social life in appearance and behaviour at its widest and most varied where one can admire beauty of body or air, hear voices remarkable either for sweetness of refinement, look on elegance of clothes or deportment. What is more, these parties are schools for training in sociability, for in them we have to treat strangers as friends. So, in them we see social sympathy in widest commonalty spread, or at least should. We show an atrophy of the natural human instinct of getting pleasure and happiness out of other human beings if we cannot treat strangers as friends for the moment. And I would go further and paraphrase Pater to say that not to be able to discriminate every moment some passionate attitude in those about us, even when we meet them casually, is on this short day of frost and sun which out life is, to sleep before evening. 

So, it will be seen that my conception of social life is modest, for it makes no demands on what we have, though it does make some on what we are. Interest, wonder, sympathy, and love, the first two leading to the last two, are the psychological prerequisites for social life; and the need for the first two must not be underrated. We cannot make the most even of our intimate social life unless we are able to make strangers of our oldest friends everyday by discovering unknown areas in their personality, and transform them into new friends. In sum, social life is a function of vitality. 

It is tragic, however, to observe that it is these very natural springs of social life which are drying up among us. It is becoming more and more difficult to come across fellow-feeling for human beings as such in our society and in all its strata. 

In the poor middle class, in the course of all my life. I have hardly seen any social life properly so-called. Not only has the grinding routine of making a living killed all desire for it in them, it has also generated a standing mood of peevish hostility to other human beings. Increasing economic distress in recent years has infinitely worsened this state of affairs, and has also brought a sinister addition class hatred. This has become the greatest collective emotional enjoyment of the poor middle class, and indeed they feel most social when they form a pack, and snarl or howl at people who are better off than they. 

Their most innocent exhibition of sociability is seen when they spill out from their intolerable homes into the streets and bazaars. I was astonished to see the milling crowds in the poor suburbs of Calcutta. But even there a group of flippant young loafers would put on a conspiratorial look if they saw a man in good clothes passing by them either on foot or in a car. I had borrowed a car from a relative to visit a friend in one of these suburbs, and he became very anxious when I had not returned before dusk. Acid and bombs, he said, were thrown at card almost every evening in that area. I was amazed. But I also know as a fact that my brother was blackmailed to pay five rupees on a trumped up charge when passing in a car through one such locality. 

The situation is differently inhuman, but not a whit more human, among the well-to-do. Kindliness for fellow human beings has been smothered in them, taken as a class, by the arrogance of worldly position, which among the Bengalis who show this snobbery is often only a third-class position. 

Q. 116 The word ‘they’ in the first sentence of the third paragraph refers to 

A Large parties consisting of casual acquaintances and strangers. 

B Intimate meetings of old friends. 

C New friends. 

D Both (1) and (2). 

Answer: A 

Explanation: 

Refer to the last lines of the second paragraph:”These large gatherings will be only what we make of them if not anything better, they can be as good places to collect new friends from as the slavemarkets of Istanbul were for beautiful slaves or New Market for race horses.” 

Here “They” refers to the large gatherings of casual acquaintances as illustrated in these lines

Q. 117 In this passage the author is essentially 

A showing how shallow our social life is. 

B poking fun at the lower middle class people who howl at better off people. 

C lamenting the drying up of our real social life. 

D criticizing the upper class for lavish showy parties. 

Answer: C 

Explanation: 

The passage starts with the author’s perception of social life. In the whole passage the author criticises the fact that we do not possess any social life. The author also gives his opinions of social life and then moves on to lament our little social life we have left. Option b and d are not clearly the main subject of the author. Option a is inappropriate as the author do not highlight our real social life but rather complaints of the social life. 

Q. 118 The author’s conception of ‘social life’ requires that 

A People attend large gatherings. 

B People possess qualities like wonder and interest. 

C People do not spend too much time in the company of intimate friends. 

D Large parties consist of casual acquaintances and intimate friends. 

Answer: B 

Explanation: 

The author’s conception of social life is mentioned in the fourth paragraph where he says wonder and interest. Hence B is the correct answer. 

Q. 119 The word ‘discriminate’ in the last sentence of the third paragraph means 

A recognize. 

B count. 

C distinguish. 

D analyse. 

Answer: A 

Explanation: 

The author is trying to say that people are not able to recognize the passionate attitude. 

So, the word “discriminate” in this context means “recognize”.  

Q. 120 What is the author trying to show through the two incidents in the paragraph beginning, “Their most innocent exhibition of sociability…”? 

A The crowds in poor Calcutta suburbs can turn violent without any provocation. 

B Although poor, the people of poor Calcutta suburbs have a rich social life. 

C It is risky for rich people to move around in poor suburbs. 

D Achieving a high degree of sociability foes not stop the poor from hating the rich. 

Answer: D 

Explanation: 

The author has not mentioned that crowd in poor Calcutta can turn violent => option A is wrong. B negates the statements said by the author in the passage. 

C is too generalized to be the answer for this question. 

Hence, option D is the answer. 

Instructions 

Modern science, exclusive of geometry, is a comparatively recent creation and can be said to have originated with Galileo and Newton. Galileo was the first scientist to recognize clearly that the only way to further our understanding of the physical world was to resort to experiment. However obvious Galileo’s contention may appear in the light of our present knowledge, it remains a fact that the Greeks, in spite of their proficiency in geometry, never seem to have realized the importance of experiment. To a certain extent this may be attributed to the crudeness of their instruments of measurement. Still an excuse of this sort can scarcely be put forward when the elementary nature of Galileo’s experiments 

and observations is recalled. Watching a lamp oscillate in the cathedral of Pisa, dropping bodies from the leaning tower of Pisa, rolling balls down inclined planes, noticing the magnifying effect of water in a spherical glass vase, such was the nature of Galileo’s experiments and observations. As can be seen, they might just as well have been performed by the Greeks. At any rate, it was thanks to such experiments that Galileo discovered the fundamental law of dynamics, according to which the acceleration imparted to a body is proportional to the force acting upon it. 

The next advance was due to Newton, the greatest scientist of all time if account be taken of his joint contributions to mathematics and physics. As a physicist, he was of course an ardent adherent of the empirical method, but his greatest title to fame lies in another direction. Prior to Newton, mathematics, chiefly in the form of geometry, had been studied as a fine art without any view to its physical applications other than in very trivial cases. But with Newton all the resources of mathematics were turned to advantage in the solution of physical problems. Thenceforth mathematics appeared as an instrument of discovery, the most powerful one known to man, multiplying the power of thought just as in the mechanical domain the lever multiplied our physical action. It is this application of mathematics to the solution of physical problems, this combination of two separate fields of investigation, which constitutes the essential characteristic of the Newtonian method. Thus problems of physics were metamorphosed into problems of mathematics. 

But in Newton’s day the mathematical instrument was still in a very backward state of development. In this field again Newton showed the mark of genius by inventing the integral calculus. As a result of this remarkable discovery, problems, which would have baffled Archimedes, were solved with ease. We know that in Newton’s hands this new departure in scientific method led to the discovery of the law of gravitation. But here again the real significance of Newton’s achievement lay not so much in the exact quantitative formulation of the law of attraction, as in his having established the presence of law and order at least in one important realm of nature, namely, in the motions of heavenly bodies. Nature thus exhibited rationality and was not mere blind chaos and uncertainty. To be sure, Newton’s investigations had been concerned with but a small group of natural phenomena, but it appeared unlikely that this mathematical law and order should turn out to be restricted to certain special phenomena; and the feeling was general that all the physical processes of nature would prove to be unfolding themselves according to rigorous mathematical laws. 

When Einstein, in 1905, published his celebrated paper on the electrodynamics of moving bodies, he remarked that the difficulties, which surrouned the equations of electrodynamics, together with the negative experiments of Michelson and others, would be obviated if we extended the validity of the Newtonian principle of the relativity of Galilean motion, which applies solely to mechanical phenomena, so as to include all manner of phenomena: electrodynamics, optical etc. When extended in this way the Newtonian principle of relativity became Einstein’s special principle of relativity. Its significance lay in its assertion that absolute Galilean motion or absolute velocity must ever escape all experimental detection. Henceforth absolute velocity should be conceived of as physically meaningless, not only in the particular ream of mechanics, as in Newton’s day, but in the entire realm of physical phenomena. Einstein’s special principle, by adding increased emphasis to this relativity of velocity, making absolute velocity metaphysically meaningless, created a still more profound distinction between velocity and accelerated or rotational motion. This latter type of motion remained absolute and real as before. It is most important to understand this point and to realize that Einstein’s special principle is merely an extension of the validity of the classical Newtonian principle to all classes of phenomena. 

Q. 121 According to the author, why did the Greeks NOT conduct experiments to understand the physical world? 

A Apparently they did not think it necessary to experiment. 

B They focused exclusively on geometry. 

C Their instruments of measurement were very crude. 

D The Greeks considered the application of geometry to the physical world more important. Answer: A 

Explanation: 

Options B and D negate the information given in the passage => B and D are incorrect. 

C is stated in the passage but does not answer the question. 

Option A is the correct answer. 

Q. 122 The statement “Nature thus exhibited rationality and was not mere blind chaos and uncertainty” suggests that 

A problems that had baffled scientists like Archimedes were not really problems. 

B only a small group of natural phenomena was chaotic. 

C physical phenomena conformed to mathematical laws. 

D natural phenomena were evolving towards a less chaotic future. 

Answer: C 

Explanation: 

In the second line after the line mentioned in the question, the author says that “the feeling was general that all the physical processes of nature would prove to be unfolding themselves according to the rigorous mathematical laws”. 

Option C is the answer. 

Q. 123 Newton may be considered one of the greatest scientists of all time because he 

A discovered the law of gravitation. 

B married physics with mathematics. 

C invented integral calculus. 

D started the use of the empirical method in science. 

Answer: B 

Explanation: 

The author says that “The next advance was due to Newton, the greatest scientist of all time if account be taken of his joint contributions to mathematics and physics.” 

Joint contributions is metaphorically said as married in option B. Hence, option B is the answer.

Q. 124 Which of the following statements about modern science best captures the theme of the passage? 

A Modern science rests firmly on the platform built by the Greeks. 

B We need to go back to the method of enquiry used by the Greeks to better understand the laws of dynamics. 

C Disciplines like Mathematics and Physics function best when integrated into one. 

D New knowledge about natural phenomena builds on existing knowledge. 

Answer: D 

Explanation: 

The author says that Einstein’s principle is merely an extension of classical Newtonian principle. Option D agrees with this saying that new knowledge about natural phenomena builds on existing knowledge. Hence, option D is the answer. 

Q. 125 The significant implication of Einstein’s special principle of relativity is that 

A absolute velocity was meaningless in the realm of mechanics. 

B Newton’s principle of relativity needs to be modified. 

C there are limits to which experimentation can be used to understand some physical phenomena. 

D it is meaningless to try to understand the distinction between velocity and accelerated or rotational motion. 

Answer: C 

Explanation: 

The author says that “Its SIGNIFICANCE lay in its assertion that absolute Galilean motion or absolute velocity must ever escape all experimental detection.” 

Here, “it” refers to Einstein’s principle. 

The meaning of the sentence is that it is not always possible to experiment. 

Option C gives a similar meaning. Hence, C is the answer. 

 

Instructions 

As you set out for Ithaka 

hope the journey is a long one, 

full of adventure, full of discovery. 

Laistrygonians and Cyclops, 

angry Poseidon – don’t be afraid of them: 

you’ll never find things like that on your way 

as long as you keep your thoughts raised high, 

as long as a rare excitement 

stirs your spirit and your body. 

Laistrygonians and Cyclops, 

wild Poseidon – you won’t encounter them 

unless you bring them along inside your soul, 

unless your soul sets them up in front of you. 

Hope the voyage is a long one, 

may there be many a summer morning when, 

with what pleasure, what joy, 

you come into harbours seen for the first time; 

may you stop at Phoenician trading stations 

to buy fine things, 

mother of pearl and coral, amber and ebony, 

sensual perfume of every kind – 

as many sensual perfumes as you can; 

and may you visit many Egyptian cities 

to gather stores of knowledge from their scholars. 

Keep Ithaka always in your mind. 

Arriving there is what you are destined for. 

But do not hurry the journey at all. 

Better if it lasts for years, 

so you are old by the time you reach the island, 

wealthy with all you have gained on the way, 

not expecting Ithaka to make you rich. 

Ithaka gave you the marvelous journey, 

without her you would not have set out. 

She has nothing left to give you now. 

And if you find her poor, Ithaka won’t have fooled you. 

Wise as you will have become, so full of experience, 

you will have understood by then what these Ithakas mean. 

 

Q. 126 Which of the following best reflects the central theme of this poem? 

A If you don’t have high expectations, you will not be disappointed. 

B Don’t rush to your goal; the journey is what enriches you. 

C The longer the journey the greater the experiences you gather. 

D You cannot reach Ithaka without visiting Egyptian ports. 

Answer: B 

Explanation: 

The four lines “Better if it last for years ………… not expecting Ithaka to make you rich” gives us the central theme of the poem. 

It says that the journey is more important than the goal. 

This meaning is conveyed in option B. 

Hence, option B is the answer. 

 

Q. 127 The poet recommends a long journey. Which of the following is the most comprehensive reason for it? 

A You can gain knowledge as well as sensual experience. 

B You can visit new cities and harbours. 

C You can experience the full range of sensuality. 

D You can buy a variety of fine things. 

Answer: A 

Explanation: 

Option A gives the big picture of why the poet recommends a long journey. 

The remaining options are short-sighted and the poet was not much concerned about them. 

Q. 128 In the poem, Ithaka is a symbol of 

A the divine mother. 

B your inner self. 

C the path to wisdom. 

D life’s distant goal. 

Answer: D 

Explanation: 

The whole poem was about the goal and the journey to reach the goal. 

If we see the lines “Better if it lasts for years …………. not expecting Ithaka to make your rich”, we can see that the poet is comparing Ithaka to life’s distant goal. 

Option D is the answer. 

Q. 129 What does the poet mean by ‘Laistrygonians’ and ‘Cyclops’? 

A Creatures which, along with Poseidon, one finds during a journey. 

B Mythological characters that one should not be afraid of. 

C Intra-personal obstacles that hinder one’s journey. 

D Problems that one has to face to derive the most from one’s journey. 

Answer: C 

Explanation: 

The poet is trying to say that one who pursues the journey to one’s goal must not be afraid of hindrances in that journey. Option C is similar to our conclusion. 

Hence, option C is the answer. 

Q. 130 Which of the following best reflects the tone of the poem? 

A Prescribing. 

B Exhorting. 

C Pleading. 

D Consoling. 

Answer: B 

Explanation: 

The poet says that Ithaka is the reason for one’s journey. 

So, he is trying to be encouraging. 

Among the given options, only exhorting fits the be correct answer. 

Instructions 

For the following questions answer them individually 

Q. 131 Choose the option that conforms most closely to Standard English usage. 

A The running of large businesses consist of getting somebody to make something that somebody else sold to somebody else for more than its cost. 

B The running of a large business consists of getting somebody to make something that somebody else will sell to somebody else for more than it costs. 

C The running of a large business consists of getting somebody to sell something that somebody else made for more than it cost. 

D The running of large businesses consist of getting somebody to make something else that somebody else will sell to somebody else for more than it costs. 

E None of the above 

Answer: B 

Explanation: 

Option a and d are grammatically incorrect as “The running of large business” is singular and hence should be followed by “consists.” Option c doesn’t make any sense as it is contextually incorrect. Option b is the most appropriate.  

Q. 132 Choose the option that conforms most closely to Standard English usage. 

A From the sixteenth century onwards, people started feeling disdainful and self-conscious about their body and its products that led to a heightened focus on emotional and bodily regulations. 

B The heightened focus on controlling the body and emotions comes from disdain and selfconsciousness about the body and its products, found in the sixteenth century. 

C From the sixteenth century onwards, a growing disdain for and self-consciousness about the body and its products took hold, leading to a heightened focus on emotional and bodily regulation. 

D The heightened focus on emotional and bodily regulations started from the sixteenth century onwards, when people felt disdain and self-consciousness about the body and its products. 

Answer: C 

Explanation: 

Option a is incorrect as the usage of “feeling disdainful” is incorrect. Option d is also incorrect due to the usage of “felt disdain.” Option b is having a tense error as there is a present form used in the sentence. 

Q. 133 Choose the option that conforms most closely to Standard English usage. 

A We are forced to fall back on fatalism as an explanation of irrational events. 

B We are forced to falling back on the fatalism as an explanation of irrational events. 

C We are forced to fall back on fatalism as explanations of irrational events. 

D We are forced to fall back to fatalism as an explanation of irrational events. 

Answer: A 

Explanation: 

Option d is incorrect as “to fatalism” is incorrect, option b is incorrect as the usage of “falling back” is incorrect. Option c is incorrect as “falling back on fatalism” is singular and hence the correct usage should be “explanation” rather than “explanations.” 

Q. 134 Choose the option that conforms most closely to Standard English usage. 

A Creativity in any field is regarded not only as valuable for itself but also as a service to the nation. 

B Creativity in any field is not regarded only as valuable on its own, but also as a service to the nation. 

C Creativity, in any field, is not only regarded as valuable, but also as a service to the nation. 

D Creativity in any field is regarded not only as valuable in itself but also as a service to the nation. 

Answer: D 

Explanation: 

The correct usage would be “The creativity is regarded not only as” 

Or in order to infer the correct usage we would break the sentence into 2 parts “Creativity in any field is regarded as valuable in itself and also regarded as a service to the nation”. 

Q. 135 Choose the option that conforms most closely to Standard English usage. 

AIf precision of thought had facilitated precision of behaviour, and if reflection had preceded action, it would be ideal for humans. 

B It would be ideal for humans if reflection preceded action and precision of thought facilitated precision of behaviour. 

C It would be ideal for humans if precedence of reflection was followed by action and precision of thought, by precise behaviour. 

D It would have been ideal for humans, if precise action and behaviour preceded precise reflection. 

Answer: B 

Explanation: 

In this sentence, the ideal situation is satisfactory. “Reflection” should precede action and thought should facilitate behaviour. 

Q. 136 Choose the most logical order of sentences among the given choices to construct a coherent paragraph. 

A. A few months ago I went to Princeton University to see what the young people who are going to be running our country in a few decades are like. 

B. I would go to sleep in my hotel room around midnight each night, and when I awoke, my mailbox would be full of replies—sent at 1:15 a.m., 2:59 a.m., 3:23 a.m. 

C. One senior told me that she went to bed around two and woke up each morning at seven; she could afford that much rest because she had learned to supplement her full day of work by studying in her sleep. 

D. Faculty members gave me the names of a few dozen articulate students, and I sent them e mails, inviting them out to lunch or dinner in small groups. 

E. As she was falling asleep she would recite a math problem or a paper topic to herself; she would then sometimes dream about it, and when she woke up, the problem might be solved. 

A DABCE 

B DACEB 

C ADBCE 

D AECBD 

Answer: C 

Explanation: 

A is the best opening sentence. It introduces the author’s trip to Princeton University. 

This is followed by D, which talks about how the author prepared for his trip by gathering the email ids of a few Princeton students. 

B then follows and describes what happened when the author emailed the students of Princeton. The situation is then explained in sentence C and followed by sentence E. 

Q. 137 Choose the most logical order of sentences among the given choices to construct a coherent paragraph. A. Four days later, Oracle announced its own bid for PeopleSoft, and invited the firm’s board to a discussion. 

B. Furious that his own plans had been endangered, PeopleSoft’s boss, Craig Conway, called Oracle’s offer “diabolical”, and its boss, Larry Ellison, a “sociopath”. 

C. In early June, PeopleSoft said that it would buy J.D. Edwards, a smaller rival. 

D. Moreover, said Mr. Conway, he “could imagine no price nor combination of price and other conditions to recommend accepting the offer.” 

E. On June 12th, PeopleSoft turned Oracle down. 

A CABDE 

B CADBE 

C CEDAB 

D CAEBD 

Answer: A 

Explanation: 

The best opening sentence is C. It gives an introduction to the paragraph saying that Peoplesoft offered to by J.D.Edwards. This is followed by A, which talks about how Oracle wanted to buy Peoplesoft just 4 days later. Statement B then follows, by introducing Peoplesoft’s boss Craig Conway and quoting his statements. D follows B as it continues the statements of Mr.Craig. 

E is the concluding sentence. 

Q. 138 Choose the most logical order of sentences among the given choices to construct a coherent paragraph. 

A. Surrendered, or captured, combatants cannot be incarcerated in razor wire cages; this ‘war’ has a dubious legality. 

B. How can then one characterize a conflict to be waged against a phenomenon as war? C. The phrase ‘war against terror’, which has passed into the common lexicon, is a huge misnomer. 

D. Besides, war has a juridical meaning in international law, which has codified the laws of war, imbuing them with a humanitarian content. 

E.Terror is a phenomenon, not an entity—either State or non-State. 

A ECDBA 

B BECDA 

C EBCAD 

D CEBDA 

Answer: D 

Explanation: 

C opens the paragraph brilliantly. E follows C and explains why the phrase “war on terror” is a misnomer. B continues the explanation started in E. D then follows, and gives another reason why the phrase doesn’t hold water. A concludes the paragraph aptly. 

Q. 139 Choose the most logical order of sentences among the given choices to construct a coherent paragraph. 

A. I am much more intolerant of a human being’s shortcomings than I am of an animal’s, but in this respect I have been lucky, for most of the people I have come across have been charming. 

B. Then you come across the unpleasant human animal—the District Officer who drawled, ‘We chaps are here to help you chaps,’ and then proceeded to be as obstructive as possible. 

C. In these cases of course, the fact that you are an animal collector helps; people always seem delighted to meet someone with such an unusual occupation and go out of their way to assist you. 

D. Fortunately, these types are rare, and the pleasant ones I have met more than compensated for them—but even so, I think I will stick to animals. 

E. When you travel round the world collecting animals you also, of necessity, collect human beings. 

A EACBD 

B ABDCE 

C ECBDA 

D ACBDE 

Answer: A 

Explanation: 

BD is a link. This is because B talks about “the unpleasant human animal” that the author has met and D says that such types are fortunately rare. Similarly, AC is a link. A says that the author is intolerant of human shortcomings, but that he has been lucky in meeting mostly charming people. C explains why this might be the case. E is the best opening sentence. The order of sentences is, therefore, EACBD. 

Q. 140 Choose the most logical order of sentences among the given choices to construct a coherent paragraph. 

A.To avoid this, the QWERTY layout put the keys most likely to be hit in rapid succession on opposite sides. This made the keyboard slow, the story goes, but that was the idea. 

B. A different layout, which had been patented by August Dvorak in 1936, was shown to be much faster. 

C. The QWERTY design (patented by Christopher Sholes in 1868 and sold to Remington in 1873) aimed to solve a mechanical problem of early typewriters. 

D.Yet the Dvorak layout has never been widely adopted, even though (with electric typewriters and then PCs) the anti-jamming rational for QWERTY has been defunct for years. 

E. When certain combinations of keys were struck quickly, the type bars often jammed. 

A BDACE 

B CEABD 

C BCDEA 

D CAEBD 

Answer: B 

Explanation: 

C is the best opening sentence. It says that the Qwerty keyboard was designed for solving a specific problem. E follows and talks about what that problem is. This is followed by A which talks about how the Qwerty design solved this problem. It also talks about the shortcoming of the Qwerty design. This is followed by B which talks about a different design which tried to solve the same problem. 

The last sentence is D, which says the second design was never widely adopted even though, with the advent of electrical typewriters and PCs, the original problem of the typrewriters ceased to be a problem anymore. 

Q. 141 Choose the option in which the usage of the word ‘Bundle’ is Incorrect or Inappropriate. 

A newborn baby was a bundle of joy for the family. 

B Mobile operators are offering a bundle of additional benefits. 

C He made a bundle in the share market. 

D It was sheer luck that brought a bundle of boy-scouts to where I was lying wounded. 

Answer: D 

Explanation: 

Option 1 is correct as “bundle of joy” refers to the baby. Option 2 is correct as bundle refers to “a collection of benefits” wrapped together. Option 3 is correct as “made a bundle” means “earning a lot of money.” Option 4 is incorrect usage. 

Q. 142 Choose the option in which the usage of the word ‘Distinct’ is Incorrect or Inappropriate. 

A He is distinct about what is right and what is wrong. 

B Mars became distinct on the horizon in the month of August. 

C The distinct strains of Ravi’s violin could be heard above the general din. 

D Ghoshbabu’s is a distinct case of water rising above its own level. 

Answer: A 

Explanation: 

Option a is incorrect as the correct usage would be “He is distinctive about what is right and what is wrong” Q. 143 

Choose the option in which the usage of the word ‘Implication’ is Incorrect or Inappropriate. 

A Everyone appreciated the headmaster’s implication in raising flood relief in the village. B This letter will lead to the implication of several industrialists in the market scam. 

C Several members of the audience missed the implication of the minister’s promise. 

D Death, by implication, is the only solution the poem offers the reader. 

Answer: A 

Explanation: 

Option a is incorrect as the correct usage would be “Everyone appreciated the headmaster’s efforts.” “Implication” is the incorrect usage here.  

Q. 144 Choose the option in which the usage of the word ‘Host’ is Incorrect or Inappropriate. 

A If you host the party, who will foot the bill? 

B Kerala’s forests are host to a range of snakes 

C Ranchi will play the host to the next national film festival. 

D A virus has infected the host computer. 

Answer: C 

Explanation: 

The correct usage would be “Ranchi will play host” 

Q. 145 Choose the option in which the usage of the word ‘Sort’ is Incorrect or Inappropriate. 

A What sort of cheese do you use in pizza? 

B Farmers of all sort attended the rally. 

C They serve tea of a sort on these trains. 

D Let’s sort these boys into four groups. 

Answer: B 

Explanation: 

The correct usage would be “Farmers of all sorts” 

Q. 146 Fill in the Blanks: 

The British retailer, M&S, today formally ______ defeat in its attempt to ______ King’s, its US subsidiary, since no potential purchasers were ready to cough up the necessary cash. 

A admitted, acquire 

B conceded, offload 

C announced, dispose 

D ratified, auction 

Answer: B 

Explanation: 

Option c is incorrect as we never dispose subsidiaries. The usage of dispose is flawed. Option d is incorrect as “formally ratified defeat” is incorrect usage. Option a is incorrect as “acquire” is an incorrect usage because no purchasers are required while acquiring. Option b correctly fits the sentence. 

Q. 147 Fill in the Blanks: 

Early ______ of maladjustment to college culture is ______ by the tendency to develop friendship networks outside college which mask signals of maladjustment. 

A treatment, compounded 

B detection, facilitated 

C identification, complicated 

D prevention, helped 

Answer: C 

Explanation: 

The detection of maladjustment to college culture becomes complicated when we develop friendships outside the college. Hence option c is correct. 

Q. 148 Fill in the Blanks: 

The ______ regions of Spain all have unique cultures, but the ______ views within each region make the issue of an acceptable common language of instruction an even more contentious one. 

A different, discrete 

B distinct, disparate 

C divergent, distinct 

D different, competing 

Answer: A 

Explanation: 

The answer to the first blank is “different”. There are always different regions in a country. On the other hand views can be “discrete” or “distinct.” Hence option a is the most appropriate option. 

Q. 149 Fill in the Blanks: 

A growing number of these expert professionals ______ having to train foreigners as the students end up ______ the teachers who have to then unhappily contend with no jobs at all or new jobs with drastically reduced pay packets. 

A resent, replacing 

B resist, challenging 

C welcome, assisting 

D are, supplanting 

Answer: A 

Explanation: 

The second part of the sentence says that the expert professionals will end up with no jobs. Hence replace is the correct word for the second blank and the answer is option A. “Resent” is the correct answer for the first blank. 

Q. 150 Fill in the Blanks: 

Companies that try to improve employees’ performance by ______ rewards encourage negative kinds of behavior instead of ______ a genuine interest in doing the work well. 

A giving, seeking 

B bestowing, discouraging 

C conferring, discrediting 

D withholding, fostering 

Answer: D 

Explanation: 

In order to complement the phrase “in doing the work well” the second blank must have a positive word hence eliminating options B and C. “Seeking” is a wrong usage in this sentence. Author of this sentence is trying to say that the performance is not improved by withholding the rewards. Hence the answer is option D. 

CAT Previous Year Paper 2002

CAT 2002 

Instructions 

A boy is asked to put one mango in a basket when ordered ‘One’, one orange when ordered ‘Two’, one apple when ordered ‘Three’, and is asked to take out from the basket one mango and an orange when ordered ‘Four’. 

A sequence of orders is given as: 1 2 3 3 2 1 4 2 3 1 4 2 2 3 3 1 4 1 1 3 2 3 4 

Q. 1 How many total oranges were in the basket at the end of the above sequence? 

A.

B.

C.

D.

Answer: D. 

Explanation: 

First counting only nos. 1,2 and 3. 

So, we have 6 mangoes, 6 oranges and 7 apples. 

We have 4 times 4 so finally we have 2 mangoes, 2 oranges and 7 apples, which is a total of 11 fruits. 

Q. 2 How many total fruits will be in the basket at the end of the above order sequence? 

A.

B.

C. 11 

D. 10 

Answer: C. 

Explanation: 

On counting only numbers 1,2 and 3, we have 6 mangoes, 6 oranges and 7 apples. 

We have 4 times number 4 => Finally we have 2 mangoes , 2 oranges and 7 apples. So, a total of 11 fruits. 

 

Instructions 

Directions for the next two questions: Answer the questions based on the following information. 

Each of the 11 letters A, H, I, M, O, T, U, V, W, X and Z appears same when looked at in a mirror. They are called symmetric letters. Other letters in the alphabet are asymmetric letters. 

Q. 3 How many four-letter computer passwords can be formed using only the symmetric letters (no repetition allowed)? 

A. 7,920 

B. 330 

C. 14,640 

D. 4,19,430 

Answer: A. 

Explanation: 

The number of ways in which this can be done is 11*10*9*8 = 7920 

Q. 4 How many three-letter computer passwords can be formed (no repetition allowed) with at least one symmetric letter? 

A. 990 

B. 2,730 

C. 12,870 

D. 15,600 

Answer: C. 

Explanation: 

If there are 3 symmetric letters, it can be formed in 11*10*9 ways 

If there are 2 symmetric letters, it can be formed in 11C2 * 15C1 * 3! ways 

If there is only 1 symmetric letter, the password can be formed in 15C2*11C1*3! ways Total = 990+330*15+630*11 = 12870 ways 

Instructions 

Directions for the next two questions: Answer the questions based on the following diagram.

In the following diagram, ∠ABC= 90° = ∠DCH=∠DOE=∠EHK=∠FKL=∠GLM=∠LMN  AB = BC = 2CH = 2CD = EH = FK = 2HK = 4KL = 2LM = MN 

Q. 5 The magnitude of FGO

A. 30° 

B. 45° 

C. 60° 

D. None of these 

Answer: D. 

Explanation: 

The length of FI is twice the length of IG.  

So, the sides of the triangle FIG are in the ratio 2:1: √5. 

So, angle FGO = angle FGI, which is definitely not equal to 30 or 45 or 60. 

Hence, option D. is the answer. 

Q. 6 What is the ratio of the areas of the two quadrilaterals ABCD. to DEFG? 

A. 1 : 2 

B. 2 : 1 

C. 12 : 7 

D. None of these 

Answer: C. 

Explanation: 

Area of ABCD. = Area of triangle + Area of rectangle = Let AB = x

So, area = 3/2  ∗ x2/2 =¾ * x2

Area of ODE =½  ∗ x/2 ∗ x/2 = x2/8 

Area of OEFI = x/2 ∗ x/2 = x2/4

Area of FGI = x2 /16 

Total area of DEFG = 7x2 /16 

Required ratio = 12:7 

 

Instructions 

For the following questions answer them individually 

Q. 7 How many numbers greater than 0 and less than a million can be formed with the digits 0, 7 and 8? 

A. 486 

B. 1,084 

C. 728 

D. None of these 

Answer: C. 

Explanation: 

According to given conditions, 

number of 1 digit nos. =2, 

number of 2 digit nos. =2 ∗ 3, 

number of 3 digit nos. = 2 ∗ 32 

number of 4 digit nos. =2 ∗ 33

number of 5 digit numbers. =2 ∗ 34

Number of 6 digit nos. = 2 ∗ 35

Total summation 2*(1+3+9+27+81+243) = 728 . 

Q. 8 If there are 10 positive real numbers n1 < n2 < n3… < n10 , how many triplets of these numbers (n1, n2, n3),(n2, n3, n4) can be generated such that in each triplet the first number is always less than the second number, and the second number is always less than the third number? 

A. 45 

B. 90 

C. 120 

D. 180 

Answer: C. 

Explanation: 

For any selection of three numbers, there is only one way in which they can be arranged in ascending order. 

So, the answer is 10C3 = 120

Q. 9 In triangle ABC, the internal bisector of A meets BC at D. If AB = 4, AC = 3 and A= 60° , then the length of AD is

A. 2√3

B. 12√3 /7

C. 15√3 / 8

D. 6√3/ 7

Answer: B. 

Explanation: 

By using cosine rule we can find BC = √13. By angle bisector theorem we have BA. / BD = AC / DC. Also BD + DC = √13. 

So by substitution we get we get BD. = 4* √13/7 . Now using cosine rule in triangle ABD. taking AD. = x, we get 12 3 

x2 − 4 ∗ √13 ∗ x + 16 ∗ (36/49) = 0.

Solving the equation we get x =12√3 /7. 

Q. 10 The length of the common chord of two circles of radii 15 cm and 20 cm, whose centres are 25 cm apart, is 

A. 24 cm 

B. 25 cm 

C. 15 cm 

D. 20 cm 

Answer: A. 

Explanation: 

The radii of both the circles and the line joining the centers of the two circles form a right angled triangle. So, the length of the common chord is twice the length of the altitude dropped from the vertex to the hypotenuse. 

Let the altitude be h and let it divide the hypotenuse in two parts of length x and 25-x 

So, h2 + x2 = 152 and h2 + (25 − x)2 = 202 

=>225 − x2 = 400 − x2 + 50x − 625

=> 50x = 450 => x = 9 and h = 12 

So, the length of the common chord is 24 cm.

Q. 11 If f(x) = log [(1+x)/(1−x)], then f(x) + f(y) is 

A. f(x + y)

B. f(x + y)/(1 +xy)

C. (x + y) f 1/(1 +xy)

D. f(x) +f(y) / (1 +xy)

Answer: B. 

Explanation: 

If f(x)= log [(1+x)/(1-x)] then f(y)= log [(1+y)/(1-y)] 

Also Log (A*B)= Log A + Log B 

f(x)+f(y) =log [(1+x)(1+y)/(1-x)(1-y)] solving we get log [1+(1+xy) (x+y)/ 1−(1+xy) (x+y)]

Hence option B. 

Q. 12 Four horses are tethered at four corners of a square plot of side 14 m so that the adjacent horses can just reach one another. There is a small circular pond of area 20m2 at the centre. Find the ungrazed area. 

A. 22m2

B. 42m2 

C. 842m2 

D. 168m2

Answer: A. 

Explanation: 

Length of the rope tied to each horse = 7 m. 

Total area of the portion that the horses can graze = 4*π72 /4 = 49π 

Area of the circular pond = 20 m2 

So, area left ungrazed =142 − 20 − 49πm2  = 22m2 (approx) 

Q. 13 On a straight road XY, 100 m long, five heavy stones are placed 2 m apart beginning at the end X. A worker, starting at X, has to transport all the stones to Y, by carrying only one stone at a time. The minimum distance he has to travel is 

A. 472 m 

B. 422 m 

C. 744 m 

D. 860 m 

Answer: D. 

Explanation: 

The weights are at distances of 0 m, 2 m, 4 m, 6 m and 8 m from X. 

Let us first keep all the weights at a distance of 8 m from X. This would be 8 + 6*2 + 4*2 + 2*2 = 32 m. 

Now from the point where all the weights are kept is at a distance of 92 m from Y. So total distance required = 184+184+184+184+92 = 828 m. 

So in all 860 m

Q. 14 In the figure given below, ABCD is a rectangle. The area of the isosceles right triangle ABE = 7cm2 ; EC = 3(BE). The area of ABCD (in cm2 ) is 

A. 21 cm2 

B. 28 cm2 

C. 42 cm2 

D. 56 cm2 

Answer: D. 

Explanation: 

Let AB. = BE = x 

Area of triangle ABE =x2/2 = 14; we get x = √14

So we have side BC = 4*√14 

Now area is AB*BC = 14 *4 = 56

Q. 15 The area of the triangle whose vertices are (a,a), (a + 1, a + 1) and (a + 2, a) is 

A. a3

B. 1

C. 2a

D. 21/2

Answer: B. 

Explanation: 

The length of three sides is √2,√2 and 2. 

This is a right-angled triangle. 

Hence, it’s area equals 1/2 ∗ √2 ∗ √2 = 1 

So, the correct answer is b) 

Q. 16 Instead of walking along two adjacent sides of a rectangular field, a boy took a short cut along the diagonal and saved a distance equal to half the longer side. Then the ratio of the shorter side to the longer side is 

A. 1/2 

B. 2/3 

C. 1/4 

D. 3/4 

Answer: D. 

Explanation: 

Let x be the shorter side and y be the longer one. The shortcut route would be of length √(x2 + y2). According to given condition we know that (x+y)- √(x2 + y2)= y/2 . Solving, we get 1=(x/y)+(1/4) 

=> x : y = 3 : 4. Hence option D. is the correct answer. 

Q. 17 Only a single rail track exists between stations A. and B. on a railway line. One hour after the northbound super fast train N leaves station A. for station B, a south-bound passenger train S reaches station A. from station B. The speed of the super fast train is twice that of a normal express train E, while the speed of a 

passenger train S is half that of E. On a particular day, N leaves for B. from A, 20 min behind the normal schedule. In order to maintain the schedule, both N and S increased their speeds. If the super fast train doubles its speed, what should be the ratio (approximately) of the speeds of the passenger train to that of the super fast train so that the passenger train S reaches exactly at the scheduled time at A. on that day? 

A. 1 : 3 

B. 1 : 4 

C. 1 : 5 

D. 1 : 6 

Answer: D. 

Explanation: 

Let the speed of an express train be 4x, normal train be 2x and passenger train be x. 

Let the distance between the 2 stations be D. 

Since there is only 1 railway track, train N must reach station B. before train S leaves. 

Therefore, D/4x + D/x = 60 

5D/4x = 60 

D/x = 48 

Train N leaves 20 minutes late. Therefore, the 2 trains must have covered the distance within 40 minutes on this particular day. 

Train N doubles its speed. Therefore, speed of train N will be 8x. Let the new speed of the passenger train be y. 

D/8x + D/y = 40 

48/8 + D/y = 40 

D/y = 34. 

Speed of super fast train = D/8x = 6 

Speed of passenger train = D/y = 34 

Ratio of the speeds = 6/34 = 3/17. 

The ratio is approximately equal to 1:6. Therefore, option D. is the right answer. 

Q. 18 On a 20 km tunnel, connecting two cities A and B, there are three gutters (1, 2 and 3). The distance between gutters 1 and 2 is half the distance between gutters 2 and 3. The distance from city A to its nearest gutter, gutter 1, is equal to the distance of city B from gutter 3. On a particular day, the hospital in city A receives information that an accident has happened at gutter 3. The victim can be saved only if an operation is started within 40 min. An ambulance started from city A at 30 km/hr and crossed gutter 1 after 5 min. If the driver had doubled the speed after that, what is the maximum amount of time would the doctor get to attend the patient at the hospital. Assume 1 min is elapsed for taking the patient into and out of the ambulance? 

A. 4 min 

B. 2.5 min 

C. 1.5 min 

D. The patient died before reaching the hospital 

Answer: C. 

Explanation: 

Let the distance between gutter 1 and A. be x and between gutter 1 and 2 be y. 

Hence, x + y + 2y + x = 20 => 2x+3y=20 

Also x = 30kmph * 5/60 = 2.5km 

Hence, y = 5km 

After the ambulance doubles its speed it goes at 60kmph i.e. 1km per min. Hence, time taken for the rest of the journey = 15*2 + 2.5 = 32.5 

It takes 1 min to load and unload the patient. 

Hence, total time = 5 + 32.5 + 1 = 38.5 mins 

So, the doctor would get 1.5 min to attend to the patient. 

Q. 19 Number S is obtained by squaring the sum of digits of a two-digit number D. If difference between S and D. is 27, then the two-digit number D. is 

A. 24 

B. 54 

C. 34 

D. 45 

Answer: B. 

Explanation: 

Consider the options: 

24: (Square of sum of digits – the number) = 36 – 24 = 12 

54: (Square of sum of digits – the number) = 81 – 54 = 27 

34: (Square of sum of digits – the number) = 49 – 34 = 15 

45: (Square of sum of digits – the number) = 81 – 45 = 36 

So, option b) is the correct answer. 

Q. 20 The nth element of a series is represented as Xn = (−1)nXn−1 

If X0 = x and x > 0, then which of the following is always true? 

A. Xn  is positive if n is even 

B. Xn  is positive if n is odd 

C. Xn  is negative if n is even 

D. None of these 

Answer: D. 

Explanation: 

Let x = 1, so, X0= 1 

X1 = -1 

X2 = -1 

X3= 1 

X4 = 1 

X5 = -1 

X6 = -1 

So,Xn  need not be positive when n is even, Xn  need not be positive when n is odd, Xn  need not be negative when n is even. So, none of the first three options are correct. 

Q. 21 If x, y and z are real numbers such that x + y + z = 5 and xy + yz + zx = 3, what is the largest value that x can have? 

A. 5/3

B. √19 

C. 13/3 

D. None of these 

Answer: C. 

Explanation: 

The given equations are x + y + z = 5 — (1) , xy + yz + zx = 3 — (2) 

xy + yz + zx = 3 

x(y + z) + yz = 3 

=> x ( 5 -x ) +y ( 5 – x – y) = 3 

=> −y2 y(5 − x) − x2 + 5x = 3 

=> −y2 + y(x − 5) + (x2 − 5x + 3) = 0 

The above equation should have real roots for y, => Determinant >= 0 

=>(x − 5)2 − 4(x2 − 5x + 3) ≥ 0 

=>3x2 − 10x − 13 ≤ 0 

=>−1 ≤ x ≤ 13/3 

Hence maximum value x can take is 13/3, and the corresponding values for y,z are ⅓ , ⅓  

Q. 22 Neeraj has agreed to mow a lawn, which is a 20 m × 40 m rectangle. He mows it with 1 m wide strip. If Neeraj starts at one corner and mows around the lawn toward the centre, about how many times would he go round before he has mowed half the lawn? (Round off the answer to two decimal digits) 

A. 2.5 

B. 3.5 

C. 3.75 

D. None of these 

Answer: D. 

Explanation: 

To mow half of lawn is to mow 400 squares of 1m width each. Neeraj mows 40+18+40+18 = 116 squares of 1m width each in 1st round, in 2nd round he mows 38+16+38+16 = 108 squares and in 3rd round, he mows 36+14+36+14 = 100 squares. So, in total, he mows 324 squares; but he needs 400-324 = 76 more. 

If he covers the 4th round completely, he mows 92 squares, to cover only 76, he needs to cover 76/92 = 0.826 rounds. 

So in total 3.826≈3.83 rounds are required. 

Q. 23 The owner of a local jewellery store hired three watchmen to guard his diamonds, but a thief still got in and stole some diamonds. On the way out, the thief met each watchman, one at a time. To each he gave 1/2 of the diamonds he had then, and 2 more besides. He escaped with one diamond. How many did he steal originally? 

A. 40 

B. 36 

C. 25 

D. None of these 

Answer: B. 

Explanation: 

Suppose the thief stole ‘x’ diamonds.  

After giving the share to the first watchman, the thief has (x/2)-2 diamonds. 

After giving to the second watchman, the thief has (x/4)-3 diamonds. 

After giving to the third watchman, the thief has (x/8)-(7/2) diamonds. 

This is equal to 1. So, (x/8) – 7/2 = 1 

Solving this equation, we get x = 36 

Q. 24 Mayank, Mirza, Little and Jaspal bought a motorbike for $60. Mayank paid one-half of the sum of the amounts paid by the other boys. Mirza paid one-third of the sum of the amounts paid by the other boys. Little paid one-fourth of the sum of the amounts paid by the other boys. How much did Jaspal have to pay? 

A. $15 

B. $13 

C. $17 

D. None of these 

Answer: B. 

Explanation: 

Let the amount paid by Mayank be x. So, amount paid by the other three = 2x 

=> Total bill = x+2x = 3x = 60 => x = 20. So, Mayank paid 20 

Similarly, amount paid by Mirza + 3*Amount paid by Mirza = 60 

=> Amount paid by Mirza = 15 

Amount paid by Little + 4*Amount paid by Little = 60 

=> Amount paid by Little = 12 

So, amount paid by Jaspal = 60 – (20+15+12) = 60 – 47 = $13 

Q. 25 A rich merchant had collected many gold coins. He did not want anybody to know about him. One day, his wife asked, ” How many gold coins do we have?” After a brief pause, he replied, “Well! if I divide the coins into two unequal numbers, then 48 times the difference between the two numbers equals the difference between the squares of the two numbers.” The wife looked puzzled. Can you help the merchant’s wife by finding out how many gold coins the merchant has? 

A. 96 

B. 53 

C. 43 

D. None of these 

Answer: D. 

Explanation: 

Let x = y+z such that z > y. 

We know that 48 ∗ (z y)2 = z2 y2 

Solving the above equation, we get z + y = 48 

So, option d) is the correct answer. 

Q. 26 Shyam visited Ram during his brief vacation. In the mornings they both would go for yoga. In the evenings they would play tennis. To have more fun, they indulge only in one activity per day, i.e. either they went for yoga or played tennis each day. There were days when they were lazy and stayed home all day long. There were 24 mornings when they did nothing, 14 evenings when they stayed at home, and a total of 22 days when they did yoga or played tennis. For how many days Shyam stayed with Ram? 

A. 32 

B. 24 

C. 30 

D. None of these 

Answer: C. 

Explanation: 

Let the number of lazy days=x 

They played tennis for 24-x days 

They did yoga for 14-x days 

So, they did yoga or played tennis for 24-x+14-x = 38-2x days 

38-2x=22 

x = 8 

Number of lazy days=8 

Number of days where they played yoga or tennis = 22 

Number of Shyam stayed with Ram = 22+8=30 

Q. 27 Let S denotes the infinite sum 2 + 5x + 9x2 + 14x3 + 20x4 + … xn−1 , where |x| < 1 and the coefficient of is n( n + 3 )/2 , ( n = 1, 2 , . . . ) . Then S equals: 

A. (2 − x)/(1 − x)3

B. (2 − x)/(1 + x)3

C. (2 + x)/(1 − x)3

D. (2 + x)/(1 + x)3

Answer: A. 

Explanation: 

Let S = 2 + 5x + 9x2 +…. 

S x = 2x + 5x2 + 9x3 + … 

S(1 − x) = 2 + 3x + 4x2 + … 

S(1 − x) ∗ x = 2x + 3x2 + 4x3 + … 

S(1 − x)(1 − x) = 2 + x + x2 + x3 + … = 2 + x/(1 − x

So, S = [2(1 − x) + x]/(1 − x)3 => S = (2 − x)/(1 − x)3 

Q. 28 If x2 + 5y2 + z2 = 2y(2x + z), then which of the following statements is(are) necessarily true? A. x = 2y B. x = 2z C. 2x = z 

A. Only A 

B. B and C 

C. A and B 

D. None of these 

Answer: C. 

Explanation: 

The equation is not satisfied for only x = 2y. 

Using statements B. and C, i.e., x = 2z and 2x = z, we see that the equation is not satisfied. 

Using statements A. and B, i.e., x = 2y and x = 2z, i.e., z = y = x/2, the equation is satisfied. 

Option c) is the correct answer. 

Q. 29 Amol was asked to calculate the arithmetic mean of 10 positive integers, each of which had 2 digits. By mistake, he interchanged the 2 digits, say a and b, in one of these 10 integers. As a result, his answer for the arithmetic mean was 1.8 more than what it should have been. Then |b – a| equals 

A.

B.

C.

D. None of these 

Answer: B. 

Explanation: 

Let the actual average be n. So, the new average is n + 1.8 

Actual total = 10n 

New total = 10n + 18 

Let the number which was miswritten = ab(a is the tenth’s digit and b is the units digit) = 10a+b and reversed number ba = 10b+a 

So, 10b + a – (10a + b) = 18 

=> 9(b-a) = 18 

=> b-a = 2 

Q. 30 A car rental agency has the following terms. If a car is rented for 5 hr or less, then, the charge is Rs. 60 per hour or Rs. 12 per kilometre whichever is more. On the other hand, if the car is rented for more than 5 hr, the charge is Rs. 50 per hour or Rs. 7.50 per kilometre whichever is more. Akil rented a car from this agency, drove it for 30 km and ended up playing Rs. 300. For how many hours did he rent the car? 

A. 4 hr 

B. 5 hr 

C. 6 hr 

D. None of these 

Answer: C. 

Explanation: 

Suppose Akil drove the car for less than 5 hrs. In this case, by distance basis, Rs 360 should be charged. This is not the case. 

So he dove for more than 5 hrs. Cost comes more using time basis; which is Rs 300, i.e. he used the car for 6 hours. 

Q. 31 A child was asked to add the first few natural numbers (i.e. 1 + 2 + 3 + …) so long his patience permitted. As he stopped, he gave the sum as 575. When the teacher declared the result wrong, the child discovered he had missed one number in the sequence during addition. The number he missed was 

A. less than 10 

B. 10 

C. 15 

D. more than 15 

Answer: D. 

Explanation: 

If the child adds all the numbers from 1 to 34, the sum of the numbers would be 1+2+3+…+34 = 34*35/2 = 595 Since the child got the sum as 575, he would have missed the number 20. 

Q. 32 Suppose for any real number x, [x] denotes the greatest integer less than or equal to x. Let L(x, y) = [x] + [y] + [x + y] and R(x, y) = [2x] + [2y]. Then it is impossible to find any two positive real numbers x and y for which 

A. L(x, y) = R(x, y) 

B. L(x, y) ≠ R(x, y) 

C. L(x, y) < R(x, y) 

D. L(x, y) > R(x, y) 

Answer: D. 

Explanation: 

Consider different values of x and y: 

x = -1.5 and y = -1.5; x = 1.5 and y = -1.5; x = -1.5 and y = 1.5; x = 1.5 and y = 1.5. 

For these possibilities, options A,B. and C. gets satisfied , but it is impossible to find any two positive real numbers x and y for which L(x, y) > R(x, y). 

Q. 33 Ten straight lines, no two of which are parallel and no three of which pass through any common point, are drawn on a plane. The total number of regions (including finite and infinite regions) into which the plane would be divided by the lines is 

A. 56 

B. 255 

C. 1024 

D. not unique 

Answer: A. 

Explanation: 

If there are ‘m’ non-parallel lines, then the maximum number of regions into which the plane is divided is given by [m(m+1)/2]+1 

In this case, ‘m’ = 10 

So, the number of regions into which the plane is divided is (10*11/2) + 1 = 56 

Q. 34 When 2256 is divided by 17, the remainder would be 

A.

B. 16 

C. 14 

D. None of these 

Answer: A. 

Explanation: 

24 = 16 = −1(mod 17) 

So, 2256 = (−1)64 (mod 17) 

= 1 (mod 17) 

Hence, the answer is 1. Option a). 

Q. 35 The number of real roots of the equation A2/x + B2/(x − 1) = 1 , where A and B are real numbers not equal to zero simultaneously, is 

A.

B.

C.

D. Cannot be determined 

Answer: D. 

Explanation: 

(x − 1) + B. ∗ 

The given equation can be written as : A2 ∗ (x − 1) +B2 ∗ x = x2 − x

=> x2 + x(−1 − A2 B2) + A2 = 0

Discriminant of the equation = (−1 − A2 B2)2 − 4A2

= A4 + B4 + 1 − 2A2 + 2B2 + 2A2B2 

= A4 + B4 + 1 − 2A2 − 2B2 + 2A2B2 + 4B2 

= (A2 + B2 − 1)2 + 4B2 

=> 0, 0 when B =0 and A =1 

Hence, the number of roots can be 1 or 2. 

Option d) is the correct answer. 

Q. 36 At a bookstore, ‘MODERN BOOK STORE’ is flashed using neon lights. The words are individually flashed at the intervals of 2.5 s, 4.25 s and 5.125 s respectively, and each word is put off after a second. The least time after which the full name of the bookstore can be read again for a full second is 

A. 49.5 s 

B. 73.5 s 

C. 1744.5 s 

D. 855 s 

Answer: B. 

Explanation: 

In this problem, the lights are flashed at the intervals 2.5, 4.25 and 5.125 seconds and put off after one second each. The total duration of intervals of these lights are (2.5+1) = 3.5 s, (4.25+1) = 5.25 s and (5.125+1) = 6.125 s. 

We have to find the minimum duration. It would be the LCM of these three numbers. 

Since each word is put after a second. So LCM [(5/2 +1)(17/4+1)(41/8+1)] = LCM of numerator / HCF of denominator = 49*3/2 = 73.5. 

Q. 37 Three pieces of cakes of weights 4.5 lb, 6.75 lb and 7.2 lb respectively are to be divided into parts of equal weight. Further, each part must be as heavy as possible. If one such part is served to each guest, then what is the maximum number of guests that could be entertained? 

A. 54 

B. 72 

C. 20 

D. None of these 

Answer: D. 

Explanation: 

HCF of [(9/2), (27/4), (36/5)] = HCF of numerators / LCM of denominators = 9/20 

Total weight = 18.45 lb 

So no. of parts = 18.45 / (9/20) = 18.45*20/9 = 41 

Hence option d) is the correct answer. 

Q. 38 After the division of a number successively by 3, 4 and 7, the remainders obtained are 2, 1 and 4 respectively. What will be the remainder if 84 divides the same number? 

A. 80 

B. 75 

C. 41 

D. 53 

Answer: D. 

Explanation: 

Since after division of a number successively by 3, 4 and 7, the remainders obtained are 2, 1 and 4 respectively, the number is of form ((((4*4)+1)*3)+2)k = 53K 

Let k = 1; the number becomes 53 

If it is divided by 84, the remainder is 53. 

Option d) is the correct answer. 

Alternative Solution. 

Consider only for 3 and 4 and the remainders are 2 and 1 respectively. 

So 5 is the first number to satisfy both the conditions. The number will be of the form 12k+5. Put different integral values of k to find whether it will leave remainder 5 when divided by 7. So the first number to satisfy such condition is 48×4+5= 53 

Q. 39 Six people are playing a card game sitting around a circular table. Suresh is facing Raghubir who is to the left of Ajay and to the right of Pramod. Ajay is to the left of Dhiraj. Yogendra is to the left of Pramod. If Dhiraj exchanges his seat with Yogendra and Pramod exchanges with Raghubir, who will be sitting to the left of Dhiraj? 

A. Yogendra 

B. Raghubir 

C. Suresh 

D. Ajay 

Answer: C. 

Explanation: 

The correct original circular arrangement sequence in clockwise manner is Suresh, Dhiraj, Ajay, Raghuveer, Pramod, Yogendra. 

So after the changes, Suresh is to the left of Dhiraj. 

Q. 40 A train approaches a tunnel AB. Inside the tunnel is a cat located at a point that is 3/8 of the distance AB measured from the entrance A. When the train whistles the cat runs. If the cat moves to the entrance of the tunnel A, the train catches the cat exactly at the entrance. If the cat moves to the exit B, the train catches the cat at exactly the exit. What is the ratio of speed of train and cat ? 

A. 3 : 1 

B. 4 : 1 

C. 5 : 1 

D. None of these 

Answer: B. 

Explanation: 

Let the length of the tunnel be x and distance of the train to entrance A. be y. Let the speeds of train and cat be t and c respectively. 

Hence, when the cat runs 3x/8, the train covers y. 

=> (3x/8)/c = y/t — (1) 

When the cat runs 5x/8 to the other end, the train covers x+y 

=>(5x/8)/c = (x+y)/t —(2) 

Taking ratio of (1) to (2) 

3/5 = y/(x+y) => 3x = 2y —(3) 

Substituting (3) in (1) 

(2y/8)/c = y/t 

=> t = 4c 

Hence the ratio t:c is 4:1. 

Q. 41 A piece of string is 40 cm long. It is cut into three pieces. The longest piece is three times as long as the middle-sized and the shortest piece is 23 cm shorter than the longest piece. Find the length of the shortest piece. 

A. 27 

B.

C.

D.

Answer: C. 

Explanation: 

Let the longest piece be x 

Shortest piece = x – 23 

Middle-sized piece = x/3 

So, x + x – 23 + x/3 = 40 => 7x/3 = 63 => x = 27 

Shortest piece = 27 – 23 = 4 

Q. 42 Three travellers are sitting around a fire, and are about to eat a meal. One of them has 5 small loaves of bread, the second has 3 small loaves of bread. The third has no food, but has 8 coins. He offers to pay for some bread. They agree to share the 8 loaves equally among the three travellers, and the third traveller will pay 8 coins for his share of the 8 loaves. All loaves were the same size. The second traveller (who had 3 loaves) suggests that he will be paid 3 coins, and that the first traveller be paid 5 coins. The first traveller says that he should get more than 5 coins. How much should the first traveller get? 

A.

B.

C.

D. None of these 

Answer: B. 

Explanation: 

Suppose A, B. and C. have 5 pieces of bread, 3 pieces of bread and 8 coins respectively. Since in total there are 8 pieces of bread, each one should get around 2.66 bread. So A. must give 2.33 part of his bread to C. and B. must give 0.33. Distributing the amount in the same ratio of bread contribution, A. must get 7 coins and B. must get 1 coin. 

Q. 43 In the following figure, ACB. is a right-angled triangle. AD. is the altitude. Circles are inscribed within the triangle ACD. and triangle BCD. P and Q are the centers of the circles. The distance PQ is 

A. 7 m 

B. 4.5 m 

C. 10.5 m 

D. 6 m 

Answer: A. 

Explanation: 

By Pythagoras theorem we get BC. = 25 . Let BD. = x;Triangle ABD. is similar to triangle CBA. => AD/15 = x/20 and also triangle ADC. is similar to triangle ACB=> AD/20 = (25-x)/15. From the 2 equations, we get x = 9 and DC. = 16 

We know that AREA. = (semi perimeter ) * inradius 

For triangle ABD, Area = 1/2 x BD. X AD. = 1/2 x 12 x 9 = 54 and semi perimeter = (15 + 9 + 12)/2 = 18. On using the above equation we get, inradius, r = 3. 

Similarly for triangle ADC. we get inradius R = 4 . 

PQ = R + r = 7 cm 

Q. 44 If u, v, w and m are natural numbers such that um + vm = wm, then which one of the following is true? 

A. m >= min(u, v, w) 

B. m >= max(u, v, w) 

C. m < min(u, v, w) 

D. None of these 

Answer: D. 

Explanation: 

Substitute value of u = v = 2, w = 4 and m = 1. Here the condition holds and options A. and B. are false. Hence, we can eliminate options A. and B. 

Substitute u = v = 1, w=2 and m= 1. Here m=min(u, v, w). Hence, option C. also does not hold. Hence, we can eliminate option C. 

Option d) is the correct answer. 

Q. 45 In how many ways is it possible to choose a white square and a black square on a chessboard so that the squares must not lie in the same row or column? 

A. 56 

B. 896 

C. 60 

D. 768 

Answer: D. 

Explanation: 

First a black square can be selected in 32 ways. Out of remaining rows and columns, 24 white squares remain. 1 white square can them be chosen in 24 ways. So total no. of ways of selection is 32*24 = 768. 

Q. 46  76n − 66n, where n is an integer > 0, is divisible by 

A. 13 

B. 127 

C. 559 

D. All of these 

Answer: D. 

Explanation: 

Consider n=1 we have 76 − 66 which is =(73 + 63)(73 − 6 )3 = 13 * 127 * 43 which is divisible by all the 3 options. Option d) is the correct answer. 

Q. 47 If pqr = 1, the value of the expression 1/(1 + p + q−1) + 1/(1 + q + r−1) + 1/(1 + r + p ) −1

A. p+q+r 

B. 1/(p+q+r) 

C.

D. p−1 + q−1 + r−1 

Answer: C. 

Explanation: 

Let p = q = r = 1 

So, the value of the expression becomes 1/3 + 1/3 + 1/3 = 1 

If we substitute these values, options a), b) and d) do not satisfy. 

Option c) is the answer. 

Q. 48 It takes six technicians a total of 10 hr to build a new server from Direct Computer, with each working at the same rate. If six technicians start to build the server at 11 am, and one technician per hour is added beginning at 5 pm, at what time will the server be completed? 

A. 6.40 pm 

B. 7 pm 

C. 7.20 pm 

D. 8 pm 

Answer: D. 

Explanation: 

Let the work done by each technician in one hour be 1 unit. 

Therefore, total work to be done = 60 units. 

From 11 AM to 5 PM, work done = 6*6 = 36 units. 

Work remaining = 60 – 36 = 24 units. 

Work done in the next 3 hours = 7 units + 8 units + 9 units = 24 units. 

Therefore, the work gets done by 8 PM. 

Q. 49 Davji Shop sells samosas in boxes of different sizes. The samosas are priced at Rs. 2 per samosa up to 200 samosas. For every additional 20 samosas, the price of the whole lot goes down by 10 paise per samosa. What should be the maximum size of the box that would maximise the revenue? 

A. 240 

B. 300 

C. 400 

D. None of these 

Answer: B. 

Explanation: 

Let the optimum number of samosas be 200+20n 

So, price of each samosa = (2-0.1*n) 

Total price of all samosas = (2-0.1*n)*(200+20n) =400 − 20n + 40n − 2n2  = 400 + 20n − 2n2 

This quadratic equation attains a maximum at n = -20/2*(-2) = 5 

So, the number of samosas to get the maximum revenue = 200 + 20*5 = 300 

Q. 50 Three small pumps and a large pump are filling a tank. Each of the three small pumps works at 2/3 the rate of the large pump. If all four pumps work at the same time, they should fill the tank in what fraction of the time that it would have taken the large pump alone? 

A. 4/7 

B. 1/3 

C. 2/3 

D. 3/4 

Answer: B. 

Explanation: 

Let the work done by the big pump in one hour be 3 units. 

Therefore, work done by each of the small pumps in one hour = 2 units. 

Let the total work to be done in filling the tank be 9 units. 

Therefore, time taken by the big pump if it operates alone = 9/3 = 3 hours. 

If all the pumps operate together, the work done in one hour = 3 + 2*3 = 9 units. 

Together, all of them can fill the tank in 1 hour. 

Required ratio = 1/3 

Data Interpretation 

Instructions 

Four students — Ashish, Dhanraj, Felix and Sameer sat for the Common Entrance Exam for Management (CEEM). One student got admission offers from three NIMs (National Institutes of Management), another from two NIMs, the third from one NIM, while the fourth got none. Below are some of the facts about who got admission offers from how many NIMs and what is their educational background. 

I. The one who is an engineer didn’t get as many admissions as Ashish. 

II. The one who got offer for admissions in two NIMs isn’t Dhanraj nor is he a chartered accountant. III. Sameer is an economist. 

IV. Dhanraj isn’t an engineer and received more admission offers than Ashish. 

V. The doctor got the most number of admission offers. 

Q. 51 Which one of the following statements is necessarily true? 

A. Ashish is a chartered accountant and got offer for admission in three NIMs. 

B. Dhanraj is a doctor and got admission offer in one NIM. 

C. Sameer is an economist who got admission offers in two NIMs. 

D. Felix who is not an engineer did not get any offer for admission. 

Answer: C. 

Explanation: 

According to given information, we finally know that ashish is a CA. and will get 1 or no offers. dhanraj is doctor and gets 3 offers felix is an engineer and can get 2 offers. 

Sameer is an economist and can get 2 offers.

 

Instructions 

For the following questions answer them individually 

Q. 52 Five boys went to a store to buy sweets. One boy had Rs. 40. Another boy had Rs. 30. Two other boys had Rs. 20 each. The remaining boy had Rs. 10. Below are some more facts about the initial and final cash positions. 

I. Alam started with more than Jugraj. 

II. Sandeep spent Rs. 1.50 more than Daljeet. 

III. Ganesh started with more money than just only one other person. 

IV. Daljeet started with 2/3 of what Sandeep started with. 

V. Alam spent the most, but did not end with the least. 

VI. Jugraj spent the least and ended with more than Alam or Daljeet. 

VII.Ganesh spent Rs.3.50. 

VIII. Alam spent 10 times more than what Ganesh did. 

In the choices given below, all statements except one are false. Which one of the following statements can be true? 

A. Alam started with Rs. 40 and ended with Rs. 9.50 

B. Sandeep started with Rs. 30 and ended with Re. 1 

C. Ganesh started with Rs. 20 and ended with Rs. 4 

D. Jugraj started with Rs. 10 and ended with Rs. 7 

Answer: D. 

Explanation: 

According to given condition we know that Alam started with 40 spent 35 and saved 5 , Daljeet started with 20 , ganesh started with 20 and saved 16.5 , Jugraj started with 10 and spent less than 3 rs , sandeep started with 30 and spent more than 1.5 rs. So among the options only option D. satisfies given requirements. 

Q. 53 In a hospital there were 200 diabetes, 150 hyperglycaemia and 150 gastro-enteritis patients. Of these, 80 patients were treated for both diabetices and hyperglycaemia. Sixty patients were treated for gastro enteritis and hyperglycaemia, while 70 were treated for diabetes and gastroenteritis. Some of these patients have all the three diseases. Dr. Dennis treats patients with only gastro-enteritis. Dr. Paul is a generalist. Therefore, he can treat patients with multiple diseases. Patients always prefer a specialist for their disease. If Dr. Dennis had 80 patients, then the other three doctors can be arranged in terms of the number of patients treated as: 

A. Paul > Gerard > Hormis 

B. Paul > Hormis > Gerard 

C. Gerard > Paul > Hormis 

D. Cannot be determined 

Answer: D. 

Explanation: 

We don’t know out of 2 other doctors which doctor is specialist in which disease. 

So it is not possible to find out the exact order. 

Q. 54 Three children won the prizes in the Bournvita Quiz contest. They are from the schools: Loyola, Convent and Little Flowers, which are located at different cities. Below are some of the facts about the schools, the children and the city they are from. 

I. One of the children is Bipin. 

II. Loyola School’s contestant did not come first. 

III. Little Flower’s contestant was named Riaz. 

IV. Convent School is not in Hyderabad. 

V. The contestant from Pune is not from Loyola School. 

VI. The contestant from Bangalore did not come first. 

VII. Convent School’s contestant’s name is not Balbir. 

VIII. The contestant from Pune came in third. 

Which of the following statements is true? 

A. 1st prize: Riaz (Little Flowers), 2nd prize: Bipin (Convent), 3rd prize: Balbir (Loyola) 

B. 1st prize: Bipin (Convent), 2nd prize: Riaz (Little Flowers), 3rd prize: Balbir (Loyola) 

C. 1st prize: Riaz (Little Flowers), 2nd prize: Balbir (Loyola), 3rd prize: Bipin (Convent) 

D. 1st prize: Bipin (Convent), 2nd prize: Balbir (Loyola), 3rd prize: Riaz (Litttle Flowers) 

Answer: C. 

Explanation: 

From given condition we can find out that Balbir is from loyola and came 2nd , Bipin is from banglore and came 3rd , Riaz is from little flower and came 1st . 

Q. 55 Two boys are playing on a ground. Both the boys are less than 10 years old. Age of the younger boy is equal to the cube root of the product of the age of the two boys. If we place the digit representing the age of the younger boy to the left of the digit representing the age of the elder boy, we get the age of father of the younger boy. Similarly, if we place the digit representing the age of the elder boy to the left of the digit representing the age of the younger boy and divide the figure by 2, we get the age of the mother of the younger boy. The mother of the younger boy is younger to his father by 3 years. Then, what is the age of the younger boy? 

A.

B.

C.

D. None of these 

Answer: C. 

Explanation: 

Let x and y be the age of the older and younger boy respectively(both single digit). According to given condition we know that y2 =

Also Father’s age = 10y + x and Mother’s age = (10x+y)/2. 

Only value which satisfies the above equations is x=4 and y=2 . 

Q. 56 Flights A and B are scheduled from an airport within the next one hour. All the booked passengers of the two flights are waiting in the boarding hall after check-in. The hall has a seating capacity of 200, out of which 10% remained vacant. 40% of the waiting passengers are ladies. When boarding announcement came, passengers of flight A. left the hall and boarded the flight. Seating capacity of each flight is two third of the passengers who waited in the waiting hall for both the flights put together. Half the passengers who boarded flight A are women. After boarding for flight A, 60% of the waiting hall seats became empty. For every twenty of those who are still waiting in the hall for flight B, there is one air hostess in flight A What is the ratio of empty seats in flight B to the number of air hostesses in flight A? 

A. 10 : 1 

B. 5 : 1 

C. 20 : 1 

D. 1 : 1 

Answer: A. 

Explanation: 

Out of 200 of the seating capacity, 180 seats are filled out of which 108 are males and 72 are females. Remaining 20 seats are vacant. According to given condition seating capacity in both the planes is 120 . Considering flight A – we can find that 100 passenger in waiting hall will be taking fight A . So 80 people remain in in the waiting hall who will be taking flight B . Now for every 20 people taking flight B we have a air hostess in flight A . So in total there are 4 air hostess in flight A Flight B having 120 as seating capacity, 40 remain vacant. So required ratio 40:4 = 10:1 . 

 

Instructions 

A. country has the following types of traffic signals. 

3 red lights = stop 

2 red lights = turn left 

1 red light = turn right 

3 green lights = go at 100 km/hr speed 

2 green lights = go at 40 km/hr speed 

1 green light = go at 20 km/hr speed 

A. motorist starts at a point on a road and follows all traffic signals. His car is heading towards the north. He encounters the following signals (the time mentioned in each case below is applicable after crossing the previous signal). 

Starting point – 1 green light 

After half an hour, 1st signal – 2 red and 2 green lights 

After 15 min, 2nd signal – 1 red light 

After half an hour, 3rd signal – 1 red and 3 green lights 

After 24 min, 4th signal – 2 red and 2 green lights 

After 15 min, 5th signal – 3 red lights 

Q. 57 The total distance travelled by the motorist from the starting point till the last signal is 

A. 90 km 

B. 100 km 

C. 120 km 

D. None of these 

Answer: A. 

Explanation: 

The distance traveled by the car in particular direction is as follows : First 10 km in north then 10 km in west then 20 km north then 40 km east then again 10 km north. 

So in all total distance traveled is 10+10+20+40+10 = 90 kms. 

Q. 58 What is the position (radial distance) of the most motorist when he reaches the last signal? 

A. 45 km directly north of the starting point 

B. 30 km directly to the east of the starting point 

C. 50 km away to the north-east of the starting point 

D. 45 km away to the north-west of the starting point 

Answer: C. 

Explanation: 

According to given conditions, the car first travels 10 km north then 10 km to west then gain 20 km north then further travels east for 40 km then travel north for another 10 km before finally stopping. So in all the car travels 30 km to the east and 40 km north. Hence, calculating the distance radially we get 50 km in North-East direction. 

Q. 59 After the starting point if the 1st signal were 1 red and 2 green lights, what would be the final position of the motorist? 

A. 30 km to the west and 20 km to the south 

B. 30 km to the west and 40 km to the north 

C. 50 km to the east and 40 km to the north 

D. Directly 30 km to the east 

Answer: A. 

Explanation: 

Now if after starting point at first point the 1st signal were 1 red and 2 green lights then distance traveled by the car would be as follows : 10 km in north then 10 km east then 20 km south then 40 km to west then 10 km to south . So overall the position of car is 30 km to the west and 20 km to south as compared to original position. 

Q. 60 If at the starting point, the car was heading towards south, what would be the final position of the motorist? 

A. 30 km to the east and 40 km to the south 

B. 50 km to the east and 40 km to the south 

C. 30 km to the west and 40 km to the south 

D. 50 km to the west and 20 km to the north 

Answer: C. 

Explanation: 

If at the starting point, the car was heading towards south then following would have occured : 1st 10 km in south ,2nd 10 km east, 3rd 20 km in south ,4th 40 km west, 5tht 10 km in south so overall 30 km to the west and 40 km to the south from starting point. 

Instructions 

The following table provides data on the different countries and location of their capitals. (the data may not match the actual Latitude, Longitudes) Answer the following questions on the basis of this table. 

Q. 61 What percentage of cities located within 10 E° and 40 E° (10° East and 40° East) lie in the Southern Hemisphere? 

A. 15% 

B. 20% 

C. 25% 

D. 30% 

Answer: B. 

Explanation: 

There are 5 countries which lie from 10 Deg E to 40 Deg E , but out of those only 1 lie in southern hemisphere. hence 1*100/5 = 20%. 

Q. 62 The number of cities whose names begin with a consonant and are in the Northern Hemisphere in the table 

A. exceeds the number of cities whose names begin with a consonant and are in the southern hemisphere by 1 

B. exceeds the number of cities whose names begin with a consonant and are in the southern hemisphere by 2

 C. is less than the number of cities whose names begin with a consonant and are in the east of the meridian by 1 

D. is less than the number of countries whose name begins with a consonant and are in the east of the meridian by 2 

Answer: D. 

Explanation: 

There are 11 cities whose names begin with a consonant and are in the Northern Hemisphere in the table and there are 13 number of countries whose name begins with a consonant and are in the east of the meridian. Hence option D. 

Q. 63 The ratio of the number of countries whose name starts with vowels and located in the southern hemisphere, to the number of countries, the name of whose capital cities starts with a vowel in the table above is 

A. 3 : 2 

B. 3 : 3 

C. 3 : 1 

D. 4 : 3 

Answer: A. 

Explanation: 

Number of countries whose name starts with vowels and located in the southern hemisphere are 3 and number of countries, the name of whose capital cities starts with a vowel in the table is 2. 

Hence the ratio 3:2. 

Instructions 

The following table gives details regarding the total earnings of 15 employees and the number of days they have worked on complex, medium and simple operation in the month of June 2002. Even though the employees might have worked on an operation, they would be eligible for earnings only if they have a minimum level of efficiency. 

Q. 64 The number of employees who have earned more than Rs. 50 per day in complex operations is 

A.

B.

C.

D.

Answer: C. 

Explanation: 

Average income can be found by dividing the total income by total number of days. 

The average income is calculated as show in the table given below. 

Using this formula, we can find that there are 5 employees (Employee IDs: 1151, 1158, 1164, 1172, and 1173) who  earned more than Rs 50 per day. 

Q. 65 The number of employees who have earned more than Rs. 600 and having more than 80% attendance (there are 25 regular working days in June 2002; some might be coming on overtime too) is 

A.

B.

C.

D.

Answer: D. 

Explanation: 

80% of attendance comes out to 20 days. 

We can see from the table that there are 7 such employees who have earned more than Rs. 600 and having more than 80% of attendance. 

Hence option D. 

Q. 66 The employee number of the person who has earned the maximum earnings per day in medium operation is 

A. 2001180 

B. 2001164 

C. 2001172 

D. 2001179 

Answer: A. 

Explanation: 

We need to find the person who has the highest average per-day earnings in medium operations. So, we divide the total income in medium operations by the total number of day in these operations. Employee no. 2001180 earned 1262.9 by working 19 daysfor medium operation which is highest. Hence option A. 

Q. 67 Among the employees who were engaged in complex and medium operations, the number of employees whose average earning per day in complex operations is more than average earning per day in medium operations is 

A.

B.

C.

D.

Answer: D. 

Explanation: 

The average income is calculated as show in the table given below. 

From the table given we can make out that number of employees whose average earning per day in complex 

operations is more than average earning per day in medium operations for 8 employees (2001148, 2001149, 2001150, 2001151, 2001158, 2001164, 2001171, 2001172) 

Instructions 

The following table shows the revenue and expenses in millions of Euros (European currency) associated with REPSOL YPF company’s oil and gas producing activities in operations in different parts of the world for 1998-2000. 

Q. 68 How many operations (Spain, North Africa and Middle East,..) of the company accounted for less than 5% of the total revenue earned in 1999? 

A.

B.

C.

D. None of these 

Answer: C. 

Explanation: 

5% of total revenue in the year 1999 is 168.7 . Manually counting number of countries where the revenue is less than 168.7, we see that 4 countries have revenue less than 168.7 

Q. 69 How many operations (Spain, North Africa and Middle East…) of the company witnessed more than 200% increase in revenue from 1999 to 2000? 

A.

B.

C.

D. None of these 

Answer: B. 

Explanation: 

Spain and rest of latin america has witnessed more than 200% increase in revenue from 1999 to 2000 for the company. 

Q. 70 How many operations registered a sustained yearly increase in income before taxes and charges from 1998 to 2000? 

A.

B.

C.

D. None of these 

Answer: B. 

Explanation: 

There are 4 such operations – North Africa & Middle East, Argentina, Rest of Latin America and Far East – that registered a sustained yearly increase in income before taxes and charges from 1998 to 2000. 

Q. 71 Ignoring the loss making operations of the company in 1998, for how many operations was the percentage increase in net income before taxes and charges higher than the average from 1998 to 1999? 

A.

B.

C.

D. None of these 

Answer: B. 

Explanation: 

Percentage increase in net income before tax and 

charges for total world (1998-99) 

=1375−248/248 *100 = 454.4% 

Spain is making loss. 

Percentage increase for North Africa and Middle-East =341−111/111*100  = 207.2% 

Percentage increase for Argentina =838−94/94*100=791.5% 

From the table one can directly say that there is no operation other than Argentina, whose percentage increase in net income before taxes and charges is higher than the average (world). 

Q. 72 If profitability is defined as the ratio of net income after taxes and charges to expense, which of the following statements is true? 

A. The Far East operations witnessed its highest profitability in 1998 

B. The North Sea operations increased its profitability from 1998 to 1999 

C. The operations in Argentina witnessed a decrease in profitability from 1998 to 1999 

D. Both 2 and 3 are true 

Answer: B. 

Explanation: 

Profitability for the North Sea operations in 1998 was 20/52 and in the year 1999 was 54/65 . Hence we can clearly see that the profitability increased. hence option B. 

Q. 73 In 2000, which among the following countries had the best profitability? 

A. North Africa and Middle East 

B. Spain 

C. Rest of Latin America 

D. Far East 

Answer: B. 

Explanation: 

Profitability= Net Income After Taxes / Expenses

In 2000, Spain had the best profitability of 225/43. 

Hence option B. 

Q. 74 If efficiency is defined as the ratio of revenue to expenses, which operation was the least efficient in 2000? 

A. Spain 

B. Argentina 

C. Far East 

D. None of these 

Answer: D. 

Explanation: 

Efficiency = revenue / expenses

In 2000, except Rest of the world, all the other operations had an efficiency of greater than 1.

Efficiency of Rest of the Word =20/33 , which is the least. 

Q. 75 Of the following statements, which one is not true? 

A. The operations in Spain had the best efficiency in 2000 

B. The Far East operations witnessed an efficiency improvement from 1999 to 2000

C. The North Sea operations witnessed an efficiency improvement from 1998 to 1999 

D. In 1998, the operations in Rest of Latin America were the least efficient

Answer: D. 

Explanation: 

In 1998, the efficiency in Rest of Latin America was 34/57. 

In 1998, the efficiency in Rest of the world was 5/15, which is less than 34/57. 

Hence, the efficiency of rest of latin america was not the least. 

Instructions 

Chart 1 shows the distribution by value of top 6 suppliers of MFA. Textiles in 1995. 

Chart 2 shows the distribution by quantity of top 6 suppliers of MFA. Textiles in 1995. 

The total value is 5760 million Euro (European currency). The total quantity is 1.055 million tonnes.

Q.76 The country which has the highest average price is 

A. USA. 

B. Switzerland 

C. Turkey 

D. India 

Answer: B. 

Explanation: 

The country which has the highest average price would have low quality and high value when compared to other countries.. 

Switzerland fits the above description. 

Hence, option B. is the answer. 

Q. 77 The average price in Euro per kilogram for Turkey is roughly 

A. 6.20 

B. 5.60 

C. 4.20 

D. 4.80 

Answer: B. 

Explanation: 

Turkey has a value of 16% and a quantity of 15%. 

The average price in Euro per kilogram for Turkey is (16*5.760)/(15*1.055) which is nearly equal to 5.6. Hence option B. 

Instructions 

There are 6 refineries, 7 depots and 9 districts. The refineries are BB, BC, BD, BE, BF and BG. The depots are AA, AB, AC, AD, AE, AF and AG. The districts are AAA, AAB, AAC, AAD, AAE, AAF, AAG, AAH, and AAI. Table A. gives the cost of transporting one unit from refinery to depot. Table B. gives the cost of transporting one unit from depot to a district. 

Q. 78 What is the least cost of sending one unit from any refinery to any district? 

A. 95.2 

B.

C. 205.7 

D. 284.5 

Answer: B. 

Explanation: 

The least cost of sending one unit is from BC to AC and Then to AAC which is 0 . Hence option B.

Q. 79 What is the least cost of sending one unit from any refinery to the district AAB? 

A.

B. 284.5 

C. 95.2 

D. None of these 

Answer: A. 

Explanation: 

The least cost of sending one unit from any refinery to the district AAB is from BD to AE and then from AE to AAB. The whole cost is 0. 

Hence option A. 

Q. 80 What is the least cost of sending one unit from refinery BB. to any district? 

A. 284.5 

B. 311.8 

C. 451.1 

D. None of these 

Answer: B. 

Explanation: 

Route for least cost of sending one unit from refinery BB to any district, would be first from BB to AB and then from AB to AAC 

Hence total cost 311.8 + 0 = 311.8 

Q. 81 What is the least cost of sending petrol from refinery BB. to district AAA? 

A. 765.6 

B. 1137.3 

C. 1154.3 

D. None of these 

Answer: D. 

Explanation: 

From the table, we can observe that the minimum cost of sending oil fro BB. to AAA. is via AB. Hence the total cost is 311.8+205 = 516.8 

Q. 82 How many possible ways are there for sending petrol from any refinery to any district? 

A. 63 

B. 42 

C. 54 

D. 378 

Answer: D. 

Explanation: 

1 refinery can be selected among 7 in 7 ways, depots can be further selected in 6 ways and any district ca be selected in 9 different ways. 

So, answer is 7*6*9 = 378 

Q. 83 The largest cost of sending petrol from any refinery to any district is 

A. 2172.6 

B. 2193.0 

C. 2091.0 

D. None of these 

Answer: B. 

Explanation: 

The costliest route would be from either BE or BF to AE then AE to AAH i.e 1157.7+1035.3 = 2193

Instructions 

The chart given below indicates the annual sales tax revenue collections (in rupees in crores) of seven states from 1997 to 2001. 

The values given at the top of each bar represents the total collections in that year 

Q. 84 If for each year, the states are ranked in terms of the descending order of sales tax collections, how many states do not change the ranking more than once over the five years? 

A.

B.

C.

D.

Answer: B. 

Explanation: 

We have to consider the ranking of that state that doesn’t change throughout or changes only once. The rankings of the states are as follows: 

So, the states West Bengal, Tamil Nadu, Maharashtra, Karnataka and Andhra Pradesh do not change their rankings more than once. There are 5 states in total under the criteria. 

Q. 85 Which of the following states has changed its relative ranking most number of times when you rank the states in terms of the descending volume of sales tax collections each year? 

A. Andhra Pradesh 

B. Uttar Pradesh 

C. Karnataka 

D. Tamil Nadu 

Answer: B. 

Explanation: 

Ranking of UP changed 2 times which is highest among other states. 

Q. 86 The percentage share of sales tax revenue of which state has increased from 1997 to 2001? 

A. Tamil Nadu 

B. Karnataka 

C. Gujarat 

D. Andhra Pradesh 

Answer: D. 

Explanation: 

The overall revenue has increased by a ratio of 49639 / 29870 = 1.67. 

So, the state whose revenue ratio increases by more than 1.67 is the answer. 

Among the given options, only AP’s sales tax revenue ratio has increased. 

Q. 87 Which pair of successive years shows the maximum growth rate of tax revenue in Maharashtra? 

A. 1997 to 1998 

B. 1998 to 1999 

C. 1999 to 2000 

D. 2000 to 2001 

Answer: C. 

Explanation: 

Max growth rate can be calculated by finding percentage growth over previous year. 

We can see a significant increase in the year 1999-2000. 

On calculating the ratio, we get 10284 / 8067 = 1.275, which is more than any other year. 

Hence, 1999-2000 is the answer.

Q. 88 Identify the state whose tax revenue increased exactly by the same amount in two successive pairs of years? 

A. Karnataka 

B. West Bengal 

C. Uttar Pradesh 

D. Tamil Nadu 

Answer: A. 

Explanation: 

Simple calculations show difference in values of sales for karnataka over 2 successive pair of years – 1999-2000 and 2000-2001 – same. 

The increase in 1999-2000 = 4839 – 4265 = 574 

The increase in 2000-2001 = 5413 – 4839 = 574 

Q. 89 Which state below has been maintaining a constant rank over the years in terms of its contribution to total tax collections? 

A. Andhra Pradesh 

B. Karnataka 

C. Tamil Nadu 

D. Uttar Pradesh 

Answer: C. 

Explanation: 

Tamilnadu has maintained the ranking of second highest tax revenue throughout the five years. Hence, option C. is the answer. 

 

Instructions 

The table below gives information about four different crops, their different quality, categories and the regions where they are cultivated. Based on the information given in the table answer the questions below. 

Q. 90 How many regions produce medium qualities of Crop-1 or Crop-2 and also produce low quality of Crop-3 or Crop-4? 

A. Zero 

B. One 

C. Two 

D. Three 

Answer: B. 

Explanation: 

It can be seen that region 9 produces medium qualities of Crop-1 or Crop-2 and also produce low quality of Crop-3 or Crop-4. 

Q. 91 Which of the following statements is true? 

A. All medium quality Crop-2 producing regions are also high quality Crop-3 producing regions. 

B. All high quality Crop-1 producing regions are also medium and low Crop-4 producing regions. 

C. There are exactly four Crop-3 producing regions, which also produce Crop-4 but not Crop-2. 

D. Some Crop-3 producing regions produce Crop-1, but not high quality Crop-2. 

Answer: D. 

Explanation: 

High quality crop 2 is produced by R5, R8 and R12. 

None of these regions produce crop 3. 

Hence, option D is true. 

None of the other options are true. 

Q. 92 How many low quality Crop-1 producing regions are either high quality Crop-4 producing regions or medium quality Crop-3 producing regions? 

A. One 

B. Two 

C. Three 

D. Zero 

Answer: C. 

Explanation: 

Take a look at the table, 

The low producing areas corresponding to Crop 1 is R9, R10, R11 

Now medium yield area corresponding to Crop 3 is R9 and high yield are corresponding to crop 4 are R10 and R11. Hence all three area are mentioned in the above two categories. 

Instructions 

For the following questions answer them individually 

Q. 93 In a hockey match, the Indian team was behind by 2 goals with 5 min remaining. Did they win the match? A. Deepak Thakur, the Indian striker, scored 3 goals in the last 5 min of the match. B. Korea scored a total of 3 goals in the match. 

A. The question can be answered by one of the statement alone but not by the other. 

B. The question can be answered by using either statement alone. 

C. The question can be answered by using both the statements together, but cannot be answered by using either statement alone. 

D. The question cannot be answered by either of the statements. 

Answer: D. 

Explanation: 

Considering statement B there are 2 possible score values before last 5 mins possible. Korea 3:1 India or Korea 2:0 India. Now considering statement A , in one case India wins (as the score becomes India 4:3 Korea) and in other there is a draw(as the score becomes India 3:3 Korea). Hence, The question cannot be answered by either of the statements. 

Q. 94 Four students were added to a dance class. Would the teacher be able to divide her students evenly into a dance team (or teams) of 8? 

A. If 12 students were added, the teacher could put everyone in teams of 8 without any leftovers. 

B. The number of students in the class earlier was not divisible by 8. 

A. The question can be answered by one of the statement alone but not by the other. 

B. The question can be answered by using either statement alone. 

C. The question can be answered by using both the statements together, but cannot be answered by using either statement alone. 

D. The question cannot be answered by either of the statements. 

Answer: A. 

Explanation: 

Considering statement A 12 students can be written as 4 students + 8 students. Hence Even if only 4 students are added the teacher can divide students in teams of 8. Now considering statement B , we cant perfectly tell if after adding teacher can divide in teams of 8 . Hence only A is enough to answer the question. Hence , The question can be answered by one of the statement alone but not by the other. 

Q. 95 Is x = y

A. (x + y)(1/x + 1/y) = 4

B. (x − 50)2 = (y − 50)2 

A. The question can be answered by one of the statement alone but not by the other. 

B. The question can be answered by using either statement alone. 

C. The question can be answered by using both the statements together, but cannot be answered by using either statement alone. 

D. The question cannot be answered by either of the statements. 

Answer: A. 

Explanation: 

Consider statement 1: (x + y)(1/x + 1/y) = 4

(x + y)2/xy = 4 

x2 + y2 + 2xy = 4xy 

x2 + y2 + 2xy − 4xy = 0 

(x y)2 = 0 

x = y. 

Consider statement 2: (x − 50)2 = (y − 50)2 

(x-y)(x+y-100) = 0 

Either x = y or x+y = 100. 

Statement 1 is sufficient whereas statement 2 is not sufficient to answer the question. 

Q. 96 A dress was initially listed at a price that would have given the store a profit of 20% of the wholesale cost. What was the wholesale cost of the dress? 

A. After reducing the listed price by 10%, the dress sold for a net profit of $10. 

B. The dress is sold for $50. 

A. The question can be answered by one of the statement alone but not by the other. 

B. The question can be answered by using either statement alone. 

C. The question can be answered by using both the statements together, but cannot be answered by using either statement alone. 

D. The question cannot be answered by either of the statements. 

Answer: A. 

Explanation: 

Let the initial selling price be s1 and changed value be s2, also wholesale cost price be c. According to the given condition we have s1=1.28*c. By statement A. we have s2=0.9*s1 and s2-c = 10 . Hence, wholesale cost price can be found out. 

Considering statement B. we have just s2=50. Using this we can’t find c. Hence, the question can be answered by one of the statements alone but not by the other. 

Q. 97 Is 500 the average (arithmetic mean) score in the GMAT? 

A. Half of the people who take the GMAT score above 500 and half of the people score below 500. B. The highest GMAT score is 800 and the lowest score is 200. 

A. The question can be answered by one of the statement alone but not by the other. 

B. The question can be answered by using either statement alone. 

C. The question can be answered by using both the statements together, but cannot be answered by using either statement alone. 

D. The question cannot be answered by either of the statements. 

Answer: D. 

Explanation: 

Consider statement A. alone and assume that half of the people scored 600 and half of the people scored 499, so average is not 500 

Also there is a possibility that average can be 500 when half of the people score about 500 and the other half score below 500. 

Hence we won’t get unique answer using A. 

Consider statement B. alone: We can’t find average just by knowing the highest and the lowest score. Even by using both the statements, we cannot find the answer to the question. Option d) is the correct answer. 

Q. 98 Is |x – 2| < 1? 

A. |x| < 1 

B. |x – 1| < 2 

A. if the question can be answered by using either statement alone. 

B. if the question can be answered by one of the statement alone but not by the other. 

C. if the question can be answered by using both the statements together, but cannot be answered by using either statement alone. 

D. if the question cannot be answered by either of the statements. 

Answer: B. 

Explanation: 

Statement A: -1<x<1 

-3<x-2<-1 

So 1<mod(x-2)<3 

It is definitely greater than 1 

Statement B: -2<x-1<2 

-3<x-2<1 

which implies mod(x-2) might be greater than or less than 1 

So only one statement is sufficient. 

Q. 99 People in a club either speak French or Russian or both. Find the number of people in a club who speak only French. 

A. There are 300 people in the club and the number of people who speak both French and Russian is 196. 

B. The number of people who speak only Russian is 58. 

A. if the question can be answered by one of the statement alone but not by the other. 

B. if the question can be answered by using either statement alone. 

C. if the question can be answered by using both the statements together, but cannot be answered by using either statement alone. 

D. if the question cannot be answered by either of the statements. 

Answer: C. 

Explanation: 

Let x,y,z be the no. of people speaking only French, only Russian and both respectively. By statement A. we have x+y+z = 300 and z = 196. Using this we can’t find number of people in a club who speak only French. Now considering statement B. we have x = 58. So using both the statements together we can find number of people in a club who speak only French. Hence, option C. is the correct answer. 

Q. 100 A sum of Rs. 38,500 was divided among Jagdish, Punit and Girish. Who received the minimum amount? 

A. Jadgish received 2/9 of what Punit and Girish received together. 

B. Punit received 3/11 of what Jadgish and Girish received together. 

A. if the question can be answered by one of the statement alone but not by the other. 

B. if the question can be answered by using either statement alone. 

C. if the question can be answered by using both the statements together, but cannot be answered by using either statement alone. 

D. if the question cannot be answered by either of the statements. 

Answer: C. 

Explanation: 

Let amount received by Jagdish, Punit and Girish be j,p,g rs. such that j+p+g = 38500. According to statement A , j= 2* (p+g)/9 . Using this alone we can’t answer the question. 

Now consider statement B , we have p = 3*(j+g)/11. Using this alone we can’t answer the question. But considering both statements together we can answer the question. Hence, option C. is the correct answer. 

Verbal 

Instructions 

Directions for the next 3 questions: Fill the gaps in the passages below with the most appropriate word from the options given for each gap. The right words are the ones used by the author. Be guided by the author’s overall style and meaning when you choose the answers. 

Von Nuemann and Morgenstern assume a decision framework in which all options are thoroughly considered, each option being independent of the others, with a numerical value derived for the utility of each possible outcome (these outcomes reflecting, in turn, all possible combinations of choices). The decision is then made to maximize the expected utility. (…1…) such a model reflects major simplifications of the way divisions are made in the real world. Humans are not able to process information as quickly and effectively as the model assumes; they tend not to think (…2…) as easily as the model calls for; they often deal with a particular option without really assessing its (…3…) and when they do assess alternatives, they may be extremely nebulous about their criteria of evaluation. 

Q. 101 Fill the gaps in the passages (…1…) 

A. Regrettably 

B. Firstly 

C. Obviously 

D. Apparently 

Answer: A. 

Explanation: 

The first two sentences of the paragraph start of with the description of the Von Nuemann framework on a positive note. The third sentence, however, points out the flaw in such a framework. The correct word that goes into the blank 1 is, therefore, ‘Regrettably’. 

Q. 102 Fill the gaps in the passages (…2…) 

A. quantitatively 

B. systematically 

C. scientifically 

D. analytically 

Answer: A. 

Explanation: 

The first part of the sentence, ‘Humans are not able to process information as quickly and effectively as the model assumes;’ hints that that the word in the blank should be ‘quantitatively’. 

Q. 103 Fill the gaps in the passages (…3…) 

A. implications 

B. disadvantages 

C. utility 

D. alternatives 

Answer: D. 

Explanation: 

The usage of the phrase “particular option” complements the word “alternatives”. 

Hence, option D. is the answer. 

Instructions 

Directions for the next 3 questions: Fill the gaps in the passages below with the most appropriate word from the options given for each gap. The right words are the ones used by the author. Be guided by the author’s overall style and meaning when you choose the answers. 

In a large company, (…1…) people is about as common as using a gun or a switch-blade to (…2…) an argument. As a result, most managers have little or no experience of firing people, and they find it emotionally traumatic; as result, they often delay the act interminably, much as an unhappy spouse will prolong a bad marriage. And when the firing is done, it’s often done clumsily, with far worse side effects than are necessary. Do the world-class software organizations have a different way of firing people? No, but they do the deed swiftly, humanely, and professionally. The key point here is to view the fired employee as a ‘failed product’ and to ask how the process (…3…). such a phenomenon in the first place. 

Q. 104 Fill in the blank at (…1…). 

A. dismissing 

B. punishing 

C. firing 

D. admonishing 

Answer: C. 

Explanation: 

The second sentence ‘As a result, most managers have little or no experience of firing people,…’ gives us the hint that the word in the first blank should be firing. 

Q. 105 Fill in the blank at (…2…). 

A. resolve 

B. thwart 

C. defeat 

D. close 

Answer: A. 

Explanation: 

An argument is resolved and not thwarted or defeated or closed. So, the word in the second blank is ‘resolve’. 

Q. 106 Fill in the blank at (…3…). 

A. derived 

B. engineered 

C. produced 

D. allowed 

Answer: D. 

Explanation: 

According to the statement, a fired employee is a ‘failed product’ by itself and is not something that has been produced or trained by the organization. Therefore, a failure of the process is not likely to derive or engineer or produce such a product but it is likely to allow such a product. Therefore, the correct answer is ‘allowed’. 

Instructions 

Choose the best answer for each question. 

The production of histories of India has become very frequent in recent years and may well call for some explanation. Why so many and why this one in particular? The reason is a two-fold one: changes in the Indian scene requiring a re interpretation of the facts and changes in attitudes of historians about the essential elements of Indian history. These two considerations are in addition to the normal fact of fresh information, whether in the form of archeological discoveries throwing fresh light on an obscure period or culture, or the revelations caused by the opening of archives or the release of private papers. The changes in the Indian scene are too obvious to need emphasis. Only two generations ago British rule seemed to most Indian as well as British observers likely to extend into an indefinite future; now there is a teenage generation which knows nothing of it. Changes in the attitudes of historians have occurred everywhere, changes in attitudes to the content of the subject as well as to particular countries, but in India there have been some special features. Prior to the British, Indian historiographers were mostly Muslims, who relied, as in the case of Sayyid Ghulam Hussain, on their own recollection of events and on information from friends and men of affairs. Only a few like Abu’l Fazl had access to official papers. These were personal narratives of events, varying in value with the nature of the writer. The early British writers were officials. In the 18th century they were concerned with some aspect of Company policy, or like Robert Orme in his Military Transactions gave a straight narrative in what was essentially a continuation of the Muslim tradition. In the early 119th century the writers were still, with two notable exceptions, officials, but they were now engaged in chronicling, in varying moods of zest, pride, and awe, the rise of the British power in India to supremacy. The two exceptions were James Mill, with his critical attitude to the Company and John Marchman, the Baptist missionary. But they, like the officials, were anglo-centric in their attitude, so that the history of modern India in their hands came to be the history of the rise of the British in India. 

The official school dominated the writing of Indian history until we get the first professional historian’s approach. Ramsay Muir and P. E. Roberts in England and H. H. Dodwell in India. Then Indian historians trained in the English school joined in, of whom the most distinguished was Sir Jadunath Sarkar and the other notable writers: Surendranath Sen, Dr Radhakumud Mukherji, and Professor Nilakanta Sastri. They, it may be said, restored India to Indian history, but their bias was mainly political. Finally have come the nationalists who range from those who can find nothing good or true in the British to sophisticated historical philosophers like K. M. Panikker. 

Along the types of historians with their varying bias have gone changes in the attitude to the content of Indian history. Here Indian historians have been influenced both by their local situation and by changes of thought elsewhere. It is this field that this work can claim some attention since it seeks to break new ground, or perhaps to deepen a freshly turned furrow in the field of Indian history. The early official historians were content with the glamour and drama of political history from Plassey to the Mutiny, from Dupleix to the Sikhs. But when the raj was settled down, glamour departed from politics, and they turned to the less glorious but more solid ground of administration. Not how India was conquered but how it was governed was the theme of this school of historians. It found its archpriest in H. H. Dodwell, its priestess in Dame Lilian Penson, and its chief shrine in the Volume VI of the Cambridge History of India. Meanwhile, in Britain other currents were moving, which led historical study into the economic and social fields. R. C. Dutt entered the first of these currents with his Economic History of India to be followed more recently by the whole group of Indian economic historians. W. E. Moreland extended these studies to the Mughal Period. Social history is now being increasingly studied and there is also of course a school of nationalist historians who see modern Indian history in terms of the rise and the fulfillment of the national movement. 

All these approaches have value, but all share in the quality of being compartmental. It is not enough to remove political history from its pedestal of being the only kind of history worth having if it is merely to put other types of history in its place. Too exclusive an attention to economic, social, or administrative history can be as sterile and misleading as too much concentration on politics. A. whole subject needs a whole treatment for understanding. A. historian must dissect his subject into its elements and then fuse them together again into an integrated whole. The true history of a country must contain all the features just cited but must present them as parts of a single consistent theme. 

Q. 107 Which of the following may be the closest in meaning to the statement ‘restored India to Indian history’? 

A. Indian historians began writing Indian history. 

B. Trained historians began writing Indian history. 

C. Writing India-centric Indian history began. 

D. Indian history began to be written in India 

Answer: C. 

Explanation: 

Refer to the lines where it is written that “Then Indian Historians trained in the English school joined in, of whom the most distinguished was Sir Jadunath Sarkar and the other notable writers: Surendranath Sen, Dr. Radhakumud Mukerji, and Professor Nilakanta Shastri. They, it may be said, restored India to Indian history, but their bias was mainly political.” 

Also refer the last lines of the 1st para”But they, like the officials, were anglo-centric in their attitude, so that the history of modern India in their hands came to be the history of the rise of the British in India.” 

This clearly indicates option C. 

Q. 108 Which of the following is the closest implication of the statement ‘to break new ground, or perhaps to deepen a freshly turned furrow’? 

A. Dig afresh or dig deeper. 

B. Start a new stream of thought or help establish a recently emerged perspective. 

C. Begin or conduct further work on existing archeological sites to unearth new evidence. 

D. Begin writing a history free of any biases. 

Answer: B. 

Explanation: 

The second paragraph is about changes in historians throughout British era. Then author mentioned historians and their work post British era. 

In the third paragraph author mentioned 

“Along the types of historians with their varying bias have gone changes in the attitude to the content of Indian history. Here Indian historians have been influenced both by their local situation and by changes of thought elsewhere. It is this field that this work can claim some attention since it seeks to break new ground, or perhaps to deepen a freshly turned furrow in the field of Indian history.” Here author mentioned that modern historians come with varying bias, change in their attitude. This can also result in new or in depth discoveries, perspectives. 

Hence, option B. is correct. 

Option A. is not a complete representation of what author said, or what he intended to. Option B. is a closer implication than option A. 

By the line ‘‘to break new ground, or perhaps to deepen a freshly turned furrow’ author do not imply to begin writing a history free of any biases. It is because of historians free of bias we can find new perspective or start a new stream of thought. Hence, option D. is incorrect. 

Between B. and C, B. is more appropriate as compared to C. because option C. only talks about further work rather than starting from new. 

Q. 109 Historians moved from writing political history to writing administrative history because 

A. attitudes of the historians changed. 

B. the raj was settled down. 

C. politics did not retain its past glamour. 

D. administrative history was based on solid ground. 

Answer: C. 

Explanation: 

Refer to the 2nd last para where it is written “The early official historians were content with the glamour and drama of political history: Plassey to the Mutiny, from Dupleix to the Sikhs. But when the raj was settled down, glamour departed from politics, and they turned to the less glorious but more solid ground of administration.” Using this line, we can say that option C. is the answer. 

Q. 110 According to the author, which of the following is not among the attitudes of Indian historians of Indian origin? 

A. Writing history as personal narratives. 

B. Writing history with political bias. 

C. Writing non-political history due to lack of glamour. 

D. Writing history by dissecting elements and integrating them again. 

Answer: D. 

Explanation: 

Refer to the following lines 

“Only a few like Abul Fazl had access to official papers. These were personal narrative of events varying in value with the nature of the writer.”-1st para – This line eliminates option A. 

“Then Indian Historians trained in the English school joined in, of whom the most distinguished was Sir Jadunath Sarkar and the other notable writers: Surendranath Sen, Dr. Radhakumud Mukerji, and Professor Nilakanta Shastri. They, it may be said, restored India to Indian history, but their bias was mainly political.”-2nd para – This line eliminates option B. 

“The early official historians were content with the glamour and drama of political history: Plassey to the Mutiny, from Dupleix to the Sikhs. But when the raj was settled down, glamour departed from politics, and they turned to the less glorious but more solid ground of administration.”-3rd para – This line eliminates option C. 

Option D. is not mentioned in the passage. Hence, option D. is the answer. 

Q. 111 In the list given below, match the historians to the approaches taken by them. 

A. – Administrative 

B. -Political 

C. – Narrative 

D. – Economic 

E – Robert Orme 

F – H.H. Dodwell 

G – Radha Kumud Mukherji 

H – R.C. Dutt 

A. A. – F, B. – G, C. – E, D. – H 

B. A. – G, B. – F, C. – E, D. – H 

C. A. – E, B. – F, C. – G, D. – H 

D. A. – F, B. – H, C. – E, D. – G 

Answer: A. 

Explanation: 

“Robert Orme in his Military Transactions gave a straight narrative” => C – E 

“Then Indian historians trained in the English school joined in, of whom the most distinguished was Sir Jadunath Sarkar and the other notable writers: Surendranath Sen, Dr Radhakumud Mukherji, and Professor Nilakanta Sastri. They, it may be said, restored India to Indian history, but their bias was mainly political” => B – G 

“R. C. Dutt entered the first of these currents with his Economic History” => D – H 

=> Option A. is the answer. 

Instructions 

There are a seemingly endless variety of laws, restrictions, customs and traditions that affect the practice of abortion around the world. Globally, abortion is probably the single most controversial issue in the whole area of women’s rights and family matters. It is an issue that inflames women’s right groups, religious institutions, and the self-proclaimed ‘guardians’ of public morality. The growing worldwide belief is that the right to control one’s fertility is a basic human right. This has resulted in a worldwide trend towards liberalization of abortion laws. Forty per cent of the world’s population live in countries where induced abortion is permitted on request. An additional 25 per cent live in countries where it is allowed if the women’s life would be endangered if she went to full term with her pregnancy. The estimate is that between 26 and 31 million legal abortions were performed in that year. However, there were also between 10 and 22 million illegal abortions performed in that year. 

Feminists have viewed the patriarchal control of women’s bodies as one of the prime issues facing the contemporary women’s movement. They observe that the definition and control of women’s reproductive freedom have always been the province of men. Patriarchal religion, as manifest in Islamic fundamentalism,traditionalist Hindu practice, orthodox Judaism, and Roman Catholicism, has been an important historical contributory factor for this and continues to be an important presence in contemporary societies. In recent times, govenments, usually controlled by men, have ‘given’ women the right to contraceptive use and abortion access when their countries were perceived to have an overpopulation problem. When these countries are perceived to be underpopulated, that right had been absent. Until the 19th century, a woman’s rights to an abortion followed English common law; it could only be legally challenged if there was a ‘quickening’, when the first movements of the fetus could be felt. In 1800, drugs to induce abrotions were widely advertised in local newspapers. By 1900, abortion was banned in every state except to save the life of the mother. The change was strongly influenced by medical profession, which focussed its campaign ostensibly on health and safety issues for pregnant women and the sanctity of life. Its position was also a means of control of non-licensed medical practitioners such as midwives and women healers who practiced abortion. 

The anti-abortion campaign was also influenced by political considerations. The large influx of eastern and southern European immigrants with their large families was seen as a threat to the population balance of the future United States. Middle and upper-classes Protestants were advocates of abortion as a form of birth control. By supporting abortion prohibitions the hope was that these Americans would have more children and thus prevent the tide of immigrant babies from overwhelming the demographic characteristics of Protestant America. 

The anti-abortion legislative position remained in effect in the United States through the first 65 years of the 20th century. In the early 1960s, even when it was widely known that the drug thalidomide taken during pregnancy to alleviate anxiety was shown to contribute to the formation of deformed ‘flipper-like’ hands or legs of children, abortion was illegal in the United States. A second health tragedy was the severe outbreak of rubella during the same time period, which also resulted in major birth defects. These tragedies combined with a change of attitude towards a woman’s right to privacy led a number of states to pass abortion permitting legislation. 

On one side of the controversy are those who call themselves ‘pro-life’. They view the foetus as a human life rather than as an unformed complex of cells; therefore, they hold to the belief that abortion is essentially murder of an unborn child. These groups cite both legal and religious reasons for their opposition to abortion. Pro lifers point to the rise in legalised abortion figures and see this as morally intolerable. On the other side of the issue are those who call themselves ‘pro-choice’. They believe that women, not legislators or judges, should have the right to decide whether and under what circumstances they will bear children. Pro-choicers are of the opinion that laws will not prevent women from having abortions and cite the horror stories of the past when many women died at the hands of ‘backroom’ abortionists and in desperate attempts to self-abort. They also observe that legalized abortion is especially important for rape victims and incest victims who became pregnant. They stress physical and mental health reasons why women should not have unwanted children. 

To get a better understanding of the current abortion controversy, let us examine a very important work by Kristin Luker titled Abortion and the Politics of Motherhood. Luker argues that female pro-choice and prolife activists hold different world views regarding gender, sex, and the meaning of parenthood. Moral positions on abortions are seen to be tied intimately to views on sexual bahaviour, the care of children, family life, technology, and the importance of the individual. Luker identified ‘pro-choice’ women as educated, affluent, and liberal. Their contrasting counterparts, ‘pro life’ women, support traditional concepts of women as wives and mothers. It would be instructive to sketch out the differences in the world views of these two sets of women. Luker examines California, with its liberalized abortion law, as a case history. Public documents and newspaper accounts over a 26-year period were analysed and over 200 interviews were held withheld with both pro-life and pro-choice activists. 

Luker found that pro-life and pro-choice activists have intrinsically different views with respect to gender. Pro-life women have a notion of public and private life. The proper place for men is in the public sphere of work; for women, it is the private sphere of the home. Men benefit through the nurturance of women; women benefit through the protection of men. Children are seen to be the ultimate beneficiaries of this arrangement of having the mother as a full time loving parent and by having clear role models. Pro-choice advocates reject the view of separate spheres. They object to the notion of the home being the ‘women’s sphere’. Women’s reproductive and family roles are seen as potential barriers to full equality. Motherhood is seen as a voluntary, not a mandatory or ‘natural’ role. In summarizing her findings, Luker believes that women become activists in either of the two movements as the end result of lives that centre around different conceptualizations of motherhood. Their beliefs and values are rooted to the concrete circumstances of their lives, their educations, incomes, occupations, and the different marital and family choices that they have made. They represent two different world views of women’s roles in contemporary society and as such the abortion issues represent the battleground for the justification of their respective views. 

Q. 112 According to your understanding of the author’s arguments, which countries are more likely to allow abortion? 

A. Over populated countries like India and China 

B. Underpopulated countries like Australia and Mongolia 

C. Cannot be inferred from the passage 

D. Both (1) and (2) 

Answer: A. 

Explanation: 

Refer to the lines of the 2nd para “In recent times, governments, usually controlled by men, have ‘given’ women the right to contraceptive use and abortion access when their countries were perceived to have an overpopulation problem, when these countries are perceived to be under-populated, that right has been absent.” 

According to this, we can say that option A. is the answer. 

Q. 113 Which amongst these was not a reason for banning of abortions by 1900? 

A. Medical professionals stressing the health and safety of women 

B. Influx of eastern and sourthern European immigrants 

C. Control of unlicensed medical practitioners 

D. A. tradition of matriarchal control 

Answer: D. 

Explanation: 

Refer to the 2nd para and the 3rd para “The change was strongly influenced by the medical profession, which focused its campaign ostensibly on health and safety issues for pregnant women and the sanctity of life. Its position was also means of control of non-licensed medical practitioners such as midwives and women healers who practiced abortion.” “The large influx of eastern and southern European immigrants with their large families was seen as threat to the population balance of the future United States.” 

Options A, B. and C. are stated in these lines. Hence, option D. is the answer.  

Q. 114 A pro-life woman would advocate abortion if 

A. the mother of an unborn child is suicidal. 

B. bearing a child conflicts with a woman’s career prospects. 

C. the mother becomes pregnant accidentally. 

D. None of these 

Answer: D. 

Explanation: 

In the paragraph, it is not mentioned when the pro-life would advocate abortion. 

Hence, none of these is the answer. 

Q. 115 Pro-choice women object to the notion of the home being the ‘women’s sphere’ because they believe 

A. that home is a ‘joint sphere’ shared between men and women. 

B. that reproduction is a matter of choice for women 

C. that men and women are equal 

D. Both (2) and (3) 

Answer: D. 

Explanation: 

Refer to the lines of the last para where it is written ” Pro-choice advocates reject the view of separate spheres. They object to the notion of the home being the ‘women’s sphere’. Women’s reproductive and family roles are seen as potential barriers to full equality. Motherhood is seen as a voluntary, not a mandatory or ‘natural’ role.” Both option B. and option C. are stated in these lines. 

Hence, option D. is the answer. 

Q. 116 Two health tragedies affecting the US society in the 1960s led to 

A. a change in attitude to women’s right to privacy. 

B. retaining the anti-abortion laws with some exceptions. 

C. scrapping of anti-abortion laws. 

D. strengthening of the pro-life lobby. 

Answer: C. 

Explanation: 

Refer to the lines “In the early 1960s, even when it was widely known that the drug thalidomide taken during pregnancy to alleviate anxiety was shown to contribute to the formation of deformed ‘flipper-like’ hands or legs of children, abortion was illegal in the United States. A. second health tragedy combined with a change of attitude towards a woman’s right to privacy led a number of states to pass abortion-permitting legislation.” 

As some abortion permitting laws are passed, it can be inferred that some of the anti abortion laws must have been scrapped in those states. Hence, option C. is the right answer. 

Q. 117 Historically, the pro-choice movements has got support from, among others, 

A. major patriarchal religions. 

B. countries with low population density. 

C. medical profession. 

D. None of these 

Answer: D. 

Explanation: 

It is nowhere mentioned in the paragraph about the support of pro-choice group. Likewise, it is mentioned that the banning of abortion was influenced by medical profession. Patriarchial religions always observed that the women reproductive freedom is always in the domain of men. Hence the correct answer is option D. 

Instructions 

The conceptions of life and the world which we call ‘philosophical’ are a product of two factors: one inherited religious and ethical conceptions; the other, the sort of investigation which may be called ‘scientific’, using this word in its broadest sense. Individual philosophers have differed widely in regard to the proportions in which these two factors entered into their systems, but it is the presence of both, in some degree, that characterizes philosophy. 

‘Philosophy’ is a word which has been used in many ways, some wider, some narrower. I propose to use it in a very wide sense, which I will now try to explain. 

Philosophy, as I shall understand the word, is something intermediate between theology and science. Like theology, it consists of speculations on matters as to which definite knowledge has, so far, been unascertainable; but like science, it appeals to human reason rather than to authority, whether that of tradition or that of revelation. All definite knowledge so I should contend belongs to science; all dogma as to what surpasses definite knowledge belongs to thelogy. But between theology and science there is a ‘No man’s Land’, exposed to attack from both sides; this ‘No Man’s Land’ is philosophy. Almost all the questions of most interest to speculative minds are such as science cannot answer, and the confident answers of theologians no longer seem so convincing as they did in former centuries. Is the world divided into mind and matter, and if so, what is mind and what is matter? Is mind subject to matter, or is it possessed of independent powers? Has the universe any unity or purpose? It is evolving towards some goal? Are there really laws of nature, or do we believe in them only because of our innate love of order? Is man what he seems to the astronomer, a tiny lump of carbon and water impotently crawling on a small and unimportant planet? Or is he what he appears to 

Hamlet? Is he perhaps both at once? Is there a way of living that is noble and another that is base, or are all ways of living merely futile? If there is a way of living that is noble, in what does it consist, and how shall we achieve it? Must the good be eternal in order to deserve to be valued, or is it worth seeking even if the universe is inexorably moving towards death? Is there such a thing as wisdom, or is what seems such merely the ultimate refinement of folly? To such 

questions no answer can be found in the laboratory. Theologies have professed to give answers, all too definite; but their definiteness causes modern minds to view them with suspicion. The studying of these questions, if not the answering of them, is the business of philosophy. 

Why, then, you may ask, waste time on such insoluble problems? To this one may answer as a historian, or as an individual facing the terror of cosmic loneliness. 

The answer of the historian, in so far as I am capable of giving it, will appear in the course of this work. Ever since men became capable of free speculation, their actions in innumerable important respects, have depended upon their theories as to the world and human life, as to what is good and what is evil. This is as true in the present day as at any former time. To understand an age or a nation, we must understand its philosophy, and to understand its philosophy we must ourselves be in some degree philosophers. There is here a reciprocal causation: the circumstances of men’s lives do much to determine their philosophy, but, conversely, their philosophy does much to determine their circumstances. 

There is also, however, a more personal answer. Science tells us what we can know, but what we can know is little, and if we forget how much we cannot know we may become insensitive to many things of very great importance. Theology, on the other hand, induces a dogmatic belief that we have knowledge, where in fact we have ignorance, and by doing so generates a kind of impertinent insolence towards the universe. Uncertainty, in the presence of vivid hopes and fears, is painful, but must be endured if we wish to live without the support of comforting fairy tales. It is good either to forget the questions that philosophy asks, or to persuade ourselves that we have found indubitable answers to them. To teach how to live without certainty, and yet without being paralyzed by hesitation, is perhaps the chief thing that philosophy, in our age, can still do for those who study it. 

Q. 118 The purpose of philosophy is to 

A. reduce uncertainty and choas. 

B. help us to cope with uncertainty and ambiguity. 

C. help us to find explanations for uncertainty. 

D. reduce the terror of cosmic loneliness. 

Answer: B. 

Explanation: 

Refer to the last lines of the last para “To teach how to live without certainly, and yet without being paralyzed by hesitation, is perhaps the chief things that philosophy, in our age, can still do for those who study it.” 

Q. 119 Based on the passage, what can be concluded about the relation between philosophy and science? 

A. The two are antagonistic. 

B. The two are complementary. 

C. There is no relation between the two. 

D. Philosophy derives from science. 

Answer: B. 

Explanation: 

A. and C. are incorrect as there are no references to these options. 

Between B and D, B is more suitable if we are the lines of 3rd para ” All definite knowledge-so I should contend-belongs to science; all dogma as to what surpass definite knowledge to theology. But between theology and science there is a ‘No man’s Land’ exposed to attack from both sides; this ‘No Man’s Land’ is philosophy. Almost all the questions of most interest to speculative minds are such as science cannot answer, and the confident answers of theologians no longer seem so convincing as they did in former centuries. Is the world divided into mind and matter, and if so, what is mind and what is matter? ” 

Q. 120 From reading the passage, what can be concluded about the profession of the author? He is most likely not to be a 

A. historian. 

B. philosopher. 

C. scientist. 

D. theologian. 

Answer: D. 

Explanation: 

The correct answer is D as the author does not give the positive views about theologians throughout the passage. For eg. refer to the given lines:”Almost all the questions of most interest to speculative minds are such as science cannot answer, and the confident answers of theologians no longer seem so convincing as they did in former centuries.” There are many examples in the passage in which the author does not present the favourable views about theologians. 

Q. 121 According to the author, which of the following statements about the nature of universe must be definitely true? 

A. The universe has unity. 

B. The universe has a purpose. 

C. The universe is evolving towards a goal. 

D. None of these 

Answer: D. 

Explanation: 

Refer to the lines of the 3rd para ” Is mind subject matter, or is it possessed of independent powers? Has the universe any unity or purpose? Is it evolving towards some goal? Are there really laws of nature, or do we believe in them only because of our innate love of order? ” 

All options are covered in these lines. According to the author, there are no definite answers to these questions. Hence, none of these statements is definitely true. 

Instructions 

Cells are the ultimate multi-taskers: they can switch on genes and carry out their orders, talk to each other, divide in two, and much more, all at the same time. But they couldn’t do any of these tricks without a power source to generate movement. The inside of a cell bustles with more traffic than Delhi roads, and, like all vehicles, the cell’s moving parts need engines. Physicists and biologists have looked ‘under the hood’ of the cell and laid out the nuts and bolts of molecular engines. 

The ability of such engines to convert chemical energy into motion is the envy nanotechnology researchers looking for ways to power molecule-sized devices. Medical researchers also want to understand how these engines work. Because these molecules are essential for cell division, scientists hope to shut down the rampant growth of cancer cells by deactivating certain motors. Improving motor-driven transport in nerve cells may also be helpful for treating diseases such as Alzheimer’s, Parkinson’s or ALS, also known as Lou Gehrig’s disease. 

We wouldn’t make it far in life without motor proteins. Our muscles wouldn’t contract. We couldn’t grow, because the growth process requires cells to duplicate their machinery and pull the copies apart. And our genes would be silent without the services of messenger RNA, which carries genetic instructions over to the cell’s protein-making factories. 

The movements that make these cellular activities possible occur along a complex network of threadlike fibers, or polymers, along which bundles of molecules travel like trams. The engines that power the cell’s freight are three families of proteins, called myosin, kinesin and dynein. For fuel, these proteins burn molecules of ATP, which cells make when they break down the carbohydrates and fats from the foods we eat. The energy from burning ATP causes changes in the proteins’ shape that allow them to heave themselves along the polymer track. The results are impressive: In one second, these molecules can travel between 50 and 100 times their own diameter. If a car with a five-foot-wide engine were as efficient, it would travel 170 to 340 kilometres per hour. 

Ronald Vale, a researcher at the Howard Hughes Medical Institute and the University of California at San Francisco, and Ronald Milligan of the Scripps Research Institute have realized a long-awaited goal by reconstructing the process by which myosin and kinesin move, almost down to the atom. The dynein motor, on the other hand, is still poorly understood. Myosin molecules, best known for their role in muscle contraction, form chains that lie between filaments of another protein called actin. Each myosin molecule has a tiny head that pokes out from the chain like oars from a canoe. Just as rowers propel their boat by stroking their oars through the water, the myosin molecules stick their heads into the actin and hoist themselves forward along the filament. While myosin moves along in short strokes, its cousin kinesin walks steadily along a different type of filament called a microtubule. Instead of using a projecting head as a lever, kinesin walks on two ‘legs’. Based on these differences, researchers used to think that myosin and kinesin were virtually unrelated. But newly discovered similarities in the motors’ ATP-processing machinery now suggest that they share a common ancestor — molecule. At this point, scientists can only speculate as to what type of primitive cell-like structure this ancestor occupied as it learned to burn ATP and use the energy to change shape. “We’ll never really know, because we can’t dig up the remains of ancient proteins, but that was probably a big evolutionary leap,” says Vale. 

On a slightly larger scale, loner cells like sperm or infectious bacteria are prime movers that resolutely push their way through to other cells. As L. Mahadevan and Paul Matsudaira of the Massachusetts Institute of Technology explain, the engines in this case are springs or ratchets that are clusters of molecules, rather than single proteins like myosin and 

kinesin. Researchers don’t yet fully understand these engines’ fueling process or the details of how they move, but the result is a force to be reckoned with. For example, one such engine is a spring-like stalk connecting a single-celled organism called a vorticellid to the leaf fragment it calls home. When exposed to calcium, the spring contracts, yanking the vorticellid down at speeds approaching three inches (eight centimetres) per second. 

Springs like this are coiled bundles of filaments that expand or contract in response to chemical cues. A. wave of positively charged calcium ions, for example, neutralizes the negative charges that keep the filaments extended. Some sperm use spring-like engines made of actin filaments to shoot out a barb that penetrates the layers that surround an egg. And certain viruses use a similar apparatus to shoot their DNA. into the host’s cell. Ratchets are also useful for moving whole cells, including some other sperm and pathogens. These engines are filaments that simply grow at one end, attracting chemical building blocks from nearby. Because the other end is anchored in place, the growing end pushes against any barrier that gets in its way. 

Both springs and ratchets are made up of small units that each move just slightly, but collectively produce a powerful movement. Ultimately, Mahadevan and Matsudaira hope to better understand just how these particles create an effect that seems to be so much more than the sum of its parts. Might such an understanding provide inspiration for ways to power artificial nano-sized devices in the future? “The short answer is absolutely,” says Mahadevan. “Biology has had a lot more time to evolve enormous richness in design for different organisms. Hopefully, studying these structures will not only improve our understanding of the biological world, it will also enable us to copy them, take apart their components and recreate them for other purpose.” 

Q. 122 According to the author, research on the power source of movement in cells can contribute to 

A. control over the movement of genes within human systems. 

B. the understanding of nanotechnology. 

C. arresting the growth of cancer in a human being. 

D. the development of cures for a variety of diseases. 

Answer: D. 

Explanation: 

Statements A and B are not given as aims of research on the power source of movement in cells. Hence, we can eliminate the options. 

Refer to the lines:”Because these molecules are essential for cell division, scientists hope to shut down the rampant 

growth of cancer cells by deactivating certain motors. Improving motor-driven transport in nerve cells may also be helpful for treating diseases such as Alzheimer’s, Parkinson’s or ALS, also known as Lou Gehrig’s disease.” 

Between C and D, D is directly inferable from the information given above. C. requires the additional assumption that stopping rampant growth would be the same as stopping all growth.  

Q. 123 The author has used several analogies to illustrate his arguments in the article. Which of the following pairs of words are examples of the analogies used? 

A. Cell activity and vehicular traffic 

B. Polymers and tram tracks 

C. Genes and canoes 

D. Vorticellids and ratchets 

A. A and B 

B. B and C

C. A and D 

D. A and C 

Answer: A. 

Explanation: 

Refer to the lines of the 1st para where it is written that “the inside of the cell bustles with more traffic than Delhi Road”. 

Also refer to the middle of 3rd para “The movements that make these cellular activities possible occur along a complex network of threadlike fibers, or polymers, along which bundles of molecules travel like tram” 

Q. 124 Read the five statements below: A, B, C, D, and E. From the options given, select the one which includes a statement that is not representative of an argument presented in the passage. 

A. Sperms use spring like engines made of actin filament. 

B. Myosin and kinesin are unrelated. 

C. Nanotechnology researchers look for ways to power molecule-sized devices. 

D. Motor proteins help muscle contraction. 

E. The dynein motor is still poorly understood. 

A. A, B and C 

B. C, D and E 

C. A, D and E 

D. A, C and D 

Answer: A. 

Explanation: 

Refer to the 2nd last para ” Some sperm use spring like engines made of actin filaments to shoot out a barb that penetrates the layers that surround an egg.” This makes the argument A. correct. 

Refer to the 4th para “Each myosin molecule has a tiny head that pokes out from the chain like oars from a canoe. Just as rowers propel their boat by stroking their oars through the water, the myosin molecules stick their heads into the 

action and hoist themselves forward along the filament. While myosin moves along in short strokes, it cousin Kinesin walks steadily along a different type of filament called a microtubule. Instead of using a projecting head as lever, kinesin walks on two ‘legs’. Based on these differences, researchers used to think that myosin and kinesin were virtually unrelated. But newly discovered similarities in the motors’ ATP processing machinery now suggest that they share a common ancestor-molecule.” Hence, we can infer that Myosin and Kinesin are in fact related. Therefore, statement B. is False. 

Refer to the 1st line of 2nd para “The ability of such engines to convert chemical energy into motion is the envy of the nanotechnology researchers looking for ways to power molecule-sized devices.”This makes C correct. 

The third para states that “We wouldn’t make it far in life without motor proteins. Our muscles wouldn’t contract.” Hence, statement D is true. 

Statement E is directly given in para 4, line 2. 

Hence, only statement B is incorrect. Thus, the only option that includes a statement that is not representative of an argument presented in the passage is option A which includes statement B 

Q. 125 Read the four statements below: A, B, C and D From the options given, select the one which includes only statements that are representative of arguments presented in the passage. 

A. Protein motors help growth processes. 

B. Improved transport in nerve cells will help arrest tuberculosis and cancer. 

C. Cells, together, generate more power than the sum of power generated by them separately.

D. Vorticellid and the leaf fragment are connected by a calcium engine. 

A. A and B but not C 

B. A and C but not D 

C. A and D but not B 

D. C and D but not B 

Answer: B. 

Explanation: 

Refer 1st lines of 3rd para “We wouldn’t make it far in life without motor proteins. Our muscles wouldn’t contract. We couldn’t grow because the growth process requires cells to duplicate their machinery and pull the copies apart “This makes A correct. 

Refer to the last para”Both springs and ratchets are made up of small units that each move just slightly, but collectively produce a powerful movement. Ultimately, Mahadevan and Matsudaira hope to better understand just how these particles create an effect that seems to be so much more than the sum of its parts.” This makes C correct. 

Q. 126 Read the four statements below: A, B, C and D From the options given, select the one which includes statements that are representative of arguments presented in the passage. 

A. Myosin, kinesin and actin are three types of protein. 

B. Growth processes involve a routine in a cell that duplicates their machinery and pulls the copies apart. 

C. Myosin molecules can generate vibrations in muscles. 

D. Ronald and Mahadevan are researchers at Massachusetts Institute of Technology. 

A. A and B but not C and D 

B. B and C but not A 

C. B and D but not A and C 

D. A, B and C but not D 

Answer: A. 

Explanation: 

Refer to the 3rd para”The engines that power the cell’s freight are three families of proteins, called myosin, kinesin and dynein.” It is given that Actin is a protein in the sentence, “Myosin molecules, best known for their role in muscle contraction, form chains that lie between filaments of another protein called actin”. Hence, argument A is true. 

Statement B is given in para 3 in the third line. 

Statement C is not given – we know that Myosin causes contractions but not vibrations. 

Statement D is not true according to the paragraph. 

Instructions 

If translated into English, most of the ways economists talk among themselves would sound plausible enough to poets, journalists, businesspeople, and other thoughtful though non-economical folk. Like serious talk anywhere — among boat designers and baseball fans, say — the talk is hard to follow when one has not made a habit of listening to it for a while. The culture of the conversation makes the words arcane. But the people in the unfamiliar conversation are not Martians. Underneath it all (the economist’s favourite phrase) conversational habits are similar. Economics uses mathematical models and statistical tests and market arguments, all of which look alien to the literary eye. But looked at closely they are not so alien. They may be seen as figures of speech-metaphors, analogies, and appeals to authority. 

Figures of speech are not mere frills. They think for us. Someone who thinks of a market as an ‘invisible hand’ and the organization of work as a ‘production function’ and his coefficients as being ‘significant’, as an economist does, is giving the language a lot of responsibility. It seems a good idea to look hard at his language. 

If the economic conversation were found to depend a lot on its verbal forms, this would not mean that economics would be not a science, or just a matter of opinion, or some sort of confidence game. Good poets, though not scientists, are serious thinkers about symbols; good historians, though not scientists, are serious thinkers about data. Good scientists also use language. What is more (though it remains to be shown) they use the cunning of language, without particularly meaning to. The language used is a social object, and using language is a social act. It requires cunning (or, if you prefer, consideration), attention to the other minds present when one speaks. 

The paying of attention to one’s audience is called ‘rhetoric’, a word that I later exercise hard. One uses rhetoric, of course, to warn of a fire in a theatre or to arouse the xenophobia of the electorate. This sort of yelling is the vulgar meaning of the word, like the president’s ‘heated rhetoric’ in a press conference or the ‘mere rhetoric’ to which our enemies stoop. Since the Greek flame was lit, though, the word has been used also in a broader and more amiable 

sense, to mean the study of all the ways of accomplishing things with language: inciting a mob to lynch the accused, to be sure, but also persuading readers of a novel that its characters breathe, or bringing scholars to accept the better argument and reject the worse. 

The question is whether the scholar- who usually fancies himself an announcer of ‘results’ or a stater of ‘conclusions’ free of rhetoric — speaks rhetorically. Does he try to persuade? It would seem so. Language, I just said, is not a solitary accomplishment. The scholar doesn’t speak into the void, or to himself. He speaks to a community of voices. He desires to be heeded, praised, published, imitated, honoured, en-Nobeled. These are the desires. The devices of language are the means. Rhetoric is the proportioning of means to desires in speech. 

Rhetoric is an economics of language, the study of how scarce means are allocated to the insatiable desires of people to be heard. It seems on the face of it a reasonable hypothesis that economists are like other people in being talkers, who desire listeners whey they go to the library or the laboratory as much as when they go to the office or the polls. The purpose here is to see if this is true, and to see if it is useful: to study the rhetoric of economic scholarship. 

The subject is scholarship. It is not the economy, or the adequacy of economic theory as a description of the economy, or even mainly the economist’s role in the economy. The subject is the conversation economists have among themselves, for purposes of persuading each other that the interest elasticity of demand for investment is zero or that the money supply is controlled by the Federal Reserve. 

Unfortunately, though, the conclusions are of more than academic interest. The conversations of classicists or of astronomers rarely affect the lives of other people. Those of economists do so on a large scale. A. well known joke describes a May Day parade through Red Square with the usual mass of soldiers, guided missiles, rocket launchers. At last come rank upon rank of people in gray business suits. A. bystander asks, “Who are those?” “Aha!” comes the reply, 

”those are economists: you have no idea what damage they can do!” Their conversations do it. 

Q. 127 According to the passage, which of the following is the best set of reasons for which one needs to ‘look hard’ at an economist’s language? 

A. Economists accomplish a great deal through their language. 

B. Economics is an opinion-based subject. 

C. Economics has a great impact on others lives. 

D. Economics is damaging. 

A. A and B 

B. C and D 

C. A and C 

D. B and D 

Answer: C. 

Explanation: 

Option A and C are correct.Refer to the words”they think for us” and “giving a language a lot of responsibility” 

Q. 128 In the light of the definition of rhetoric given in the passage, which of the following will have the least element of rhetoric? 

A. An election speech 

B. An advertisement jingle 

C. Dialogues in a play 

D. Commands given by army officers 

Answer: D. 

Explanation: 

Option A,B and C clearly qualifies for the rhetoric as they persuade people to be heard.Option D does not fit the criteria. it is giving orders and not persuading people to be heard. 

Q. 129 As used in the passage, which of the following is the clolsest meaning to the statement ‘The cultural of the conversation makes the words arcane’? 

A. Economists belong to a different culture. 

B. Only mathematicians can understand economicsts. 

C. Economists tend to use terms unfamiliar to the lay person, but depend on familiar linguistic forms. 

D. Economists use similes and adjectives in their analysis. 

Answer: C. 

Explanation: 

Only option C correctly depicts the meaning of the statement.Rest of the options are deviating.”Arcane” means unfamiliar to the layman. Hence option c correctly depicts the meaning. 

Q. 130 As used in the passage, which of the following is the closest alternative to the word ‘arcane’? 

A. Mysterious 

B. Secret 

C. Covert 

D. Perfidious 

Answer: A. 

Explanation: 

The dictionary meaning of arcane is deceptive. 

Q. 131 Based on your understanding of the passage, which of the following conclusions would you agree with? 

A. The geocentric and the heliocentric views of the solar system are equally tenable. 

B. The heliocentric view is superior because of better rhetoric. 

C. Both views use rhetoric to persuade. 

D. Scientists should not use rhetoric. 

Answer: C. 

Explanation: 

If you refer these line:”Since the Greek flame was lit, though, the word has been used also in a broader and more amiable sense, to mean the study of all the ways of accomplishing things with language: inciting a mob to lynch the accused, to be sure, but also persuading readers of a novel that its characters breathe, or bringing scholars to accept the better argument and reject the worse.”, it suggests that both views persuade people through rhetoric. 

Instructions 

For the following questions answer them individually 

Q. 132 Out of the four possibilities given, select the one that has all the definitions and their usages most closely matched. 

“Measure” 

A. Size or quantity found by measuring 

B. Vessel of standard capacity 

C. Suitable action 

D. Ascertain extent or quantity 

E. A. measure was instituted to prevent outsiders from entering the campus 

F. Sheila was asked to measure each item that was delivered. 

G. The measure of the cricket pitch was 22 yards. 

H. Ramesh used a measure to take out one litre of oil. 

A. A-H, B-F, C-E, D-G 

B. A-G, B-E, C-F, D-H 

C. A-G, B-H, C-E, D-F 

D. A-F, B-H, C-E, D-G 

Answer: C. 

Explanation: 

In point E, a certain action was taken in order to prevent outsiders from entering the campus. (C-E) In point F, Sheila had to gauge the capacity. “Ascertain extent or quantity” makes a good fit. (D-F) In point G, the size of cricket pitch was gauged as 22 yards. (A-G) 

In point H, Ramesh used a vessel marked with 1 litre capacity to take out the oil. (B-H) 

Q. 133 Out of the four possibilities given, select the one that has all the definitions and their usages most closely matched. 

“Bound” 

A. Obliged, constrained 

B. Limiting value 

C. Move in a specified direction 

D. Destined or certain to be 

E. Dinesh felt bound to walk out when the discussion turned to kickbacks. 

F. Buffeted by contradictory forces he was bound to lose his mind. 

G. Vidya’s story strains the bounds of credulity. 

H. Bound for a career in law, Jyoti was reluctant to study Milton. 

A. A-F, B-H, C-G, D-E 

B. A-E, B-G, C-H, D-F 

C. A-E, B-H, C-F, D-G 

D. A-F, B-G, C-E, D-H 

Answer: B. 

Explanation: 

Point E says Dinesh had to leave walk out due to kickbacks. It was out of constrain by force of necessity. (A-E) 

Point F says it was certain that the contradictory forces would mess up his mind. (D-F) 

According to point G, Vidya’s story was not readily believable. (B-G) 

Point H stresses on Jyoti’s affinity towards a career in law, hence she was moving in a specific direction towards her goal. (C-H) 

Q. 134 Out of the four possibilities given, select the one that has all the definitions and their usages most closely matched. 

“Catch” 

A. Capture 

B. Grasp with senses of mind 

C. Deception 

D. Thing or person worth trapping 

E. All her friends agreed that Prasad was a good catch. 

F. The proposal sounds very good but where is the catch? 

G. Hussain tries to catch the spirit of India in this painting. 

H. Sorry, I couldn’t catch you. 

A. A-H, B-F, C-E, D-G 

B. A-F, B-G, C-E, D-H 

C. A-G, B-F, C-E, D-H 

D. A-G, B-H, C-F, D-E 

Answer: D. 

Explanation: 

Point E says, Prasad is a good catch, meaning a good person worth trapping. (D-E) is a clear match and hence option D. is the answer as it is the only option with (D-E) pair. For reference sake, we’ll look at the other points. Point F senses a bit of deception as the proposal was too good to not be deceptive. (C-F) 

In point G, Hussain tries to capture the spirit of India in his painting. (A-G) 

In point H, one person was not able to make out what the other person was trying to say. The word “grasp” fits in well. (B-H) 

Q. 135 Out of the four possibilities given, select the one that has all the definitions and their usages most closely matched. 

“Deal” 

A. Manage, attend to 

B. Stock, sell 

C. Give out to a number of people 

D. Be concerned with 

E. Dinesh insisted on dealing the cards. 

F. This contract deals with handmade cards. 

G. My brother deals in cards. 

H. I decided not to deal with handmade cards. 

A. A-F, B-E, C-G, D-H 

B. A-H, B-G, C-E, D-F 

C. A-F, B-H, C-G, D-E 

D. A-H, B-E, C-G, D-F 

Answer: B. 

Explanation: 

Point E says Dinesh wanted to give out the cards. So (C-E) is a pair. 

Point F implies that the contract is related to (concerned with) handmade cards. So, (D-F) is a pair. From the options, we can see that only B. has this combination. 

Q. 136 Out of the four possibilities given, select the one that has all the definitions and their usages most closely matched. 

“Turn” 

A. Give new direction to 

B. Send 

C. Change in form 

D. Opportunity coming successively for each person 

E. It was now his turn to be angry. 

F. Leena never turned away a beggar. 

G. Ashish asked Laxman to turn his face to the left. 

H. The old school building has been turned into a museum. 

A. A-H, B-E, C-F, D-G 

B. A-G, B-F, C-E, D-H 

C. A-G, B-E, C-F, D-H 

D. A-G, B-F, C-H, D-E 

Answer: D. 

Explanation: 

In point E, it was his chance to be angry, so E-D. is a pair. 

Point F means that Leena never sent the beggar away, so F-B. is a pair. 

In point G, Ashish asked Lakshman to change the direction of his face, so G-A. is a pair. 

In point H, the school changed its form to a museum, so H-C. is a pair. 

Option d) is the correct answer. 

Q. 137 The sentences given in each question, when properly sequenced, form a coherent paragraph. Each sentence is labelled with a letter. Choose the most logical order of sentences from among the given choices to construct a coherent paragraph. 

A. Branded disposable diapers are available at many supermarkets and drug stores. 

B. If one supermarket sets a higher price for a diaper, customers may buy that brand elsewhere. 

C. By contrast, the demand for private-label products may be less price sensitive since it is available only at a corresponding supermarket chain. 

D. So the demand for branded diapers at any particular store may be quite price sensitive. E. For instance, only SavOn Drugs stores sell SavOn Drugs diapers. 

F. Then stores should set a higher incremental margin percentage for private label diapers. 

A. ABCDEF 

B. ABCEDF 

C. ADBCEF 

D. AEDBCF 

Answer: C. 

Explanation: 

Among the statements we can see that the AD. is the pair as A starts the topic and D continues with saying that demand is price sensitive asbranded diapers are available at many stores. Also there is visible link between EF as E gives an example and F continues with saying what should be done if E happens. Hence correct sequence is ADBCEF. 

Q. 138 The sentences given in each question, when properly sequenced, form a coherent paragraph. Each sentence is labelled with a letter. Choose the most logical order of sentences from among the given choices to construct a coherent paragraph. 

A. Having a strategy is a matter of discipline. 

B. It involves the configuration of a tailored value chain that enables a company to offer unique value. 

C. It requires a strong focus on profitability and a willingness to make tough tradeoffs in choosing what not to do. 

D. Strategy goes far beyond the pursuit of best practices. 

E. A company must stay the course even during times of upheaval, while constantly improving and extending its distinctive positioning. 

F. When a company’s activities fit together as a self-reinforcing system, any competitor wishing to imitate a strategy must replicate the whole system. 

A. ACEDBF 

B. ACBDEF 

C. DCBEFA 

D. ABCEDF 

Answer: A. 

Explanation: 

A introduces the main idea of the para – having a strategy. 

C and E extend this idea in that order. So, ACE is a sequence. 

DB is a mandatory pair. The ‘it’ in sentence B. refers to ‘the strategy’ in sentence D 

F is the concluding sentence. 

So, the correct order of sentences is ACEDBF

Q. 139 The sentences given in each question, when properly sequenced, form a coherent paragraph. Each sentence is labelled with a letter. Choose the most logical order of sentences from among the given choices to construct a coherent paragraph. 

A. As officials, their vision of a country shouldn’t run too far beyond that of the local people with whom they have to deal. 

B. Ambassadors have to choose their words. 

C. To say what they feel they have to say, they appear to be denying or ignoring part of what they know. 

D. So, with ambassadors as with other expatriates in black Africa, there appears at a first meeting a kind of ambivalence. 

E. They do a specialized job and it is necessary for them to live ceremonial lives. 

A. BCEDA 

B. BEDAC 

C. BEADC 

D. BCDEA

E. BCEAD

Answer: C. 

Explanation: 

Statement B introduces the subject of the paragraph – “Ambassadors”. Hence, it should be the opening sentence of the paragraph. 

The main point of the paragraph is that Ambassadors need to carefully weigh their words so that their vision is acceptable to the locals of the country. 

Statement A says that the ambassador’s vision of the country should not run beyond that of the local people and D adds to this thought by saving as a result of this necessity there is an ambivalence in their behaviour. C explains how this ambivalence is exhibited by them. Hence, A-D-C are logically connected. 

Statement E which introduces the specialized nature of their job acts a connecting line between B and ADC Hence, the order is BEADC 

Q. 140 The sentences given in each question, when properly sequenced, form a coherent paragraph. Each sentence is labelled with a letter. Choose the most logical order of sentences from among the given choices to construct a coherent paragraph. 

A. “This face-off will continue for several months given the strong convictions on either side,” says a senior functionary of the high-powered task force on drought. 

B. During the past week-and-half, the Central Government has sought to deny some of the earlier apprehensions over the impact of drought. 

C. The recent revival of the rains had led to the emergence of a line of divide between the two. 

D. The state governments, on the other hand, allege that the Centre is downplaying the crisis only to evade its full responsibility of financial assistance that is required to alleviate the damage. 

E. Shrill alarm about the economic impact of an inadequate monsoon had been sounded by the Centre as well as most of the states, in late July and early August. 

A. EBCDA 

B. DBACE 

C. BDCAE 

D. ECBDA 

Answer: D. 

Explanation: 

E is the opening sentence since it introduces the topic. This is followed by sentence C. The ‘two’ in C. refers to the ‘Center and states’ mentioned in E. B-D. is a pair. B. talks about the actions of Central Government whereas D. contrasts it with the actions of the State Governments. A. is the concluding sentence. The correct order of sentences is ECBDA. 

Q. 141 The sentences given in each question, when properly sequenced, form a coherent paragraph. Each sentence is labelled with a letter. Choose the most logical order of sentences from among the given choices to construct a coherent paragraph. 

A. This fact was established in the 1730s by French survey expenditions to Equador near the Equator and Lapland in the Arctic, which found that around the middle of the earth the arc was about a kilometer shorter. 

B. One of the unsettled scientific questions in the late 18th century was that exact nature of the shape of the earth. 

C. The length of one-degree arc would be less near the equatorial latitudes than at the poles. 

D. One way of doing that is to determine the length of the arc along a chosen longitude or meridian at one degree latitude separation. 

E. While it was generally known that the earth was not a sphere but an ‘oblate spheroid’, more curved at the equator and flatter at the poles, the question of ‘how much more’ was yet to be established. 

A. BECAD 

B. BEDCA 

C. EDACB 

D. EBDCA 

Answer: B. 

Explanation: 

Statement B is the first sentence of the paragraph. It introduces the topic. This is followed by E, which dwells more on the ‘unsettled scientific questions’ talked about in B. Also there is link between statements E and D as D provides the solution to the question asked in E This is followed by sentence C and A is the closing sentence of the paragraph. 

Option b) is the correct answer. 

Q. 142 Choose the best way of writing the sentence. 

A. The main problem with the notion of price discrimination is that it is not always a bad thing, but that it is the monopolist who has the power to decide who is charged what price. 

B. The main problem with the notion of price discrimination is not that it is always a bad thing, it is the monopolist who has the power to decide who is charged what price. 

C. The main problem with the notion of price discrimination is not that it is always a bad thing, but that it is the monopolist who has the power to decide who is charged what price. 

D. The main problem with the notion of price discrimination is not it is always a bad thing, but that it is the monopolist who has the power to decide who is charged what price. 

A.

B.

C.

D.

Answer: C. 

Explanation: 

The idiom used in these sentences is “not __, but __”. So, the two phrases that follow ‘not’ and ‘but’ must be parallel. 

In sentence A, the phrase after ‘but’ starts with the subject but the phrase after ‘not’ doesn’t. Hence, sentence A. has a parallelism error. There are no other errors in sentence A. Option A. is wrong. 

All the options that have the same error can be eliminated. In sentence B, “not that it is” and “but it is” are not parallel. Hence, option B. is wrong. 

Similarly, option D. is also wrong because the phrases “not it is” and “but that it is” are not parallel. 

This error is corrected in option C. “Not that it is” and “but that it is” are parallel and hence option C. is the correct answer. 

Q. 143 Choose the best way of writing the sentence. 

A. A symbiotic relationship develops among the contractors, bureaucracy and the politicians, and by a large number of devices costs are artificially escalated and black money is generated by underhand deals. 

B. A symbiotic relationship develops among contractors, bureaucracy and politicians, and costs are artificially escalated with a large number of devices and black money is generated through underhand deals. 

C. A symbiotic relationship develops among contractors, bureaucracy and the politicians, and by a large number of devices costs are artificially escalated and black money is generated on underhand deals. 

D. A symbiotic relationship develops among the contractors, bureaucracy and politicians, and by large number of devices costs are artificially escalated and black money is generated by underhand deals. 

A.

B.

C.

D.

Answer: B. 

Explanation: 

Contractors, bureaucracy and politicians are being talked about for the first time in the sentence. So, the definite article ‘the’ should not be placed before any of them. Option b) is the grammatically correct sentence. 

Q. 144 Choose the best way of writing the sentence. 

A. The distinctive feature of tariffs and export subsidies is that they create differences in prices at which goods are traded on the world market and their price within a local market. 

B. The distinctive feature of tariffs and export subsidies is that they create a difference of prices at which goods are traded with the world market and their prices in the local market. 

C. The distinctive feature of tariffs and export subsidies is that they create a difference between prices at which goods are traded on the world market and their prices within a local market. 

D. The distinctive feature of tariffs and export subsidies is that they create a difference across prices at which goods are traded with the world market and their prices within a local market. 

A.

B.

C.

D.

Answer: C. 

Explanation: 

There are two prices that are being compared – prices at which goods are traded on the world market and prices at which goods are sold in the local market. So, the difference is created ‘between’ the two prices. Also, goods are traded ‘on’ the world market and not ‘with’ the world market. So, the grammatically correct sentence is option c). 

Q. 145 Choose the best way of writing the sentence. 

A. Any action of government to reduce the systemic risk inherent in financial markets will also reduce the risks that private operators perceive and thereby encourage excessive hedging. 

B. Any action by government to reduce the systemic risk inherent in financial markets will also reduce the risks that private operators perceive and thereby encourage excessive gambling. 

C. Any action by government to reduce the systemic risk inherent in financial markets will also reduce the risks that private operators perceive and thereby encourages excessive gambling. 

D. Any action of government to reduce the systemic risk inherent in financial markets will also reduce the risks that private operators perceive and thereby encourages excessive gambling. 

A.

B.

C.

D.

Answer: B. 

Explanation: 

The correct preposition to be used before government is ‘by’ and not ‘of’. So, options a) and d) are incorrect. 

In the second part of the sentence, which is in simple future tense, the grammatically correct form is “thereby encourage” and not “thereby encourages”. So, option b) is the correct answer. 

Q. 146 From the alternatives given pick the word or phrase that is closest in meaning in the given context. 

Opprobrium: The police officer appears oblivious to the opprobrium generated by his blatantly partisan conduct. 

A. Harsh criticism 

B. Acute distrust 

C. Bitter enmity 

D. Stark oppressiveness 

Answer: A. 

Explanation: 

The meaning of ‘opprobrium’ is ‘criticize scornfully’. 

Only option a) brings out this meaning. None of the other options are applicable. 

Q. 147 From the alternatives given pick the word or phrase that is closest in meaning in the given context. Portend: It appears to many that the US ‘war on terrorism’ portends trouble in the Gulf. 

A. Introduces 

B. Bodes 

C. Spells 

D. Evokes 

Answer: C. 

Explanation: 

In the given context, “to portend” is ‘to mean’ something. It doesn’t mean ‘to start’ something, hence options A and D can be eliminated. Between “bodes” and “spells”, “spells” fits better in this context. Option c) is the correct answer. 

Q. 148 From the alternatives given pick the word or phrase that is closest in meaning in the given context. 

Prevaricate: When a videotape of her meeting was played back to her and she was asked to explain her presence there, she started prevaricating. 

A. Speaking evasively 

B. Speaking violently 

C. Lying furiously 

D. Throwing a tatrum 

Answer: A. 

Explanation: 

Option A. is correct. 

‘To prevaricate’ is ‘to mislead deliberately’. In other words, it is to speak evasively. 

Other options talk about furiousness and violence, which are wrong in this context. 

Q. 149 From the alternatives given pick the word or phrase that is closest in meaning in the given context. 

Restive: The crowd became restive when the minister failed to appear even by 10 pm. 

A. Violent 

B. Angry 

C. Restless 

D. Distressed 

Answer: C. 

Explanation: 

The correct option is ‘restless’. 

‘Restive’ means impatient or restless and doesn’t imply the presence of anger and violence. So, option c) is the correct answer. 

Q. 150 From the alternatives given pick the word or phrase that is closest in meaning in the given context. Ostensible: Manohar’s ostensible job was to guard the building at night. 

A. Apparent 

B. Blatant 

C. Ostentatious 

D. Insidious 

Answer: A. 

Explanation: 

The correct option is “apparent”. 

The meaning of the word ‘ostensible’ is ‘clearly evident’ and does not hint any kind of pretense or deceit. So, the word ‘apparent’ is the suitable word in this context. 

Option a) is the correct answer. 

CAT Previous Year Paper 2001

CAT 2001

Q. 1 A student took five papers in an examination, where the full marks were the same for each paper. His marks in these papers were in the proportion of 6 : 7 : 8 : 9 : 10. In all papers together, the candidate obtained 60% of the total marks. Then the number of papers in which he got more than 50% marks is

A. 2

B. 3

C. 4

D. 5

 

Q. 2 A square, whose side is 2 m, has its corners cut away so as to form an octagon with all sides equal. Then the length of each side of the octagon, in metres, is

A. √2/√2+1

B. 2/√2+1

C. 2/√2-1

D. √2/√2-1

 

Q. 3 Let x, y and z be distinct integers. x and y are odd and positive, and z is even and positive. Which one of the following statements cannot be true?

A. y(x – z)² is even

B. y²(x – z) is odd

C. y(x – z) is odd

D. z(x – y)² is even

 

Q. 4 If x > 5 and y < –1, then which of the following statements is true?

A. (x + 4y) > 1

B. x > –4y

C. –4x < 5y

D. None of these

 

Q. 5 A red light flashes three times per minute and a green light flashes five times in 2 min at regular intervals. If both lights start flashing at the same time, how many times do they flash together in each hour?

A. 30

B. 24

C. 20

D. 60

 

Q. 6 Of 128 boxes of oranges, each box contains at least 120 and at most 144 oranges. The number of boxes containing the same number of oranges is at least

A. 5

B. 103

C. 6

D. Cannot be determined

 

Q. 7 A certain city has a circular wall around it, and this wall has four gates pointing north, south, east and west. A house stands outside the city, 3 km north of the north gate, and it can just be seen from a point 9 km east of the south gate. What is the diameter of the wall that surrounds the city?

A. 6 km

B. 9 km

C. 12 km

D. none of these

 

Q. 8 In the above diagram, ABCD is a rectangle with AE = EF = FB. What is the ratio of the areas of ΔCEF and that of the rectangle?

A. 1/6

B. 1/8

C. 1/9

D. None of these

 

Q. 9 A can complete a piece of work in 4 days. B takes double the time taken by A, C takes double that of B, and D takes double that of C to complete the same task. They are paired in groups of two each. One pair takes two-thirds the time needed by the second pair to complete the work. Which is the first pair?

A. A and B

B. A and C

C. B and C

D. A and D

 

Q. 10 In a four-digit number, the sum of the first 2 digits is equal to that of the last 2 digits. The sum of the first and last digits is equal to the third digit. Finally, the sum of the second and fourth digits is twice the sum of the other 2 digits. What is the third digit of the number?

A. 5

B. 8

C. 1

D. 4

 

Q. 11 Two men X and Y started working for a certain company at similar jobs on January 1, 1950. X asked for an initial salary of Rs. 300 with an annual increment of Rs. 30. Y asked for an initial salary of Rs. 200 with a rise of Rs. 15 every 6 months. Assume that the arrangements remained unaltered till December 31, 1959. Salary is paid on the last day of the month. What is the total amount paid to them as salary during the period?

A. Rs. 93,300

B. Rs. 93,200

C. Rs. 93,100

D. None of these

 

Q. 12 Anita had to do a multiplication. In stead of taking 35 as one of the multipliers, she took 53. As a result, the product went up by 540. What is the new product?

A. 1050

B. 540

C. 1440

D. 1590

 

Q. 13 A college has raised 75% of the amount it needs for a new building by receiving an average donation of Rs. 600 from the people already solicited. The people already solicited represent 60% of the people the college will ask for donations. If the college is to raise exactly the amount needed for the new building, what should be the average donation from the remaining people to be solicited?

A. Rs. 300

B. Rs. 250

C. Rs. 400

D. Rs. 500

 

Q. 14 x and y are real numbers satisfying the conditions 2 < x < 3 and – 8 < y < –7. Which of the following expressions will have the least value?

A. x²y

B. xy²

C. 5xy

D. None of these

 

Q. 15 m is the smallest positive integer such that for any integer n ≥ m , the quantity n³ – 7n² + 11n – 5 is positive. What is the value of m?

A. 4

B. 5

C. 8

D. None of these

 

Q. 16 A ladder leans against a vertical wall. The top of the ladder is 8 m above the ground. When the bottom of the ladder is moved 2 m farther away from the wall, the top of the ladder rests against the foot of the wall. What is the length of the ladder?

A. 10m

B. 15m

C. 20m

D. 17m

 

Q. 17 Three friends, returning from a movie, stopped to eat at a restaurant. After dinner, they paid their bill and noticed a bowl of mints at the front counter. Sita took one-third of the mints, but returned four because she had a momentary pang of guilt. Fatima then took one-fourth of what was left but returned three for similar reason. Eswari then took half of the remainder but threw two back into the bowl. The bowl had only 17 mints left when the raid was over. How many mints were originally in the bowl?

A. 38

B. 31

C. 41

D. none of these

 

Q. 18 If 09/12/2001(DD/MM/YYYY) happens to be Sunday, then 09/12/1971 would have been a

A. Wednesday

B. Tuesday

C. Saturday

D. Thursday

 

Q. 19 In a number system, the product of 44 and 11 is 3414. The number 3111 of this system, when converted to the decimal number system, becomes

A. 406

B. 1086

C. 213

D. 691

 

Q. 20 At his usual rowing rate, Rahul can travel 12 miles downstream in a certain river in 6 hr less than it takes him to travel the same distance upstream. But if he could double his usual rowing rate for this 24 miles round trip, the downstream 12 miles would then take only 1 hr less than the upstream 12 miles. What is the speed of the current in miles per hour?

A. 7/3

B. 4/3

C. 5/3

D. 8/3

 

Q. 21 Every 10 years the Indian Government counts all the people living in the country. Suppose that the director of the census has reported the following data on two neighbouring villages Chota Hazri and Mota Hazri.

Chota Hazri has 4,522 fewer males than Mota Hazri.

Mota Hazri has 4,020 more females than males.

Chota Hazri has twice as many females as males.

Chota Hazri has 2,910 fewer females than Mota Hazri.

What is the total number of males in Chota Hazri?

A. 11264

B. 14174

C. 5632

D. 10154

 

Q. 22 Three classes X, Y and Z take an algebra test.

The average score in class X is 83.

The average score in class Y is 76.

The average score in class Z is 85.

The average score of all students in classes X and Y together is 79.

The average score of all students in classes Y and Z together is 81.

What is the average for all the three classes?

A. 81

B. 81.5

C. 82

D. 84.5

 

Q. 23 Two sides of a plot measure 32 m and 24 m and the angle between them is a perfect right angle. The other two sides measure 25 m each and the other three angles are not right angles.What is the area of the plot?

A. 768 m²

B. 534 m²

C. 696.5 m²

D. 684 m²

 

Q. 24 All the page numbers from a book are added, beginning at page 1. However, one page number was added twice by mistake. The sum obtained was 1000. Which page number was added twice?

A. 44

B. 45

C. 10

D. 12

 

Q. 25 Shyama and Vyom walk up an escalator (moving stairway). The escalator moves at a constant speed. Shyama takes three steps for every two of Vyom’s steps. Shyama gets to the top of the escalator after having taken 25 steps, while Vyom (because his slower pace lets the escalator do a little more of the work) takes only 20 steps to reach the top. If the escalator were turned off, how many steps would they have to take to walk up?

A. 40

B. 50

C. 60

D. 80

 

Q. 26 At a certain fast food restaurant, Brian can buy 3 burgers, 7 shakes, and one order of fries for Rs. 120 exactly. At the same place it would cost Rs. 164.5 for 4 burgers, 10 shakes, and one order of fries. How much would it cost for an ordinary meal of one burger, one shake, and one order of fries?

A. Rs. 31

B. Rs. 41

C. Rs. 21

D. Cannot be determined

 

Q. 27 If a, b, c and d are four positive real numbers such that abcd = 1, what is the minimum value of (1 + a)(1 + b)(1 + c)(1 + d)?

A. 4

B. 1

C. 16

D. 18

 

Q. 28 There’s a lot of work in preparing a birthday dinner. Even after the turkey is in the oven, there’s still the potatoes and gravy, yams, salad, and cranberries, not to mention setting the table. Three friends — Asit, Arnold and Afzal — work together to get all of these chores done. The time it takes them to do the work together is 6 hr less than Asit would have taken working alone, 1 hr less than Arnold would have taken alone, and half the time Afzal would have taken working alone. How long did it take them to do these chores working together?

A. 20 min

B. 30 min

C. 40 min

D. 50 min

 

Q. 29 Euclid has a triangle in mind. Its longest side has length 20 and another of its sides has length 10. Its area is 80. What is the exact length of its third side?

A. √260

B. √250

C. √240

D. √270

 

Q. 30 For a Fibonacci sequence, from the third term onwards, each term in the sequence is the sum of the previous two terms in that sequence. If the difference in squares of 7th and 6th terms of this sequence is 517, what is the 10th term of this sequence?

A. 147

B. 76

C. 123

D. Cannot be determined

 

Q. 31 Fresh grapes contain 90% water by weight while dried grapes contain 20% water by weight. What is the weight of dry grapes available from 20 kg of fresh grapes?

A. 2 kg

B. 2.4 kg

C. 2.5 kg

D. None of these

 

Q. 32 Train X departs from station A at 11 a.m. for station B, which is 180 km so far. Train Y departs from station B at 11 a.m. for station A. Train X travels at an average speed of 70 km/hr and does not stop anywhere until it arrives at station B. Train Y travels at an average speed of 50 km/hr, but has to stop for 15 min at station C, which is 60 km away from station B enroute to station A. Ignoring the lengths of the trains, what is the distance, to the nearest kilometre, from station A to the point where the trains cross each other?

A. 112 km

B. 118 km

C. 120 km

D. none of these

 

Q. 33 A set of consecutive positive integers beginning with 1 is written on the blackboard. A student came along and erased one number. The average of the remaining numbers is 35 7/17 . What was thenumber erased?

A. 7

B. 8

C. 9

D. none of these

 

Q. 34 In DDEF shown below, points A, B and C are taken on DE, DF and EF respectively such that EC = AC and CF = BC. If ∠D =40° , then ∠ACB =

A. 140

B. 70

C. 100

D. none of these

 

Q. 35 The owner of an art shop conducts his business in the following manner: every once in a while he raises his prices by X%, then a while later he reduces all the new prices by X%. After one such updown cycle, the price of a painting decreased by Rs. 441. After a second updown cycle the painting was sold for Rs. 1,944.81. What was the original price of the painting?

A. Rs. 2,756.25

B. Rs. 2,256.25

C. Rs. 2,500

D. Rs. 2,000

 

Q. 36 Three runners A, B and C run a race, with runner A finishing 12 m ahead of runner B and 18 m ahead of runner C, while runner B finishes 8 m ahead of runner C. Each runner travels the entire distance at a constant speed. What was the length of the race?

A. 36 m

B. 48 m

C. 60 m

D. 72 m

 

Q. 37 Let x and y be two positive numbers such that x + y = 1.Then the minimum value of (x+1/x)²+ (y+1/y)² is

A. 12

B. 20

C. 12.5

D. 13.3

 

Questions: 38 – 39

Answer the questions based on the following information.The batting average (BA) of a Test batsman is computed from runs scored and innings played — completed innings and incomplete innings (not out) in the following manner:

r1 = Number of runs scored in completed innings

n1 = Number of completed innings

r2 = Number of runs scored in incomplete innings

n2 = Number of incomplete innings

BA=(r1+r2)/n1

To better assess a batsman’s accomplishments, the ICC is considering two other measures MBA1 and MBA2 defined as follows:

MBA1=r1/n1+n2/n1 max[0,(r2/n2-r1/n1)]

MBA2=r1+r2/n1+n2

 

Q. 38 Based on the above information which of the following is true?

A. MBA1 ≤ BA ≤ MBA2

B. BA ≤ MBA2 ≤ MBA1

C. MBA2 ≤ BA ≤ MBA1

D. none of these

 

Q. 39 An experienced cricketer with no incomplete innings has BA of 50. The next time he bats, the innings is incomplete and he scores 45 runs. It can be inferred that

A. BA and MBA1 will both increase

B. BA will increase and MBA2 will decrease

C. BA will increase and not enough data is available to assess change in MBA1 and MBA2

D. None of these

 

Q. 40 Based on the figure below, what is the value of x, if y = 10?

A. 10

B. 11

C. 12

D. none of these

 

Q. 41 A rectangular pool 20 m wide and 60 m long is surrounded by a walkway of uniform width. If the total area of the walkway is 2 516 m , how wide, in metres, is the walkway?

A. 43 m

B. 4.3 m

C. 3 m

D. 3.5 m

 

Q. 42 Let b be a positive integer and a = b² – b. If b ≥ 4 , then a² – 2a is divisible by

A. 15

B. 20

C. 24

D. all of these

 

Q. 43 Ashish is given Rs. 158 in one-rupee denominations. He has been asked to allocate them into a number of bags such that any amount required between Re 1 and Rs. 158 can be given by handing out a certain number of bags without opening them. What is the minimum number of bags required?

A. 11

B. 12

C. 13

D. none of these

 

Q. 44 In some code, letters a, b, c, d and e represent numbers 2, 4, 5, 6 and 10. We just do not know which letter represents which number. Consider the following relationships:

I. a + c = e, II. b – d = d and III. e + a = b Which of the following statements is true?

A. b = 4, d = 2

B. a = 4, e = 6

C. b = 6, e = 2

D. a = 4, c = 6

 

Q. 45 Ujakar and Keshab attempted to solve a quadratic equation. Ujakar made a mistake in writing down the constant term. He ended up with the roots (4, 3). Keshab made a mistake in writing down the coefficient of x. He got the roots as (3, 2). What will be the exact roots of the original quadratic equation?

A. (6, 1)

B. (–3, –4)

C. (4, 3)

D. (–4, –3)

 

Q. 46 A change-making machine contains one-rupee, two-rupee and five-rupee coins. The total number of coins is 300. The amount is Rs. 960. If the numbers of one-rupee coins and two rupee coins are interchanged, the value comes down by Rs. 40. The total number of five rupee coins is

A. 100

B. 140

C. 60

D. 150

 

Q. 47 The figure below shows the network connecting cities A, B, C, D, E and F. The arrows indicate permissible direction of travel. What is the number of distinct paths from A to F?

A. 9

B. 10

C. 11

D. none of these

 

Q. 48 Let n be the number of different five-digit numbers, divisible by 4 with the digits 1, 2, 3, 4, 5 and 6, no digit being repeated in the numbers. What is the value of n?

A. 144

B. 168

C. 192

D. none of these

 

Questions: 49 – 50

Answer the questions based on the following information. The petrol consumption rate of a new model car ‘Palto’ depends on its speed and may be described by the

graph below.

 

Q. 49 Manasa makes a 200 km trip from Mumbai to Pune at a steady speed of 60 km/hr. What is the volume of petrol consumed for the journey?

A. 12.5 L

B. 13.33 L

C. 16 L

D. 19.75 L

 

Q. 50 Manasa would like to minimize the fuel consumption for the trip by driving at the appropriate speed. How should she change the speed?

A. Increase the speed

B. Decrease the speed

C. Maintain the speed at 60 km/hr

D. Cannot be determined

 

Questions: 51 – 55

Answer the questions based on the following information. For the word given at the top of each table, match the dictionary definitions on the left (A, B, C, D) with their corresponding usage on the right (E, F, G, H). Out of the four possibilities given in the boxes below the table, select the one that has all the definitions and their usages correctly matched.

 

Q. 51 choose the correct answer from the figure 1

A. a

B. b

C. c

D. d

 

Q. 52 choose the correct answer from the figure 2

A. a

B. b

C. c

D. d

 

Q. 53 choose the correct answer from the figure 3

A. a

B. b

C. c

D. d

 

Q. 54 choose the correct answer from the figure 4

A. a

B. b

C. c

D. d

 

Q. 55 choose the correct answer from the figure 5

A. a

B. b

C. c

D. d

 

Questions: 56 – 60

The sentences given in each question, when properly sequenced, form a coherent paragraph. Each sentence is labelled with a letter. Choose the most logical order of sentences from among the given choices to construct a coherent paragraph.

 

Q. 56 A. Although there are large regional variations, it is not infrequent to find a large number of people sitting here and there and doing nothing.

B. Once in office, they receive friends and relatives who feel free to call any time without prior appointment.

C. While working, one is struck by the slow and clumsy actions and reactions, indifferent attitudes, procedure rather than outcome orientation, and the lack of consideration for others.

D. Even those who are employed often come late to the office and leave early unless they are forced to be punctual.

E. Work is not intrinsically valued in India.

F. Quite often people visit ailing friends and relatives or go out of their way to help them in their personal matters even during office hours.

A. ECADBF

B. EADCFB

C. EADBFC

D. ABFCBE

 

Q. 57 A. But in the industrial era destroying the enemy’s productive capacity means bombing the factories which are located in the cities.

B. So in the agrarian era, if you need to destroy the enemy’s productive capacity, what you want to do is burn his fields, or if you’re really vicious, salt them.

C. Now in the information era, destroying the enemy’s productive capacity means

destroying the information infrastructure.

D. How do you do battle with your enemy?

E. The idea is to destroy the enemy’s productive capacity, and depending upon the economic foundation, that productive capacity is different in each case.

F. With regard to defence, the purpose of the military is to defend the nation and be prepared to do battle with its enemy

A. FDEBAC

B. FCABED

C. DEBACF

D. DFEBAC

 

Q. 58 A. Michael Hofman, a poet and translator, accepts this sorry fact without approval or complaint.

B. But thanklessness and impossibility do not daunt him.

C. He acknowledges too — in fact, he returns to the point often — that best translators of poetry always fail at some level.

D. Hofman feels passionately about his work and this is clear from his writings.

E. In terms of the gap between worth and rewards, translators come somewhere near nurses and street-cleaners

A. EACDB

B. ADEBC

C. EACBD

D. DCEAB

 

Q. 59 A. Passivity is not, of course, universal.

B. In areas where there are no lords or laws, or in frontier zones where all men go armed, the attitude of the peasantry may well be different.

C. So indeed it may be on the fringe of the unsubmissive.

D. However, for most of the soil-bound peasants the problem is not whether to be normally passive or active, but when to pass from one state to another.

E.This depends on an assessment of the political situation

A. BEDAC

B. CDABE

C. EDBAC

D. ABCDE

 

Q. 60 A. The situations in which violence occurs and the nature of that violence tends to be clearly defined at least in theory, as in the proverbial Irishman’s question: “Is this a private fight or can anyone join in?”

B. So the actual risk to outsiders, though no doubt higher than our societies, is calculable.

C. Probably the only uncontrolled applications of force are those of social superiors to social inferiors and even here there are probably some rules.

D. However, binding the obligation to kill, members of feuding families engaged in mutual massacre will be genuinely appalled if by some mischance a bystander or outsider is killed.

A. DABC

B. ACDB

C. CBAD

D. DBAC

 

Questions: 61 – 65

In each of the following sentences, parts of the sentence are left blank. Beneath each sentence, four different ways of completing the sentence are indicated. Choose the best alternative from among the four.

 

Q. 61 But ___ are now regularly written not just for tools, but well-established practices, organisations and institutions, not all of which seem to be ___ away

A. reports … withering

B. stories … trading

C. books … dying

D. obituaries … fading

 

Q. 62 The Darwin who ___ is most remarkable for the way in which he ___ the attributes of the world class thinker and head of the household

A. comes … figures

B. arises … adds

C. emerges … combines

D. appeared … combines

 

Q. 63 Since her face was free of ___ there was no way to ___ if she appreciated what had happened

A. make-up … realise

B. expression … ascertain

C. emotion … diagnose

D. scars … understand

 

Q. 64 In this context, the ___ of the British labour movement is particularly ___.

A. affair … weird

B. activity … moving

C. experience … significant

D. atmosphere … gloomy

 

Q. 65 Indian intellectuals may boast, if they are so inclined, of being ___ to the most elitist among the intellectual ___ of the world

A. subordinate … traditions

B. heirs … cliques

C. ancestors … societies

D. heir … traditions

 

Questions: 66 – 70

For each of the words below, a contextual usage is provided. Pick the word from the alternatives given that is most inappropriate in the given context.

 

Q. 66 Specious: A specious argument is not simply a false one but one that has the ring of truth.

A. Deceitful

B. Fallacious

C. Credible

D. Deceptive

 

Q. 67 Obviate: The new mass transit system may obviate the need for the use of personal cars

A. prevent

B. Forestall

C. Preclude

D. Bolster

 

Q. 68 Disuse: Some words fall into disuse as technology makes objects obsolete.

A. Prevalent

B. Discarded

C. Obliterated

D. Unfashionable

 

Q. 69 Parsimonious: The evidence was constructed from very parsimonious scraps of information

A. Frugal

B. Penurious

C. Thrifty

D. Altruistic

 

Q. 70  Facetious: When I suggested that war is a method of controlling population, my father remarked that I was being facetious.

A. Jovian

B. Jovial

C. Jocular

D. Joking

 

Questions: 71 – 75

The Union Government’s present position vis-a-vis the upcoming United Nations conference on racial and related discrimination world-wide seems to be the following: discuss race please, not caste; caste is our very own and not at all as bad as you think. The gross hypocrisy of that position has been lucidly underscored by Kancha Ilaiah. Explicitly, the world community is to be cheated out of considering the matter on the technicality that caste is not, as a concept, tantamount to a racial category. Internally, however, allowing the issue to be put  on agenda at the said conference would, we are patriotically admonished, damage the country’s image. Somehow, India’s virtual beliefs elbow out concrete actualities. Inverted representations, as we know, have often been deployed in human histories as balm for the forsaken — religion being the most persistent of such inversions. Yet, we would humbly submit that if globalising our markets is thought as good for the ‘national’ pocket, globalising our social inequities might not be so bad for the mass of our people. After all, racism was as uniquely institutionalised in South Africa as caste discrimination has been within our society; why then can’t we permit the world community to express itself on the latter with a fraction of the zeal with which, through the years, we pronounced on the former?

As to the technicality about whether or not caste is admissible into an agenda about race (that the conference is also about ‘related discriminations’ tends to be forgotten), a reputed sociologist has recently argued that where race is a ‘biological’ category caste is a ‘social’ one. Having earlier fiercely opposed implementation of the Mandal Commission Report, the said sociologist is at least to be complemented now for admitting, however tangentially, that caste discrimination is a reality, although, in his view, incompatible with racial discrimination. One would like quickly to offer the hypothesis that biology, in important ways that affect the lives of many millions, is in itself perhaps a social construction. But let us look at the matter in another way. If it is agreed — as per the position today at which anthropological and allied scientific determinations rest — that the entire race of homo sapiens derived from an originary black African female (called ‘Eve’), then one is hard put to understand how, one some subsequent ground, ontological distinctions are to be drawn either between races or castes. Let us also underline the distinction between the supposition that we are all god’s children and the rather more substantiated argument about our descent from ‘Eve’, lest both positions are thought to be equally diversionary. It then stands to reason that all subsequent distinctions are, in modern parlance, ‘constructed’ ones, and like all ideological constructions, attributable to changing equations between knowledge and power among human communities through contested histories here, there, and elsewhere. This line of thought receives, thankfully, extremely consequential buttress from the findings of the Human Genome project. Contrary to earlier (chiefly 19th-century colonial) persuasions on the subject of race, as well as, one might add, the somewhat infamous Jensen offerings in the 20th century from America, those finding deny genetic difference between ‘races’. If anything, they suggest that environmental factors impinge on gene-function, as a dialectic seems to unfold between nature and culture. It would thus seem that ‘biology’ as the constitution of pigmentation enters the picture first only as a part of that dialectic. Taken together, the originary mother stipulation and the Genome findings ought indeed to furnish ground for human equality across the board, as well as yield policy initiatives towards equitable material dispensations aimed at building a global order where, in Hegel’s stirring formulation, only the rational constitutes the right. Such, sadly, is not the case as everyday fresh arbitrary grounds for discrimination are constructed in the interests of sectional dominance.

 

Q. 71 When the author writes ‘globalising our social inequities’, the reference is to

A. going beyond an internal deliberation on social inequity

B. dealing with internal poverty through the economic benefits of globalisation.

C. going beyond an internal delimitation of social inequity

D. achieving disadvantaged people’s empowerment, globally

 

Q. 72 achieving disadvantaged people’s empowerment, globally

A. s good for the forsaken and often deployed in human histories.

B. is good for the forsaken, but not often deployed historically for the oppressed.

C. occurs often as a means of keeping people oppressed

D. occurs often to invert the status quo

 

Q. 73 Based on the passage, which broad areas unambiguously fall under the purview of the UN conference being discussed?

A. Racial prejudice

B. Racial pride

C. Discrimination, racial or otherwise

D. Caste-related discrimination

E. Race-related discrimination

A. A and E

B. C and E

C. A, C and E

D. B, C and D

 

Q. 74 According to the author, the sociologist who argued that race is a ‘biological’ category and caste is a ‘social’ one,

A. generally shares the same orientation as the author’s on many of the central issues discussed

B. tangentially admits to the existence of ‘caste’ as a category

C. admits the incompatibility between the people of different race and caste

D. admits indirectly that both caste-based prejudice and racial discrimination exist.

 

Q. 75 An important message in the passage, if one accepts a dialectic between nature and culture, is that

A. the results of the Human Genome Project reinforces racial differences.

B. race is at least partially a social construct.

C. discrimination is at least partially a social construct.

D. caste is at least partially a social construct.

 

Questions: 76 – 80

Studies of the factors governing reading development in young children have achieved a remarkable degree of consensus over the past two decades. The consensus concerns the causal role of ‘phonological skills in young children’s reading progress. Children who have good phonological skills, or good ‘phonological awareness’ become good readers and good spellers. Children with poor phonological skills progress more poorly. In particular, those who have a specific phonological deficit are likely to be classified as dyslexic by the time that they are 9 or 10 years old. Phonological skills in young children can be measured at a number of different levels. The term phonological awareness is a global one, and refers to a deficit in recognising smaller units of sound within spoken words. Development work has shown that this deficit can be at the level of syllables, of onsets and rimes, or phonemes. For example, a 4-year old child might have difficulty in recognising that a word like valentine has three syllables, suggesting a lack of syllabic awareness. A five-year-old might have difficulty in recognising that the odd work out in the set of words fan, cat, hat, mat is fan. This task requires an awareness of the sub-syllabic units of the onset and the rime. The onset corresponds to any initial consonants in a syllable words, and the rime corresponds to the vowel and to any following consonants. Rimes correspond to rhyme in single-syllable words, and so the rime in fan differs from the rime in cat, hat and mat. In longer words, rime and rhyme may differ. The onsets in val:en:tine are /v/ and /t/, and the rimes correspond to the selling patterns ‘al’, ‘en’ and’ ine’. A six-year-old might have difficulty in recognising that plea and pray begin with the same initial sound. This is a phonemic judgement. Although the initial phoneme /p/ is shared between the two words, in plea it is part of the onset ‘pl’ and in pray it is part if the onset ‘pr’. Until children can segment the onset (or the rime), such phonemic judgements are difficult for them to make. In fact, a recent survey of different developmental studies has shown that the different levels of phonological awareness appear to emerge sequentially. The awareness of syllables, onsets, and rimes appears to merge at around the ages of 3 and 4, long before most children go to school. The awareness of phonemes, on the other hand, usually emerges at around the age of 5 or 6, when children have been taught to read for about a year. An awareness of onsets and rimes thus appears to be a precursor of reading, whereas an awareness of phonemes at every serial position in a word only appears to develop as reading is taught. The onset-rime and phonemic levels of phonological structure, however, are not distinct. Many onsets in English are single phonemes, and so are some rimes (e.g. sea, go, zoo). The early availability of onsets and rimes is supported by studies that have compared the development of phonological awareness of onsets, rimes, and phonemes in the same subjects using the same phonological awareness tasks. For example, a study by Treiman and Zudowski used a same/different judgement task based on the beginning or the end sounds of words. In the beginning sound task, the words either began with the same onset, as in plea and plank, or shared only the initial phoneme, as in plea and pray. In the end-sound task, the words either shared the entire rime, as in spit and wit, or shared only the final phoneme, as in rat and wit. Treiman and Zudowski showed that four- and five-year-old children found the onset-rime version of the same/different task significantly easier than the version based on phonemes. Only the sixyear-olds, who had been learning to read for about a year, were able to perform both versions of the tasks with an equal level of success.

 

Q. 76 From the following statements, pick out the true statement according to the passage.

A. A mono-syllabic word can have only one onset.

B. A mono-syllabic word can have only one rhyme but more than one rime

C. A mono-syllabic word can have only one phoneme.

D. All of these

 

Q. 77 Which one of the following is likely to emerge last in the cognitive development of a child?

A. Rhyme

B. Rime

C. Onset

D. Phoneme

 

Q. 78 A phonological deficit in which of the following is likely to be classified as dyslexia?

A. Phonemic judgement

B. Onset judgement

C. Rime judgement

D. Any one or more of the above

 

Q. 79 The Treiman and Zudowski experiment found evidence to support which of the following conclusions?

A. At age six, reading instruction helps children perform both, the same-different judgement task.

B. The development of onset-rime awareness precedes the development of an awareness of phonemes.

C. At age four to five children find the onset-rime version of the same/different task significantly easier.

D. The development of onset-rime awareness is a necessary and sufficient condition for the development of an awareness of phonemes.

 

Q. 80 The single-syllable words Rhyme and Rime are constituted by the exact same set of

A. rime(s)

B. onset(s)

C. rhyme(s)

D. phonemes(s)

A. A and B

B. A and C

C. A, B and C

D. B, C and D

 

Questions: 81 – 84

Billie Holiday died a few weeks ago. I have been unable until now to write about her, but since she will survive many who receive longer obituaries, a short delay in one small appreciation will not harm her or us. When she died we — the musicians, critics, all who were ever transfixed by the most heart-rending voice of the past generation — grieved bitterly. There was no reason to. Few people pursed self-destruction more whole-heartedly than she, and when the pursuit was at an end, at the age of 44, she had turned herself into a physical and artistic wreck. Some of us tried gallantly to pretend otherwise, taking comfort in the occasional moments when she still sounded like a ravaged echo of her greatness. Others had not even the heart to see and listen any more. We preferred to stay home and, if old and lucky enough to own the incomparable records of her heyday from 1937 to 1946, many of which are not even available on British LP, to recreate those coarse-textured, sinuous, sensual and unbearable sad noises which gave her a sure corner of immortality. Her physical death called, if anything, for relief rather than sorrow. What sort of middle age would she have faced without the voice to earn money for her drinks and fixes, without the looks — and in her day she was hauntingly beautiful — to attract the men she needed, without business sense, without anything but the disinterested worship of ageing men who had heard and seen her in her glory? And yet, irrational though it is, our grief expressed Billie Holiday’s art, that of a woman for whom one must be sorry. The great blues singers, to whom she may be justly compared, played their game from strength. Lionesses, though often wounded or at bay (did not Bessie Smith call herself ‘a tiger, ready to jump’?), their tragic equivalents were Cleopatra and Phaedra; Holiday’s was an embittered Ophelia. She was the Puccini heroine among blues singers, or rather among jazz singers, for though she sang a cabaret version of the blues incomparably, her natural idiom was the pop song. Her unique achievement was to have twisted this into a genuine expression of the major passions by means of a total disregard of its sugary tunes, or indeed of any tune other than her own few delicately crying elongated notes, phrased like Bessie Smith or Louis Armstrong in sackcloth, sung in a thin, gritty, haunting voice whose natural mood was an unresigned and voluptuous welcome for the pains of love. Nobody has sung, or will sing, Bess’s songs from Porgy as she did. It was this combination of bitterness and physical submission, as of someone lying still while watching his legs being amputated, which gives such a blood-curdling quality to her Strange Fruit, the anti-lynching poem which she turned into an unforgettable art song. Suffering was her profession; but she did not accept it. Little need be said about her horrifying life, which she described with emotional, though hardly with factual, truth in her autobiography Lady Sings the Blues. After an adolescence in which self-respect was measured by a girl’s insistence on picking up the coins thrown to her by clients with her hands, she was plainly beyond help. She did not lack it, for she had the flair and scrupulous honesty of John Hammond to launch her, the best musicians of the 1930s to accompany her — notably Teddy Wilson, Frankie Newton and Lester Young — the boundless devotion of all serious connoisseurs, and much public success. It was too late to arrest a career of systematic embittered self-immolation. To be born with both beauty and selfrespect in the Negro ghetto of Baltimore in 1915 was too much of a handicap, even without rape at the age of 10 and drug-addiction in her teens. But, while she destroyed herself, she sang, unmelodious, profound and heartbreaking. It is impossible not to weep for her, or not to hate the world which made her what she was.

 

Q. 81 Why will Billie Holiday survive many who receive longer obituaries?

A. Because of her blues creations.

B. Because she was not as self-destructive as some other blues exponents.

C. Because of her smooth and mellow voice.

D. Because of the expression of anger in her songs.

 

Q. 82 According to the author, if Billie Holiday had not died in her middle age

A. she would have gone on to make a further mark

B. she would have become even richer than what she was when she died

C. she would have led a rather ravaged existence.

D. she would have led a rather comfortable existence.

 

Q. 83 Which of the following statements is not representative of the author’s opinion?

A. Billie Holiday had her unique brand of melody.

B. Billie Holiday’s voice can be compared to other singers in certain ways

C. Billie Holiday’s voice had a ring of profound sorrow.

D. Billie Holiday welcomed suffering in her profession and in her life

 

Q. 84 According to the passage, Billie Holiday was fortunate in all but one of which of the following ways?

A. She was fortunate to have been picked up young by an honest producer.

B. She was fortunate to have the likes of Louis Armstrong and Bessie Smith accompany her.

C. She was fortunate to possess the looks.

D. She enjoyed success among the public and connoisseurs.

 

Questions: 85 – 90

The narrative of Dersu Uzala is divided into two major sections, set in 1902, and 1907, that deal with separate expeditions which Arseniev conducts into the Ussuri region. In addition, a third time frame forms a prologue to the film. Each of the temporal frames has a different focus, and by shifting them Kurosawa is able to describe the encroachment of settlements upon the wilderness and the consequent erosion of Dersu’s way of life. As the film opens, that erosion has already begun. The first image is a long shot of a huge forest, the trees piled upon one another by the effects of the telephoto lens so that the landscape becomes an abstraction and appears like a huge curtain of green. A title informs us that the year is 1910. This is as late into the century as Kurosawa will go. After this prologue, the events of the film will transpire even farther back in time and will be presented as Arseniev’s recollections. The character of Dersu Uzala is the heart of the film, his life the example that Kurosawa wishes to affirm. Yet the formal organization of the film works to contain, to close, to circumscribe that life by erecting a series of obstacles around it. The film itself is circular, opening and closing by Dersu’s grave, thus sealing off the character from the modern world to which Kurosawa once so desperately wanted to speak. The multiple time frames also work to maintain a separation between Dersu and the contemporary world. We must go back father even than 1910 to discover who he was. But this narrative structure has yet another implication. It safeguards Dersu’s example, inoculates it from contamination with history, and protects it from contact with the industrialised, urban world. Time is organised by the narrative into a series of barriers, which enclose Dersu in a kind of vacuum chamber, protecting him from the social and historical dialectics that destroyed the other Kurosawa heroes. Within the film, Dersu does die, but the narrative structure attempts to immortalise him and his example, as Dersu passes from history into myth. We see all this at work in the enormously evocative prologue. The camera tilts down to reveal felled trees littering the landscape and an abundance of construction. Roads and houses outline the settlement that is being built. Kurosawa cuts to a medium shot of Arseniev standing in the midst of the clearing, looking uncomfortable and disoriented. A man passing in a wagon asks him what he is doing, and the explorer says he is looking for a grave. The driver replies that no one has died here, the settlement is too recent. These words enunciate the temporal rupture that the film studies. It is the beginning of things (industrial society) and the end of things (the forest), the commencement of one world so young that no one has had ime yet to die and the eclipse of another, in which Dersu had died. It is his grave for which the explorer searches. His passing symbolises the new order, the development that now surrounds Arseniev. The explorer says he buried his friend three years ago next to huge cedar and fir trees, but now they are all gone. The man on the wagon replies they were probably chopped down when the settlement was built, and he drives off. Arseniev walks to a barren, treeless spot next to a pile of bricks. As he moves, the camera tracks and pans to follow, revealing a line of freshly built houses and a woman hanging her laundry to dry. A distant train whistle is heard, and the sounds of construction in the clearing vie with the cries of birds and the rustle of wind in the trees. Arseniev pauses, looks around for the grave that once was, and murmurs desolately, ‘Dersu’. The image now cuts farther into the past, to 1902, and the first section of the film commences, which describes Arseniev’s meeting with Dersu and their friendship. Kurosawa defines the world of the film initially upon a void, a missing presence. The grave is gone, brushed aside by a world rushing into modernism, and now the hunter exists only in Arseniev’s memories. The hallucinatory dreams and visions of Dodeskaden are succeeded by nostalgic, melancholy ruminations. Yet by exploring these ruminations, the film celebrates the timelessness of Dersu’s wisdom. The first section of the film has two purposes: to describe the magnificence and in human vastness of nature and to delineate the code of ethics by which Dersu lives and which permits him to survive in these conditions. When Dersu first appears, the other soldiers treat him with condescension and laughter, but Arseniev watches him closely and does not share their derisive response. Unlike them, he is capable of immediately grasping Dersu’s extraordinary qualities. In camp, Kurosawa frames Arseniev by himself, sitting on the other side of the fire from his soldiers. While they sleep or joke among themselves, he writes in his diary and Kurosawa cuts in several point-of-view shots from his perspective of trees that appear animated and sinister as the fire light dances across their gnarled, leafless outlines. This reflective dimension, this sensitivity to the spirituality of nature, distinguishes him from the others and forms the basis of his receptivity to Dersu and their friendship. It makes him a fit pupil for the hunter.

 

Q. 85 How is Kurosawa able to show the erosion of Dersu’s way of life?

A. By documenting the ebb and flow of modernisation.

B. By going back farther and farther in time.

C. By using three different time frames and shifting them.

D. Through his death in a distant time.

 

Q. 86 Arseniev’s search for Dersu’s grave

A. is part of the beginning of the film

B. symbolises the end of the industrial society

C. is misguided since the settlement is too new.

D. symbolises the rediscovery of modernity.

 

Q. 87 The film celebrates Dersu’s wisdom

A. by exhibiting the moral vacuum of the pre-modern world

B. by turning him into a mythical figure.

C. through hallucinatory dreams and visions

D. through Arseniev’s nostalgic, melancholy ruminations.

 

Q. 88 According to the author, the section of the film following the prologue

A. serves to highlight the difficulties that Dersu faces that eventually kills him

B. shows the difference in thinking between Arseniev and Dersu

C. shows the code by which Dersu lives that allows him to survive his surroundings

D. serves to criticize the lack of understanding of nature in the pre-modern era.

 

Q. 89 In the film, Kurosawa hints at Arseniev’s reflective and sensitive nature

A. by showing him as not being derisive towards Dersu, unlike other soldiers

B. by showing him as being aloof from other soldiers

C. through shots of Arseniev writing his diary, framed by trees

D. All of these

 

Q. 90 According to the author, which of these statements about the film is correct?

A. According to the author, which of these statements about the film is correct?

B. The film highlights the insularity of Arseniev

C. The film begins with the absence of its main protagonist

D. None of these

 

Questions: 91 – 96

Democracy rests on a tension between two different principles. There is, on the one hand, the principle of equality before the law, or, more generally, of equality, and, on the other, what may be described as the leadership principle. The first gives priority to rules and the second to persons. No matter how skilfully we contrive out schemes, there is a point beyond which the one principle cannot be promoted without some sacrifice of the other. Alexis do Tocqueville, the great 19th-century writer on democracy, maintained that the age of democracy, whose birth he was witnessing, would also be the age of mediocrity, in saying this he was thinking primarily of a regime of equality governed by impersonal rules. Despite his strong attachment to democracy, he took great pains to point out what he believed to be its negative side: a dead level plane of achievement in practically every sphere of life. The age of democracy would, in his view, be an unheroic age; there would not be room in it for either heroes or hero-worshippers. But modern democracies have not been able to do without heroes: this too was foreseen, with much misgiving, by Tocqueville. Tocqueville viewed this with misgiving because he believed, rightly or wrongly, that unlike in aristocratic societies there was no proper place in a democracy for heroes and, hence, when they arose they would sooner or later turn into despots. Whether they require heroes or not, democracies certainly require leaders, and, in the contemporary age, breed them in great profusion; the problem is to know what to do with them. In a world preoccupied with scientific rationality the advantages of a system based on an impersonal rule of law should be a recommendation with everybody. There is something orderly and predictable about such a system. When life is lived mainly in small, self-contained communities, men are able to take finer personal distinctions into account in dealing with their fellow men. They are unable to do this in a large and amorphous society, and organised living would be impossible here without a system of impersonal rules. Above all, such a system guarantees a kind of equality to the extent that everybody, no matter in what station of life, is bound by the same explicit, often written, rules and nobody is above them. But a system governed solely by impersonal rules can at best ensure order and stability; it cannot create any shining vision of a future in which mere formal equality will be replaced by real equality and fellowship. A world governed by impersonal rules cannot easily change itself, or when it does, the change is so gradual as to make the basic and fundamental feature of society appear unchanges. For any kind of basic or fundamental change, a push is needed from within, a kind of individual initiative which will create new rules, new terms and conditions of life. The issue of leadership thus acquires crucial significance in the context of change. If the modern age is preoccupied with scientific rationality, it is no less preoccupied with change. To accept what exists on its own terms is traditional, not modern, and it may be all very well to appreciate tradition in music, dance and drama, but for society as a whole the choice has already been made in favour of modernisation and development. Moreover, in some countries the gap between ideal and reality has become so great that the argument for development and change is now irresistible. In these countries no argument for development has greater appeal or urgency than the one which shows development to be the condition for the mitigation, if not the elimination, of inequality. There is something contradictory about the very presence of large inequalities in a society which profess to be democratic. It does not take people too long to realise that democracy by itself can guarantee only formal equality; beyond this, it can only whet people’s appetite for real or substantive equality. From this arises their continued preoccupation with plans and schemes that will help to bridge the gap between the ideal of equality and the reality which is so contrary to it. When pre-existing rules give no clear directions of change, leadership comes into its own. Every democracy invests its leadership with a measure of charisma, and expects from it a corresponding measure of energy and vitality. Now, the greater the urge for change in a society the stronger the appeal of a dynamic leadership in it. A dynamic leadership seeks to free itself from the constraints of existing rules: in a sense that is the test of its dynamism. In this process it may take a turn at which it ceases to regard itself as being bound by these rules, placing itself above them. There is always a tension between ‘charisma’ and ‘discipline’ in the case of a democratic leadership, and when this leadership puts forward revolutionary claims, the tension tends to be resolved at the expense of discipline. Characteristically, the legitimacy of such a leadership rests on its claim to be able to abolish or at least substantially reduce the existing inequalities in society. From the argument that formal equality or equality before the law is but a limited good, it is often one short step to the argument that it is a hindrance or an obstacle to the establishment of real or substantive equality. The conflict between a ‘progressive’ executive and a ‘conservative’ judiciary is but one aspect of this larger problem. This conflict naturally acquires added piquancy when the executive is elected and the judiciary appointed.

 

Q. 91 Dynamic leaders are needed in democracies because

A. they have adopted the principles of ‘formal’ equality rather than ‘substantive’ equality.

B. ‘formal’ equality whets people’s appetite for ‘substantive’ equality

C. systems that rely on the impersonal rules of ‘formal’ equality lose their ability to make large changes.

D. of the conflict between a ‘progressive’ executive and a ‘conservative’ judiciary

 

Q. 92 What possible factor would a dynamic leader consider a ‘hindrance’ in achieving the development goals of a nation?

A. Principle of equality before the law

B. Judicial activism

C. A conservative judiciary

D. Need for discipline

 

Q. 93 Which of the following four statements can be inferred from the above passage?

A. Scientific rationality is an essential feature of modernity.

B. Scientific rationality results in the development of impersonal rules.

C. Modernisation and development have been chosen over traditional music, dance and drama.

D. Democracies aspire to achieve substantive equality

A. A, B, D but not C

B. A, B but not C, D

C. A, D but not B, C

D. A, B, C but not D

 

Q. 94 Tocqueville believed that the age of democracy would be an un-heroic age because

A. democractic principles do not encourage heroes.

B. there is no urgency for development in democratic countries.

C. heroes that emerged in democracies would become despots.

D. aristocratic society had a greater ability to produce heroes.

 

Q. 95 A key argument the author is making is that

A. n the context of extreme inequality, the issue of leadership has limited significance

B. democracy is incapable of eradicating inequality.

C. formal equality facilitates development and change.

D. impersonal rules are good for avoiding instability but fall short of achieving real equality

 

Q. 96 Which of the following four statements can be inferred from the above passage?

A. There is conflict between the pursuit of equality and individuality.

B. The disadvantages of impersonal rules can be overcome in small communities.

C. Despite limitations, impersonal rules are essential in large systems.

D. Inspired leadership, rather than plans and schemes, is more effective in bridging inequality

A. B, D but not A, C

B. A, B but not C, D

C. A, D but not B, C

D. A, C but not B, D

 

Questions: 97 – 100

In the modern scientific story, light was created not once but twice. The first time was in the Big Bang, when the universe began its existence as a glowing, expanding, fireball, which cooled off into darkness after a few million years. The second time was hundreds of millions of years later, when the cold material condensed into dense suggests under the influence of gravity, and ignited to become the first stars. Sir Martin Rees, Britain’s astronomer royal, named the long interval between these two enlightements the cosmic ‘Dark Age’. The name describes not only the poorly lit conditions, but also the ignorance of astronomers about that period. Nobody knows exactly when the first stars formed, or how they organised themselves into galaxies — or even whether stars were the first luminous objects. They may have been preceded by quasars, which are mysterious, bright spots found at the centres of some galaxies. Now two independent groups of astronomers, one led by Robert Becker of the University of California, Davis, and the other by George Djorgovski of the Caltech, claim to have peered far enough into space with their telescopes (and therefore backwards enough in time) to observe the closing days of the Dark age. The main problem that plagued previous efforts to study the Dark Age was not the lack of suitable telescopes, but rather the lack of suitable things at which to point them. Because these events took place over 13 billion years ago, if astronomers are to have any hope of unravelling them they must study objects that are at least 13 billion light years away. The best prospects are quasars, because they are so bright and compact that they can be seen across vast stretches of space. The energy source that powers a quasar is unknown, although it is suspected to be the intense gravity of a giant black hole. However, at the distances required for the study of Dark Age, even quasars are extremely rare and faint. Recently some members of Dr Becker’s team announced their discovery of the four most distant quasars known. All the new quasars are terribly faint, a challenge that both teams overcame by peering at them through one of the twin Keck telescopes in Hawaii. These are the world’s largest, and can therefore collect the most light. The new work by Dr Becker’s team analysed the light from all four quasars. Three of them appeared to be similar to ordinary, less distant quasars. However, the fourth and most distant, unlike any other quasar ever seen, showed unmistakable signs of being shrouded in a fog because new-born stars and quasars emit mainly ultraviolet light, and hydrogen gas is opaque to ultraviolet. Seeing this fog had been the goal of would-be Dark Age astronomers since 1965, when James Gunn and Bruce Peterson spelled out the technique for using quasars as backlighting beacons to observe the fog’s ultraviolet shadow. The fog prolonged the period of darkness until the heat from the first stars and quasars had the chance to ionise the hydrogen (breaking it into its constituent parts, protons and electrons). Ionised hydrogen is transparent to ultraviolet radiation, so at that moment the fog lifted and the universe became the well-lit place it is today. For this reason, the end of the Dark Age is called the ‘Epoch of Re-ionisation’. Because the ultraviolet shadow is visible only in the most distant of the four quasars, Dr Becker’s team concluded that the fog had dissipated completely by the time the universe was about 900 million years old, and one seventh of its current size

 

Q. 97 In the passage, the Dark Age refers to

A. the period when the universe became cold after the Big Bang.

B. a period about which astronomers know very little.

C. the medieval period when cultural activity seemed to have come to an end.

D. the time that the universe took to heat up after the Big Bang.

 

Q. 98 the time that the universe took to heat up after the Big Bang.

A. suitable telescopes are few.

B. the associated events took place aeons ago.

C. the energy source that powers a quasars is unknown

D. their best chance is to study quasars, which are faint objects to begin with

 

Q. 99 The four most distant quasars discovered recently

A. could only be seen with the help of large telescopes

B. appear to be similar to other ordinary, quasars.

C. appear to be shrouded in a fog of hydrogen gas.

D. have been sought to be discovered by Dark Age astronomers since 1965

 

Q. 100 The fog of hydrogen gas seen through the telescopes

A. is transparent to hydrogen radiation from stars and quasars in all states.

B. was lifted after heat from starts and quasars ionised it

C. is material which eventually became stars and quasars

D. is broken into constituent elements when stars and quasars are formed

 

Questions: 101 – 104

Answer the questions based on the table given below. The following table describes garments manufactured based upon the colour and size for each lay.

There are four sizes: M – medium, L – large, XL – extra large and XXL – extra extra large. There are three colours: yellow, red and white.

 

Q. 101 How many lays are used to produce yellow fabrics?

A. 10

B. 11

C. 12

D. 14

 

Q. 102 How many lays are used to produce yellow fabrics?

A. 15

B. 16

C. 17

D. 18

 

Q. 103 How many lays are used to produce XL yellow or XL white fabrics?

A. 8

B. 9

C. 10

D. 15

 

Q. 104 How many varieties of fabrics, which exceed the order, have been produced?

A. 3

B. 4

C. 5

D. 6

 

Questions: 105 – 108

Answer the questions based on the table given below concerning the busiest 20 international airports in the world.

 

Q. 105 How many international airports of type ‘A’ account for more than 40 million passengers?

A. 4

B. 5

C. 6

D. 7

 

Q. 106 What percentage of top ten busiest airports is in the United States of America?

A. 60%

B. 80%

C. 70%

D. 90%

 

Q. 107 Of the five busiest airports, roughly, what percentage of passengers in handled by Heathrow Airport?

A. 30

B. 40

C. 20

D. 50

 

Q. 108 Of the five busiest airports, roughly, what percentage of passengers in handled by Heathrow Airport?

A. 5

B. 6

C. 10

D. 14

 

Questions: 109 – 114

Answer the questions based on the two graphs shown below. Figure I shows the amount of work distribution, in man-hours, for a software company between offshore and onsite activities. Figure 2 shows the estimated and actual work effort involved in the different offshore activities in the same company during the same period. [Note: Onsite refers to work performed at the customer’s premise and offshore refers to work performed at the developer’s premise.]

 

Q. 109 Which work requires as many man-hours as that spent in coding?

A. Offshore, design and coding

B. Offshore coding

C. Testing

D. Offshore, testing and coding

 

Q. 110 Roughly, what percentage of the total work is carried out onsite?

A. 40%

B. 20%

C. 30%

D. 10%

 

Q. 111 The total effort in man-hours spent onsite is nearest to which of the following?

A. The sum of the estimated and actual effort for offshore design.

B. The estimated man-hours of offshore coding.

C. The actual man-hours of offshore testing

D. Half of the man-hours of estimated offshore coding

 

Q. 112 If the total working hours were 100, which of the following tasks will account for approximately 50 hr?

A. Coding

B. Design

C. Offshore testing

D. Offshore testing plus design

 

Q. 113 If 50% of the offshore work were to be carried out onsite, with the distribution of effort between the tasks remaining the same, the proportion of testing carried out offshore would be

A. 40%

B. 30%

C. 50%

D. 70%

 

Q. 114 If 50% of the offshore work were to be carried out onsite, with the distribution of effort between the tasks remaining the same, which of the following is true of all work carried out onsite?

A. The amount of coding done is greater than that of testing.

B. The amount of coding done onsite is less than that of design done onsite

C. The amount of design carried out onsite is greater than that of testing

D. The amount of testing carried out offshore is greater than that of total design

 

Questions: 115 – 117

Answer the questions based on the pipeline diagram below.

The following sketch shows the pipelines carrying material from one location to another. Each location has a demand for material. The demand at Vaishali is 400, at Jyotishmati is 400, at Panchal is 700, and at Vidisha is 200. Each arrow indicates the direction of material flow through the pipeline. The flow from Vaishali to Jyotishmati is 300. The quantity of material flow is such that the demands at all these locations are exactly met. The capacity of each pipeline is 1,000.

 

Q. 115 The quantity moved from Avanti to Vidisha is

A. 200

B. 800

C. 700

D. 1000

 

Q. 116 The quantity moved from Avanti to Vidisha is

A. 0

B. 100

C. 200

D. 300

 

Q. 117 What is the free capacity available in the Avanti-Vidisha pipeline?

A. 300

B. 200

C. 100

D. 0

 

Questions: 118 – 120

Answer these questions based on the data given below:

There are six companies, 1 through 6. All of these companies use six operations, A through F. The following graph shows the distribution of efforts put in by each company in these six operations.

 

Q. 118 Suppose effort allocation is inter-changed between operations B and C, then C and D, and then D and E. If companies are then ranked in ascending order of effort in E, what will be the rank of company 3?

A. 2

B. 3

C. 4

D. 5

 

Q. 119 A new technology is introduced in company 4 such that the total effort for operations B through F get evenly distributed among these. What is the change in the percentage of effort in operation E?

A. Reduction of 12.3

B. Increase of 12.3

C. Reduction of 5.6

D. Increase of 5.6

 

Q. 120 Suppose the companies find that they can remove operations B, C and D and redistribute the effort released equally among the remaining operations. Then which operation will show the maximum across all companies and all operations?

A. Operation E in company 1

B. Operation E in company 4

C. Operation F in company 5

D. Operation E in company 5

 

Questions: 121 – 127

a. if the question can be answered by one of the statements alone and not by the other.

b. if the question can be answered by using either statement alone.

c. if the question can be answered by using both the statements together, but cannot be answered by using either statement alone.

d. if the question cannot be answered even by using both statements together.

 

Q. 121 What are the values of m and n?

I. n is an even integer, m is an odd integer, and m is greater than n.

II. Product of m and n is 30.

A. a

B. b

C. c

D. d

 

Q. 122 Is Country X’s GDP higher than country Y’s GDP?

I. GDPs of the countries X and Y have grown over the past 5 years at compounded annual rate of 5% and 6% respectively.

II. Five years ago, GDP of country X was higher than that of country Y

A. a

B. b

C. c

D. d

 

Q. 123 What is the value of X?

I. X and Y are unequal even integers, less than 10, and

X/Y is an odd integer.

II. X and Y are even integers, each less than 10, and product of X and Y is 12

A. a

B. b

C. c

D. d

 

Q. 124 On a given day a boat ferried 1,500 passengers across the river in 12 hr. How many round trips did it make?

I. The boat can carry 200 passengers at any time.

II. It takes 40 min each way and 20 min of waiting time at each terminal.

A. a

B. b

C. c

D. d

 

Q. 125 What will be the time for downloading software?

I. Transfer rate is 6 kilobytes per second.

II. The size of the software is 4.5 megabytes.

A. a

B. b

C. c

D. d

 

Q. 126 A square is inscribed in a circle. What is the difference between the area of the circle and that of the square?

I. The diameter of the circle is 25 √2 cm.

II. The side of the square is 25 cm.

A. a

B. b

C. c

D. d

 

Q. 127 Two friends, Ram and Gopal, bought apples from a wholesale dealer. How many apples did they buy?

I. Ram bought one-half the number of apples that Gopal bought.

II. The wholesale dealer had a stock of 500 apples

A. a

B. b

C. c

D. d

 

Questions: 128 – 130

Answer the questions based on the pie charts given below.

Chart 1 shows the distribution of 12 million tonnes of crude oil transported through different modes over a specific period of time. Chart 2 shows the distribution of the cost of transporting this crude oil. The total cost was Rs. 30 million.

 

Q. 128 The cost in rupees per tonne of oil moved by rail and road happens to be roughly

A. Rs. 3

B. Rs. 1.5

C. Rs. 4.5

D. Rs. 8

 

Q. 129 From the charts given, it appears that the cheapest mode of transport is

A. road

B. rail

C. pipeline

D. ship

 

Q. 130 If the costs per tonne of transport by ship, air and road are represented by P, Q and R respectively, which of the following is true?

A. R > Q > P

B. P > R > Q

C. P > Q > R

D. R > P > Q

 

Q. 131 At a village mela, the following six nautankis (plays) are scheduled as shown in the table below.You wish to see all the six nautankis. Further, you wish to ensure that you get a lunch break from 12.30 p.m. to 1.30 p.m. Which of the following ways can you do this?

A. Sati Savitri is viewed first; SundarKand is viewed third, and Jhansi ki Rani is viewed last

B. Sati Savitri is viewed last; Veer Abhimanyu is viewed third, and Reshma aur Shera is viewed first

C. Sati Savitri is viewed first; Sundar Kand is viewed third, and Joru ka Ghulam is viewed fourth

D. Veer Abhimanyu is viewed third; Reshma aur Shera is viewed fourth, and Jahansi ki Rani is viewed fifth

 

Q. 132 Mrs Ranga has three children and has difficulty remembering their ages and months of their birth.The clue below may help her remember.

. The boy, who was born in June, is 7 years old.

. One of the children is 4 years old but it was not Anshuman.

. Vaibhav is older than Suprita.

. One of the children was born in September, but it was not Vaibhav.

. Suprita’s birthday is in April.

. The youngest child is only 2-year-old.

Based on the above clues, which one of the following statements is true?

A. Vaibhav is the oldest, followed by Anshuman who was born in September, and the youngest is Suprita who was born in April

B. Anshuman is the oldest being born in June, followed by Suprita who is 4-year-old, and the youngest is Vaibhav who is 2-year-old

C. Vaibhav is the oldest being 7-year-old, followed by Suprita who was born in April, and the youngest is Anshuman who was born in September

D. Suprita is the oldest who was born in April, followed by Vaibhav who was born in June, and Anshuman who was born in September

 

Q. 133 The Bannerjees, the Sharmas, and the Pattabhiramans each have a tradition of eating Sunday lunch as a family. Each family serves a special meal at a certain time of day. Each family has a particular set of chinaware used for this meal. Use the clues below to answer the following question.

. The Sharma family eats at noon.

. The family that serves fried brinjal uses blue chinaware.

. The Bannerjee family eats at 2 o’clock.

. The family that serves sambar does not use red chinaware.

. The family that eats at 1 o’clock serves fried brinjal.

. The Pattabhiraman family does not use white chinaware.

. The family that eats last likes makkai-ki-roti.

Which one of the following statements is true?

A. The Bannerjees eat makkai-ki-roti at 2 o’clock, the Sharmas eat fried brinjal at 12 o’clock and the Pattabhiramans eat sambar from red chinaware

B. The Sharmas eat sambar served in white chinaware, the Pattabhiramans eat fried brinjal at 1 o’clock, and the Bannerjees eat makkai-ki-roti served in blue chinaware

C. The Sharmas eat sambar at noon, the Pattabhiramans eat fried brinjal served in blue chinaware, and the Bannerjees eat makkai-ki-roti served in red chinaware

D. The Bannerjees eat makkai-ki-roti served in white chinaware, the Sharmas eat fried brinjal at 12 o’clock and the Pattabhiramans eat sambar from red chinaware

 

Q. 134 While Balbir had his back turned, a dog ran into his butcher shop, snatched a piece of meat off the counter and ran out. Balbir was mad when he realised what had happened. He asked three other shopkeepers, who had seen the dog, to describe it. The shopkeepers really did not want to help Balbir. So each of them made a statement which contained one truth and one lie.

. Shopkeeper number 1 said: “The dog had black hair and a long tail.”

. Shopkeeper number 2 said: “The dog had a short tail and wore a collar.”

. Shopkeeper number 3 said: “The dog had white hair and no collar.”

Based on the above statements, which of the following could be a correct description?

A. The dog had white hair, short tail and no collar

B. The dog had white hair, long tail and a collar

C. The dog had black hair, long tail and a collar

D. The dog had black hair, long tail and no collar

 

Questions: 135 – 136

Elle is three times older than Yogesh. Zaheer is half the age of Wahida. Yogesh is older than Zaheer

 

Q. 135 Which of the following can be inferred?

A. Yogesh is older than Wahida

B. Elle is older than Wahida

C. Elle may be younger than Wahida

D. None of these

 

Q. 136 Which of the following information will be sufficient to estimate Elle’s age?

A. Zaheer is 10-year-old

B. Both Yogesh and Wahida are older than Zaheer by the same number of years

C. Both (a) and (b)

D. None of these

 

Questions: 137 – 139

A group of three or four has to be selected from seven persons. Among the seven are two women: Fiza and Kavita, and five men: Ram, Shyam, David, Peter and Rahim. Ram would not like to be in the group If Shyam is also selected. Shyam and Rahim want to be selected together in the group. Kavita would like to be in the group only if David is also there. David, if selected, would not like Peter in the group. Ram would like to be in the group only if Peter is also there. David insists that Fiza be selected in case he is there in the group

 

Q. 137 Which of the following is a feasible group of three?

A. David, Ram and Rahim

B. Peter, Shyam and Rahim

C. Kavita, David and Shyam

D. Fiza, David and Ram

 

Q. 138 Which of the following is a feasible group in four?

A. Ram, Peter, Fiza and Rahim

B. Shyam, Rahim, Kavita and David

C. Shyam, Rahim, Fiza and David

D. Fiza, David, Ram and Peter

 

Q. 139 Which of the following statements is true?

A. Kavita and Ram can be part of a group of four

B. A group of four can have two women

C. A group of four can have all four men

D. None of these

 

Q. 140 On her walk through the park, Hamsa collected 50 coloured leaves, all either maple or oak. She sorted them by category when she got home, and found the following:

The number of red oak leaves with spots is even and positive.

The number of red oak leaves without any spot equals the number of red maple leaves without spots.

All non-red oak leaves have spots, and there are five times as many of them as there are red spotted oak leaves.

There are no spotted maple leaves that are not red.

There are exactly 6 red spotted maple leaves.

There are exactly 22 maple leaves that are neither spotted nor red.

How many oak leaves did she collect?

A. 22

B. 17

C. 25

D. 18

 

Q. 141  Eight people carrying food baskets are going for a picnic on motorcycles. Their names are A, B, C, D, E, F, G, and H. They have 4 motorcycles M1, M2, M3 and M4 among them. They also have 4 food baskets O, P, Q and R of different sizes and shapes and each can be carried only on motorcycles M1, M2, M3 and M4 respectively. No more than 2 persons can travel on a motorcycle and no more than one basket can be carried on a motorcycle. There are 2 husband-wife pairs in this group of 8 people and each pair will ride on a motorcycle together. C cannot travel with A or B. E cannot travel with B or F. G cannot travel with F, or H, or D. The husband-wife pairs must carry baskets O and P. Q is with A and P is with D. F travels on M1 and E travels on M2 motorcycles. G is with Q, and B cannot go with R. Who is travelling with H?

A. A

B. B

C. C

D. D

 

Q. 142 In a family gathering there are 2 males who are grandfathers and 4 males who are fathers. In the same gathering there are 2 females who are grandmothers and 4 females who are mothers. There is at least one grandson or a granddaughter present in this gathering. There are 2 husband-wife pairs in this group. These can either be a grandfather and a grandmother, or a father and a mother. The single grandfather (whose wife is not present) has 2 grandsons and a son present. The single grandmother (whose husband is not present) has 2 grand daughters and a daughter present. A grandfather or a grandmother present with their spouses does not have any grandson or granddaughter present.What is the minimum number of people present in this gathering?

A. 10

B. 12

C. 14

D. 16

 

Q. 143 I have a total of Rs. 1,000. Item A costs Rs. 110, item B costs Rs. 90, item C costs Rs. 70, item D costs Rs. 40 and item E costs Rs. 45. For every item D that I purchase, I must also buy two of item B. For every item A, I must buy one of item C. For every item E, I must also buy two of item D and one of item B. For every item purchased I earn 1,000 points and for every rupee not spent I earn a penalty of 1,500 points. My objective is to maximise the points I earn.

What is the number of items that I must purchase to maximise my points?

A. 13

B. 14

C. 15

D. 16

 

Q. 144 Four friends Ashok, Bashir, Chirag and Deepak are out for shopping. Ashok has less money than three times the amount that Bashir has. Chirag has more money than Bashir. Deepak has an amount equal to the difference of amounts with Bashir and Chirag. Ashok has three times the money with Deepak. They each have to buy at least one shirt, or one shawl, or one sweater, or one jacket that are priced Rs. 200, Rs. 400, Rs. 600, and Rs. 1,000 a piece respectively. Chirag borrows Rs. 300 from Ashok and buys a jacket. Bashir buys a sweater after borrowing Rs. 100 from Ashok and is left with no money. Ashok buys three shirts. What is the costliest item that Deepak could buy with his own money?

A. A shirt

B. A shawl

C. A sweater

D. A jacket

 

Q. 145 In a ‘keep-fit’ gymnasium class there are 15 females enrolled in a weight-loss programme. They all have been grouped in any one of the five weight-groups W1, W2, W3, W4, or W5. One instructor is assigned to one weight-group only. Sonali, Shalini, Shubhra and Shahira belong to the same weightgroup. Sonali and Rupa are in one weight-group, Rupali and Renuka are also in one weight-group. Rupa, Radha, Renuka, Ruchika, and Ritu belong to different weight-groups. Somya cannot be with Ritu, and Tara cannot be with Radha. Komal cannot be with Radha, Somya, or Ritu. Shahira is in W1 and Somya is in W4 with Ruchika. Sweta and Jyotika cannot be with Rupali, but are in a weightgroup with total membership of four. No weight-group can have more than five or less than one member. Amita, Babita, Chandrika, Deepika and Elina are instructors of weight-groups with membership sizes 5, 4, 3, 2 and 1 respectively. Who is the instructor of Radha?

A. Babita

B. Elina

C. Chandrika

D. Deepika

 

Q. 146 A king has unflinching loyalty from eight of his ministers M1 to M8, but he has to select only four to make a cabinet committee. He decides to choose these four such that each selected person shares a liking with at least one of the other three selected. The selected persons must also hate at least one of the likings of any of the other three persons selected.

M1 likes fishing and smoking, but hates gambling.

M2 likes smoking and drinking, but hates fishing.

M3 likes gambling, but hates smoking,

M4 likes mountaineering, but hates drinking,

M5 likes drinking, but hates smoking and mountaineering.

M6 likes fishing, but hates smoking and mountaineering.

M7 likes gambling and mountaineering, but hates fishing.

M8 likes smoking and gambling, but hates mountaineering.

Who are the four people selected by the king?

A. M1, M2, M5 and M6

B. M3, M4, M5 and M6

C. M4, M5, M6 and M8

D. M1, M2, M4 and M7

 

Questions: 147 – 150

Answer the questions based on the following information.

A and B are two sets (e.g. A = Mothers, B = Women). The elements that could belong to both the sets (e.g. women who are mothers) is given by the set C = A . B. The elements which could belong to either A or B, or both, is indicated by the set D = A ∪ B . A set that does not contain any elements is known as a null set represented by ϕ (e.g. if none of the women in the set B is a mother, then C = A .B is a null set, or C = ϕ ). Let ‘V’ signify the set of all vertebrates, ‘M’ the set of all mammals, ‘D’ dogs, ‘F’ fish, ‘A’ alsatian and ‘P’, a dog named Pluto.

 

Q. 147 Given that X = M .D is such that X = D. Which of the following is true?

A. All dogs are mammals

B. Some dogs are mammals

C. X = ϕ

D. All mammals are dogs

 

Q. 148 If Y = F . (D . V) is not a null set, it implies that

A. all fish are vertebrates

B. all dogs are vertebrates

C. some fish are dogs

D. None of these

 

Q. 149 If Z = (P . D) ∪ M, then

A. the elements of Z consist of Pluto, the dog, or any other mammal

B. Z implies any dog or mammal

C. Z implies Pluto or any dog that is a mammal

D. Z is a null set

 

Q. 150 If P . A = ϕ and P ∪ A = D, then which of the following is true?

A. Pluto and alsatians are dogs

B. Pluto is an alsatian

C. Pluto is not an alsatian

D. D is a null set

 

 

Answer Sheet
Question 1 2 3 4 5 6 7 8 9 10
Answer C B A D A C B A D A
Question 11 12 13 14 15 16 17 18 19 20
Answer A D A C B D D D A D
Question 21 22 23 24 25 26 27 28 29 30
Answer C B D C B A C C A C
Question 31 32 33 34 35 36 37 38 39 40
Answer C A A C A B C D B B
Question 41 42 43 44 45 46 47 48 49 50
Answer C C D B A B B C B B
Question 51 52 53 54 55 56 57 58 59 60
Answer A D C B D A A C D B
Question 61 62 63 64 65 66 67 68 69 70
Answer D C B C B C D A D A
Question 71 72 73 74 75 76 77 78 79 80
Answer A C A D B A D D B B
Question 81 82 83 84 85 86 87 88 89 90
Answer A C D B C A D C D C
Question 91 92 93 94 95 96 97 98 99 100
Answer C C A A D D B B A B
Question 101 102 103 104 105 106 107 108 109 110
Answer D B D A A A C B A C
Question 111 112 113 114 115 116 117 118 119 120
Answer C A B A D D D B A D
Question 121 122 123 124 125 126 127 128 129 130
Answer C D A A A C D B A C
Question 131 132 133 134 135 136 137 138 139 140
Answer C C C B B C B C D B
Question 141 142 143 144 145 146 147 148 149 150
Answer C B B B B D A C A C

 

×

Hello!

Click one of our representatives below to chat on WhatsApp or send us an email to info@vidhyarthidarpan.com

×